Practical Guide to Visualizing Medicine: A Self-Assessment Manual [1st ed. 2023] 3031244648, 9783031244643

This book provides a practically orientated resource that details the use a range of imaging techniques across major spe

219 40 36MB

English Pages 488 [472] Year 2023

Report DMCA / Copyright

DOWNLOAD FILE

Polecaj historie

Practical Guide to Visualizing Medicine: A Self-Assessment Manual [1st ed. 2023]
 3031244648, 9783031244643

Table of contents :
Foreword
Preface
Picture Credits
Contents
About the Editors
1 Introduction: Cultivation of Clinical Reasoning
Introduction
Development of “Clinical Sense” or Clinical Reasoning Skills from Diagnosis to Management
Conclusions
Reference
2 Top Tips for Image Interpretation
Conclusion
Recommended Further Reading
3 Dermatology
Basic Structure and Function of Skin with Immuno-Pathogenetic Mechanisms Causing Skin Disease
Inflammatory Skin Disease
Dermatological Emergencies
Skin Infections
Systemic Diseases with Dermatological Manifestations
Benign and Malignant Skin Tumors
Hair and Nails
References
Further Reading
4 Endocrinology
Further Readings
5 Breast Disease
Further Reading
6 Pediatrics
Further Readings
7 Cardiology
References
8 Respiratory Medicine
References
9 Anesthesia and Intensive Care
Anesthesia and Pain Management
Intensive Care Medicine
Further Reading
10 Cardiothoracic Surgery
Cardiac Surgery
Thoracic Surgery
Recommended Further Reading
11 Gastrointestinal Surgery, Gastroenterology and Hepatology
References
12 Neurology and Neurosurgery
Neurology
Neurosurgery
References
13 Geriatric Medicine
References
Further Reading
14 Renal and Transplant Medicine and Urology
Acknowledgements
Further Reading
15 Trauma and Orthopedics
References
16 Rheumatology
Recommended Reading
17 Infectious Diseases
Further Readings
18 Ophthalmology
Pre-study Questions
Main Section
References
19 ENT and Head and Neck Surgery
Learning Outcomes
Ear
Nose
Throat
Emergencies and Complications
References
20 Obstetrics & Gynecology
Topic 1: Antenatal Screening
Topic 2: Gestational Hypertension and Pre-eclampsia
Topic 3: Preterm Prelabor Rupture of Membrane (PPROM)
Topic 4: Preterm Birth
Topic 5: Pregnancy of Unknown Location (PUL), Miscarriage and Ectopic Pregnancy
Topic 6: Sexually Transmitted Infections (STI)
Topic 7: Heavy Menstrual Bleeding (HMB)
Further Reading
21 Immunology
Pre-study Questions
Further Readings
22 Hematology
Basic Blood Count and Morphology
Pre-study MCQs
Main Questions
Further Readings
23 Palliative Care
Conclusions
References
24 Vascular Diseases
Pre-study Questions
Tailoring Therapy to the Individual/Whole Person Care
Other Vascular Diseases
References
Appendix_1
Appendix_2
Index

Citation preview

Kenneth Wong · Shernaz Walton · Simi Sudhakaran · John Cookson   Editors

Practical Guide to Visualizing Medicine A Self-Assessment Manual

Practical Guide to Visualizing Medicine

Kenneth Wong  Shernaz Walton Simi Sudhakaran  John Cookson



Editors

Practical Guide to Visualizing Medicine A Self-Assessment Manual

123

Editors Kenneth Wong Blackpool Teaching Hospitals NHS Foundation Trust Blackpool, UK Simi Sudhakaran Modality Partnership, Hull Hull University Teaching Hospitals NHS Trust Hull, UK

Shernaz Walton Hull University Teaching Hospitals NHS Trust Hull, UK John Cookson University of Portsmouth Portsmouth, UK

This work contains media enhancements, which are displayed with a “play” icon. Material in the print book can be viewed on a mobile device by downloading the Springer Nature “More Media” app available in the major app stores. The media enhancements in the online version of the work can be accessed directly by authorized users. ISBN 978-3-031-24464-3 ISBN 978-3-031-24465-0 https://doi.org/10.1007/978-3-031-24465-0

(eBook)

© The Editor(s) (if applicable) and The Author(s), under exclusive license to Springer Nature Switzerland AG 2023 This work is subject to copyright. All rights are solely and exclusively licensed by the Publisher, whether the whole or part of the material is concerned, specifically the rights of translation, reprinting, reuse of illustrations, recitation, broadcasting, reproduction on microfilms or in any other physical way, and transmission or information storage and retrieval, electronic adaptation, computer software, or by similar or dissimilar methodology now known or hereafter developed. The use of general descriptive names, registered names, trademarks, service marks, etc. in this publication does not imply, even in the absence of a specific statement, that such names are exempt from the relevant protective laws and regulations and therefore free for general use. The publisher, the authors, and the editors are safe to assume that the advice and information in this book are believed to be true and accurate at the date of publication. Neither the publisher nor the authors or the editors give a warranty, expressed or implied, with respect to the material contained herein or for any errors or omissions that may have been made. The publisher remains neutral with regard to jurisdictional claims in published maps and institutional affiliations. This Springer imprint is published by the registered company Springer Nature Switzerland AG The registered company address is: Gewerbestrasse 11, 6330 Cham, Switzerland

In loving memory of Mr. Mubarak A. Chaudhry, Consultant Cardiothoracic surgeon and author of the chapter on Cardiothoracic Surgery who died shortly after completing his chapter.

Foreword

Try giving a talk to some dermatologists without any images of the skin and you will have rotten eggs thrown at you. That is because dermatologists live in a highly visual world— whether in face to face or remote consultations. But the concept of a visual world is not unique to dermatology, as all attempts to improve someone’s health are concerned with creating a mental picture of what is going on. When a patient starts telling you their story, an image of what might be going on is formed, which is then confirmed or refuted by questions and examination of the body supplemented by tests—the so-called hypothetico-deductive approach. Such mental images are holistic and encompass a spectrum from micro to macro starting with underlying cellular dysfunction, progressing to organ pathology, to effects such as impairment of quality of life, disability, and the handicap that the disease and society might confer upon that person. Disease is not an entity that “attacks” people like some demon, but a myriad of presentations that result from a person’s interaction with potential disease processes. Only when you understand and can visualize the whole patient’s problem and how it integrates with the person and family in front of you can you begin to share some decisions on a healing journey. This highly interactive textbook written by a very experienced and multidisciplinary group will take you the steps on that visualization journey. Most important of all, apply your learning to your patients so that your skills result in human benefit. Nottingham, UK

Hywel C. Williams OBE DScFMedSci NIHR Senior Investigator Emeritus Professor of Dermato-Epidemiology Co-Director, Center of Evidence-Based Dermatology Nottingham University Hospitals NHS Trust

vii

Preface

A unique feature of this book is that it covers an extraordinary amount of breadth and depth of a range of specialties across medicine, surgery, pediatrics, obstetrics, and gynecology, with an emphasis on images and clinical data interpretation. The reason why this is possible is because a picture often speaks more than a thousand words. Images can convey information in a far more powerful and convincing way than words can. The main philosophy behind writing the book is to let the pictures speak for themselves, thus we have kept the text to a minimum of what is required to help students learn from the images. A question-and-answer style not only helps student learning by promoting interest, but it also helps recreate the best a textbook can the actual “environment” when an X-ray, an ECG, a scan, or a histopathology slide might crop up in real-life clinical practice. So when the student encounters a similar X-ray or other “image of disease” in “real life”, there is a higher chance he or she may remember the image and knowledge that is associated with it. The content covers the breadth of the material relevant for medical students and physician associates, including some information which will be useful for doctors preparing for postgraduate exams, and physician associates working in specialty areas, although the primary aim of including such material is to help the high-flying student shine in exams, to maximize their chances of achieving distinction, as well as to satisfy their intellectual desire to go the extra mile and learn more to help more patients. While we acknowledge that it is not realistic to produce a book with encyclopedic details of the whole spectrum of health and disease, a large number of important common conditions and some of the rarer but potentially life-threatening conditions are included. Further, we have included the discussion of several rarer conditions which illustrate useful pathophysiological principles. The readers are advised to use this book as a guide to help them identify areas of weakness which require more study from comprehensive textbooks. For the strategic learners, our Question-and-Answer approach covers a range of questions which you may encounter on ward rounds/clinics, or in your exams, e.g., multiple choice questions (MCQs) testing learning outcomes discussed at the start of each chapter. Many core case presentations will be explored in detail throughout the book, e.g., chest pain, breathlessness, and skin complaints. Also, an outstanding feature of this book is that we sought deliberately to include skin conditions from diverse ethnic groups. To further increase the appeal of this book, we have asked Professors, Senior lecturers, and Consultants across Hull York Medical School (HYMS) and Hull University Teaching Hospitals NHS Trust and beyond to contribute some words of “wisdom” that stem from years of experience which may not be easily found in standard texts. We believe general practitioners (family doctors), physician associates, nurse practitioners, junior doctors, and senior colleagues from other specialties would find them helpful in practice. We have included highlight boxes entitled “Mistakes/Pitfalls to avoid”. There are “Top tips for clinical data interpretation” sections throughout the book (e.g., ECG interpretation, blood test abnormalities), in addition to a dedicated chapter entitled “Top tips for image interpretation”, with advice on how not to miss subtle yet potentially life-threatening abnormalities. The use of a “star” system enables the reader to look for questions that are particularly relevant to their level of training: ix

x

Preface

(1*) Basic knowledge for the student starting on the journey of life-long learning in medicine (Anatomy\physiology). (2*) Sound integration of key basic science knowledge with clinical sciences, with an emphasis on clinical-pathological correlations. (3*) Level of knowledge and (picture or spot) diagnostic skill expected of an MBBS student/physician associate graduate. (4*) Depth of knowledge that will be useful for postgraduate exams or for the outstanding student to achieve distinction. Some physician associates who work in the specialty may also find these questions helpful. (5*) Specialist knowledge /”wisdom” from specialist colleagues with years of experience, which may not be easily found in other standard textbooks. Some physician associates who work in the specialty may also find these questions helpful. Thus, the reader can build on existing knowledge in a spiral learning fashion. While most questions are focused on medical school/physician associate graduate level (1–3*), authors were invited to include a few 4–5* questions to benefit the high-flying medical/physician associate student, physician associates and advanced nurse practitioners in specialty areas, and doctors preparing for postgraduate exams. In addition, we have appended a special section that provides a glimpse of a working week in the life of a Consultant (Specialist)/General Practitioner (Family Physician)/Advanced Nurse Practitioner/Physician Associate, and a medical student’s journey. We trust this will inspire 6th formers (senior high school) or other students interested in training to become healthcare professionals, medical students, and postgraduates. To further encourage students to visualize disease processes, an accompanying ONLINE E-VERSION will include educational videos. For instance, what better way is there to help students “visualize” heart murmurs than showing them an echocardiogram video of the disease process? The overall result is a book that is up-to-date, full of clinical common sense and wisdom, including specialties that are less well represented in other textbooks such as geriatric medicine, palliative care, and cardiothoracic surgery. We hope you find some of the cases challenging, but benefit from the teaching in this book, and be inspired to further study. We hope you enjoy learning from the high-quality images. Many of the images are “home-grown” by enthusiastic tutors who have dedicated much of their free time to look out for suitable images in health and disease. Other images have been obtained with permission from websites or other texts. We hope our endeavor has proved successful in covering the breadth of an extraordinary range of images in health and disease, with unique valuable insights from many of the specialist authors. Blackpool, UK Hull, UK Hull, UK Portsmouth, UK

Kenneth Wong Shernaz Walton Simi Sudhakaran John Cookson

Acknowledgements

Professor John Lee, Consultant Histopathologist, who shares our passion in teaching clinical-pathological correlation, and who inspired us to encourage students to consider how the natural history of disease progression would affect the symptoms and signs of the patient. The editors and author team would also like to thank these colleagues for their support of the first draft of the work, providing images/questions/answers and /or reviewing teaching material: Miss Antigoni Koukkoulli, Dr. David Dunleavy, Dr. Sara Hanif (Eye diseases) Miss Tosief Zahoor, Mr. M. S. Kermanshahi, Mr. Patrick Jassar, Dr. Daniel K. F. Wong, Dr. Mohammad Shebli, Mr. R. James England (ENT) Dr. Piotr Lens, Dr. Priti Sekhri, Dr. Jane Allen, Dr. Stephen Lindow Dr. Paul Jenkins, European Scanning Center, London (Vascular Diseases and Cardiothoracic Surgery chapters) Dr. Mohamed Abuzakouk (Immunology) Dr. Uday Joshi, Dr. Usha Kuchimanchi (Sexual Health) Dr. N. Todd (Microbiology) Dr. Hiten Thaker (Infectious Diseases) Dr. Jonathan Carmichael Dr. Lorcan O’Toole (Oncology) Dr. David Salvage (UK) Prof. S. Leveson (UK) Dr. Aung Phyo Oo (UK). The editors are also grateful for the help of the following people in helping source images. Hull York Medical School (HYMS) students: Claire Lilly, Baktash Khozoee, Stephanie Harrison, Catarina Santos, Craig Gouldthorpe, Alex Berry, Ata Jaffer, Anuja Joshi (UK) Mrs. Anna Bennett for echocardiography images/videos, Dr. Olga Khaleva, Research Assistant, Department of Academic Cardiology, University of Hull (UK) Dr. Ann Tweddel, Consultant Cardiologist, Hull University Teaching Hospitals NHS Trust Ms. Emma Kime, senior biomedical scientist for help with some of the pathological images Dr. Simon Taggart, consultant neurophysiologist for the nerve conduction study image Dr. Mohamed Shawgi, consultant radiologist for some of the radiographs

xi

xii

Dr. Nada Al-Muhandis, Specialty Trainee in Immunology, Hull University Teaching Hospitals NHS Trust Ms. Kristina Emsell, Senior Clinical Scientist in Immunology, Hull University Teaching Hospitals NHS Trust Recivall P. Salongcay, MD, Research Fellow, Queen’s University Belfast, UK.

Acknowledgements

Picture Credits

All the images presented in this book are either “home-grown”/obtained by our authors, or where necessary, sourced externally. Copyright permissions have been granted (where applicable). We are grateful for all our patients who have kindly consented to have their images published anonymously and for commercial purposes. In line with GMC guidance, only anonymized images will be published and where practicable, written patient consent is obtained. Pseudo-names/initials are used in case descriptions and case details may be altered to ensure anonymity. No patient identifiable material will be published thus preserving patient confidentiality, unless explicit consent was obtained. We are grateful for the following individuals and organizations for allowing us to use their illustrations. Center for Magnetic Resonance Investigations, Hull Hull University Teaching Hospitals NHS Trust Hull York Medical School (HYMS) for the use of content jointly owned with content authors. Thanks to Mr. Matthew Kempson, Programme Innovation Project Officer, for his help. Dr. Ahsan Syed, Dr. Roy (Histopathologist), and Dr. Paul Scott (Vascular Radiologist) Dr. Yusuf Patel, Consultant Rheumatologist, Hull University Teaching Hospital NHS Trust, UK Dr. Rahul Shah, Specialist Registrar in Rheumatology, Hull University Teaching Hospital NHS Trust, UK Dr. Koshy Johnson and Dr. Simon Carruthers, GPs, Hull, UK Professor Hywel Williams, Nottingham, UK Professor Jonathan Barker, London, UK Dr. David Ellis, Adelaide, Australia Professor Jayakar Thomas, Chennai, India Professor Shital Poojari, Mumbai, India Col. Dr. Manas Chatterjee, Kolkata, India.

xiii

Contents

Introduction: Cultivation of Clinical Reasoning . . . . . . . . . . . . . . . . . . . . . . . . . . . John Cookson and Kenneth Wong

1

Top Tips for Image Interpretation . . . . . . . . . . . . . . . . . . . . . . . . . . . . . . . . . . . . Martine Dujardin, Oliver Byass, and Kenneth Wong

5

Dermatology . . . . . . . . . . . . . . . . . . . . . . . . . . . . . . . . . . . . . . . . . . . . . . . . . . . . . Shernaz Walton, Katherine Walton, Vanessa Smith, Nikolaos Tyrogalas, and Javed Mohungoo

29

Endocrinology . . . . . . . . . . . . . . . . . . . . . . . . . . . . . . . . . . . . . . . . . . . . . . . . . . . . Thozhukat Sathyapalan, Ammar Wakil, David Hepburn, and Stephen L. Atkin

93

Breast Disease . . . . . . . . . . . . . . . . . . . . . . . . . . . . . . . . . . . . . . . . . . . . . . . . . . . . 105 Tasadooq Hussain and Peter Kneeshaw Pediatrics . . . . . . . . . . . . . . . . . . . . . . . . . . . . . . . . . . . . . . . . . . . . . . . . . . . . . . . . 111 Sanjay Gupta and Sanja Besarovic Cardiology . . . . . . . . . . . . . . . . . . . . . . . . . . . . . . . . . . . . . . . . . . . . . . . . . . . . . . . 127 Kenneth Wong, Abdullah Abdullah, and Renjith Antony Respiratory Medicine . . . . . . . . . . . . . . . . . . . . . . . . . . . . . . . . . . . . . . . . . . . . . . . 153 Simon Hart, Harriet Hearn, and John Cookson Anesthesia and Intensive Care . . . . . . . . . . . . . . . . . . . . . . . . . . . . . . . . . . . . . . . . 173 Marc Turnbull, Balaji Packianathaswamy, and Nabeel Siddiqui Cardiothoracic Surgery . . . . . . . . . . . . . . . . . . . . . . . . . . . . . . . . . . . . . . . . . . . . . 195 Mahmoud Loubani, Konstantinos Kotidis, and Mubarak A. Chaudhry Gastrointestinal Surgery, Gastroenterology and Hepatology . . . . . . . . . . . . . . . . . 211 Richard Young, Wingzou Wong, and Jessica Shearer Neurology and Neurosurgery . . . . . . . . . . . . . . . . . . . . . . . . . . . . . . . . . . . . . . . . . 235 Fayyaz Ahmed, Aamir Ijaz, Modar Khalil, Anna Bjornson, and Shailendra Achawal Geriatric Medicine . . . . . . . . . . . . . . . . . . . . . . . . . . . . . . . . . . . . . . . . . . . . . . . . . 259 Tun Aung and Suzanne Wong Renal and Transplant Medicine and Urology . . . . . . . . . . . . . . . . . . . . . . . . . . . . 273 Sunil Bhandari and Sigurd Kraus Trauma and Orthopedics . . . . . . . . . . . . . . . . . . . . . . . . . . . . . . . . . . . . . . . . . . . . 295 Helen Cattermole, Timothy Boddice, and Carl Fenton Rheumatology . . . . . . . . . . . . . . . . . . . . . . . . . . . . . . . . . . . . . . . . . . . . . . . . . . . . 311 Rohini Subramanya and Radhika Raghunath xv

xvi

Infectious Diseases . . . . . . . . . . . . . . . . . . . . . . . . . . . . . . . . . . . . . . . . . . . . . . . . . 329 Joanna Allen Ophthalmology . . . . . . . . . . . . . . . . . . . . . . . . . . . . . . . . . . . . . . . . . . . . . . . . . . . 343 Bryan Jason Atienza, Yahya Khedr, and Colin Vize ENT and Head and Neck Surgery . . . . . . . . . . . . . . . . . . . . . . . . . . . . . . . . . . . . . 357 Han Cao and Justin T. Murphy Obstetrics & Gynecology . . . . . . . . . . . . . . . . . . . . . . . . . . . . . . . . . . . . . . . . . . . . 375 P. Palei and U. Rajesh Immunology . . . . . . . . . . . . . . . . . . . . . . . . . . . . . . . . . . . . . . . . . . . . . . . . . . . . . . 387 Pavels Gordins Hematology . . . . . . . . . . . . . . . . . . . . . . . . . . . . . . . . . . . . . . . . . . . . . . . . . . . . . . 393 Raymond Dang and James Bailey Palliative Care . . . . . . . . . . . . . . . . . . . . . . . . . . . . . . . . . . . . . . . . . . . . . . . . . . . . 409 Raymond Dang, Kenneth Wong, and Claire M. R. Capewell Vascular Diseases . . . . . . . . . . . . . . . . . . . . . . . . . . . . . . . . . . . . . . . . . . . . . . . . . 417 Kenneth Y. K. Wong, Aung Phyo Oo, Grace W. Y. Wong, and Abdullah Abdullah Epilogue: Visualizing Medicine for Whole Person Care . . . . . . . . . . . . . . . . . . . . . . 437 Appendix . . . . . . . . . . . . . . . . . . . . . . . . . . . . . . . . . . . . . . . . . . . . . . . . . . . . . . . . . . . 439 Index . . . . . . . . . . . . . . . . . . . . . . . . . . . . . . . . . . . . . . . . . . . . . . . . . . . . . . . . . . . . . . 467

Contents

About the Editors

Dr. Kenneth Wong MA (Oxon), DM (Oxon), FRCP, FESC, FACC, PGCME (HYMS), FHEA. Consultant Cardiologist with a special interest in Imaging and Heart Failure, Lancashire Cardiac Center, Blackpool Teaching Hospitals NHS Foundation Trust; previous Senior Lecturer and Honorary Consultant Cardiologist, Hull York Medical School (HYMS), Academic Lead of the Images in Disease Programme.

Prof. Shernaz Walton DVD, MD, FRCP (Lon). Honorary Consultant Dermatologist, Hull University Teaching Hospitals NHS Trust, Consultant Dermatologist, Spire Hull and East Riding Hospital and Honorary Professor, Hull York Medical School (HYMS), University of Hull, currently serving on the editorial board of “Dermatological Therapy”, UK Dermatology Clinical Trials Network (UKDCTN) and British Association of Dermatologists Biologics and Immunomodulators Register (BADBIR) committees. She is the first recipient of the 2022 UK DCTN Honorary Lifetime Membership Award in recognition of her contribution to the efforts of the UK DCTN, having championed dermatology research to improve patient care.

xvii

xviii

About the Editors

Dr. Simi Sudhakaran MBBS, DRCOG, DFFP, MRCGP, DPD (Dermatology), FRCP (Lon). Specialty Doctor in Dermatology at Hull University Teaching Hospitals Trust (HUTHT), GP Partner at Modality Hull, Medical Director of Modality LLP Services Hull Division, Co-Author “Treatment of Skin Diseases” published in 2019, “Research News” columnist for Primary Care Dermatology Society (PCDS) quarterly bulletin, Research Fellow at UKDCTN (2019–22), Hull York Medical School (HYMS) Tutor, Mentor for HYMS DermSoc Students, HYMS Tutor of Excellence Award winner 2021. Emeritus Prof. John Cookson MD, FRCP (Lon). Previous Undergraduate Dean of Hull York Medical School (HYMS) who designed the medical school syllabus/curriculum.

Introduction: Cultivation of Clinical Reasoning John Cookson and Kenneth Wong

Introduction Our mission is to write an up-to-date book, using modern education principles [1] to enable you to apply the principles of “visualizing medicine” in a broad range of clinical scenarios across many specialties. The aim of this book differs from that of many other texts. Our experiences as practicing clinicians and clinical teachers are that the student usually faces a large gap between understanding the material presented in textbooks and using this knowledge effectively to deal with the variety of problems presented by real clinical practice. Textbooks are almost always organized by diseases, whereas patients present with symptoms, which may or may not be well defined. When a doctor meets a patient for the first time and begins to listen to the story, he or she immediately starts to interpret what is said in terms of a possible explanation or ‘image’. This explanation may be in terms of structural or functional disturbance, but often also extends to changes in family and social functioning. Thus, the “image” can and should cover the range from changes in genetic structure to changes in population structures, such as: • • • • • • •

Molecular Cell Organs Systems Patient—Presentation Family group Society

These images in the mind are then mapped as far as possible with the history and physical examination findings of the patient. In order to progress the diagnostic process, the doctor may then order a number of investigations. These are effectively the generation of some images from that patient to see if it matches other images commonly seen in the disease or diseases under consideration. In this book we have attempted to provide many of the common images that doctors and physician associates need and to link them with both the likely patient presentations and with the investigations which may be commonly used to refine a diagnosis.

Development of “Clinical Sense” or Clinical Reasoning Skills from Diagnosis to Management The purpose of taking a history, performing a physical examination and ordering tests must surely be to enable the student, physician associate or doctor to optimally manage the patient—i.e., to make a difference to the care of the patient, offering the patient hope of recovery or strategies to cope with the illness. We should try to “cure occasionally, relieve often and comfort always”. Frequently, students embarking on the exciting journey of learning to take history would read a textbook or set of lecture notes that consist of a list of questions to ask.

J. Cookson Hull York Medical School, York, England e-mail: [email protected] K. Wong (&) Lancashire Cardiac Center, Blackpool Teaching Hospitals NHS Foundation Trust, Blackpool, England e-mail: [email protected] © The Author(s), under exclusive license to Springer Nature Switzerland AG 2023 K. Wong et al. (eds.), Practical Guide to Visualizing Medicine, https://doi.org/10.1007/978-3-031-24465-0_1

1

2

J. Cookson and K. Wong

Top Tips Box—Visualize Medicine to Guide History/Examination/Investigations Elucidation of symptoms e.g. for pain – Site (radiation). – Character—e.g., throbbing, burning, dull, sharp, crushing, stabbing. – Severity. – Timing—onset, duration, change. – Precipitating and aggravating factors. – Relieving factors. – Associated symptoms. – Functional consequences: i.e. how do the symptoms affect the patient? Any physical disability, emotional, financial consequences? [e.g. Heavy Goods Vehicle (HGV) driver who suffered a myocardial infarction (MI)]. Is there support from family/friends/social services to give the patient hope and cope with the illness? Hopelessness was shown to predict all-cause mortality and MI in thousands of middle-aged men with risk factors of ischemic heart disease. – Past medical history: any risk factors of the working diagnosis? Any medical conditions which might influence the management strategies? – Drug history (don’t forget to ask about over-the-counter preparations, e.g., ibuprofen may be the cause of your patient’s bleeding from the stomach, or indeed “herbal remedies”/“Chinese medicine”, some of which may have side-effects or indeed interact with “Western medicine”). – Allergy history. – Family history (this is particularly relevant in genetic disorders). – Social history—any other risk factors e.g., smoking, alcohol, patient’s job (e.g., a shipyard worker might suffer from asbestos related lung disease), social support (see functional consequences above). – Systematic enquiry—with particular focus on points which are relevant to diagnosis and management – What are the patient’s Ideas, Concerns, and Expectations? – Physical examination and Investigations (e.g., blood test).

Systematic clinical assessment is important, but its use needs to be adapted to the patient’s story, and not used unthinkingly as a checklist for every patient. The danger of memorizing these questions and regurgitating them in every clinical encounter is potentially serious. For instance, if your patient is admitted to the Emergency Department moribund

(on death’s door) or suffers an emergency condition such as a myocardial infarction (MI), we will need to curtail the history to focus on what we really need to make a diagnosis, and initiate emergency treatment to save the patient’s life. Further, memorizing a list of questions would encourage closed questions without consideration of the pathological processes that are behind the presentation. One of the key purposes of this book is to empower the reader to visualize the processes that may lead to the presentation of the patient, and so the student, trainee or clinician can tailor the history, examination and investigations accordingly to the individual patient. So, how should we think about disease presentations? With each presentation, such as breathlessness, the doctor would consider a number of differential diagnoses (See Top tips box A framework to consider differential diagnoses (“surgical sieve”) for a framework, plus a mnemonic to help you remember the key headings to hang your knowledge upon). Top Tips Box—A Framework to Consider Differential Diagnoses (“Surgical Sieve”) Acquired – – – – – – – –

Infections Neoplasia Vascular Immunological Trauma-injury/repair Endocrine Degenerative Metabolic (e.g., Diabetic Ketoacidosis in a diabetic not taking insulin)/ nutritional (anemia in a patient who neglects herself) – Drugs/iatrogenic Other acquired causes (e.g., sleep apnea can cause hypertension but is not easily classified in the “surgical sieve”). Functional (e.g., psychosomatic disorders). Congenital – genetic – non-genetic Unknown Relationship/Social Self-neglect or self-induced Education failure-e.g., Failure of public health education regarding smoking, alcohol Mnemonic: “INVITED MD OF CoURSE”.

Introduction: Cultivation of Clinical Reasoning

An alternative way to consider differential diagnoses is to focus on the commonest causes first. “You will do well to know and be able to confidently list the 3 commonest causes of all the common presentations in your clinical exam.” This was the advice of an MRCP teacher (MRCP is a UK exam for aspiring physicians, testing whether they are ready for higher specialist training). This is a useful piece of advice and is arguably better than saying, this patient’s acute severe breathlessness several weeks after an MI can be due to infection (but the onset tends to be less abrupt than vascular causes), neoplasia (which however tends to cause gradually worsening dyspnea with other associated symptoms such as cough and hemoptysis), pulmonary embolism or myocardial infarction (vascular), inflammatory cause (e.g., acute asthma), trauma (e.g., pneumothorax), etc. This is important as all too often, students and inexperienced doctors would come up with a list of differential diagnoses which they have memorized for the purpose of exams, but which nevertheless give tutors or examiners the impression that they do not have clinical common sense. In this example, the patient has acute heart failure, which is rapidly recognized by the clinician by “pattern recognition”. Pattern recognition is the exciting process by which clinicians often use to make a diagnosis. It is like detective work, and suits those who are good at jigsaw puzzles. The patient is a 74-year-old woman who presented to the emergency department with acute sudden onset severe breathlessness that woke her up in the middle of the night. She had no chest pain or other symptoms. Past medical history includes recurrent admissions with heart failure due to mitral regurgitation as a result of mitral valve prolapse. Physical examination revealed a raised jugular venous pressure, 3rd and 4th heart sounds, a loud pansystolic murmur, and bilateral inspiratory crackles which extend from the lung bases to the mid-zones. The doctor recognized the pattern of symptoms and signs, and promptly made the diagnosis of acute heart failure, due to her underlying mitral valve disease, and initiated life-saving emergency treatment (sitting the patient up, giving her oxygen, and intravenous diuretics, morphine and nitrate). Chest Radiograph confirmed the diagnosis of acute pulmonary edema, and the patient went home after a few days, safe and sound. Whilst clinical common sense may take some years to cultivate, we have distilled here a framework to enable the student to visualize medicine when thinking about disease presentations, and hence, be better at “pattern recognition”. See Top tips box—Thinking about disease.

3

Top Tips Box—Thinking About Disease • Definition • Etiology or cause – epidemiology, incidence, prevalence, distribution • Pathogenesis – sequence of events from the initial stimulus to the ultimate expression of the disease • Morphological changes – macroscopic – microscopic • Clinical features – functional consequences – symptoms and signs • Speed of onset (acute vs. chronic) – severity – natural history (course) – treatment-effects and side-effects – prognosis – primary features versus secondary complications. Aim at making an accurate diagnosis as a package“cause, diagnosis, consequences / complications”. Remember “looking for” functional consequences of a diagnosis and anticipation of future complications are important considerations of a good physician associate, nurse practitioner or doctor “looking after” a patient. You will do well to consider each patient’s diagnosis in the context of risk factors (cause) and anticipate the development of complications. That will hopefully enable prevention of harm to the patient. Remember the example given earlier of that 74-year-old patient who kept getting readmitted into hospital with acute heart failure, thought to be caused by her mitral valve disease? She was too frail to have mitral valve surgery to remove the “cause”. We explored the possibility that she was non-compliant with her oral diuretic medication, but she denied that. However at a multi-disciplinary heart failure team meeting, the community nurse reported that she had discovered plenty of diuretic medication which had not been taken by the patient. The patient was very lonely, and really appreciated the tender, loving care which she received in hospital. In this text, we have emphasized the relation between morphological features and clinical features, including how pathogenesis relates to disease natural history as seen in presentations. This will also help the doctor rationalize

4

J. Cookson and K. Wong

management, taking into account the causes of the presentation, which very often in real clinical practice would be due to a mixture of physical illness, psychosomatic “functional” causes, and social background. For example, an elderly patient who slipped and fell but fortunately without sustaining significant injuries would normally not require hospital admission. However, lack of social support might mean that she could not remain at home, leading to a risk of a hospital-acquired infection, possibly fatal. It is helpful to organize your learning around a number of key presentations. You will find many of these core case presentations throughout the book. Throughout the book, imagine you are encountering patients as you work through the clinical scenarios. We would encourage going through each case, step-by-step, trying to answer the questions posed (before looking at the answers suggested, and before moving onto the next question/stage in the patient’s history). Each chapter will start with some “learning outcomes”, followed by the main body of the text (with questions and answers, around cases/images) to help you develop clinical reasoning skills and knowledge.

common diagnosis first, but be aware of unusual clinical data (symptoms, signs, or investigation results) which should alert you to consider alternative hypotheses and challenge your initial assumption. Regular review of your patient might alert you to lack of response to treatment and enable you to consider other differential diagnoses. Aim at making an accurate diagnosis as a package —“cause, diagnosis, consequences/complications”. Remember: “looking for” functional consequences of a diagnosis and anticipation of future complications are important considerations of a good physician associate or doctor “looking after” a patient. Last but not least, remember that all images are only a reflection of the truth. You will do well to try and integrate the evidence (from the images) to try and find out what is the true pathology or cause of your patient’s problem. Medicine is not always black and white. Even experienced doctors, nurse practitioners or physician associates can make incorrect diagnoses. When things don’t add up, look at the original evidence and find out what is true. So, keep an open mind, test your hypotheses (assumptions), look for clues that don’t fit, and always learn from past mistakes.

Conclusions

Reference

The challenge is to cultivate clinical reasoning skills or “clinical sense”, bearing in mind that “common conditions occur commonly, but rare diseases do occur”. Thus, statistically you are more likely to be correct if you consider a

1. Cookson J, Hammond A, Owen S, Wong K. Linking early clinical experience and basic science using images of disease. In Presentation at the Association for Medical Education in Europe conference, Milan; 2014.

Top Tips for Image Interpretation Martine Dujardin, Oliver Byass, and Kenneth Wong

Learning Outcomes • To recognize that images are a reflection of the true pathology. To recognize the limitations and strengths of different imaging approaches to help make accurate diagnoses efficiently. • To identify 5 key points to double check prior to appreciating any imaging in order to avoid significant error. • To list a structured approach for chest, abdominal and bony X-ray (radiograph) interpretation. • To gain experience in recognizing some of the most common life-threatening abnormalities that require emergency treatment. • To describe the difference between a benign and malignant chest nodule on a chest X-ray. • To identify the 3 different window settings used while reading CT images and discuss the main purpose for every window setting. • To describe the rationale behind acoustic shadowing and posterior enhancement in ultrasound images.

• To recognize a T1 and a T2 weighted magnetic resonance image. • To give examples of use of nuclear scan in medical imaging. Pre-study Questions Extended Matching Questions (EMQ): Topic:

Imaging modality of choice/Focus: Diagnostic imaging of the abdomen

Options: A

Erect Chest X-Ray (Radiograph)

B

Abdominal plain film

C

Abdominal Ultrasound

D

Colonoscopy

E

Abdominal Ultrasound guided biopsy

F

Abdominal Unenhanced CT

G

Abdominal Contrast enhanced CT

H

Abdominal Unenhanced MRI

I

Abdominal Contrast enhanced MR

Supplementary Information The online version contains supplementary material available at https://doi.org/10.1007/978-3-031-24465-0_2. The videos can be accessed individually by clicking the DOI link in the accompanying figure caption or by scanning this link with the SN More Media App.

J

PET-CT

M. Dujardin . O. Byass Department of Radiology, Hull University Teaching Hospitals NHS Trust, Hull, UK e-mail: [email protected]

Lead in:

For each of the patients below select the most appropriate imaging option from the list above. The same option can be used more than once

O. Byass e-mail: [email protected] K. Wong (&) Lancashire Cardiac Center, Blackpool Teaching Hospitals NHS Foundation Trust, Blackpool, England e-mail: [email protected] © The Author(s), under exclusive license to Springer Nature Switzerland AG 2023 K. Wong et al. (eds.), Practical Guide to Visualizing Medicine, https://doi.org/10.1007/978-3-031-24465-0_2

5

6

M. Dujardin et al.

Stem:

Stem:

1

A 23-year old female underwent appendectomy 5 days ago. She presents with fever and a neutrophilic blood count

C

1

A 42-year old man presents to the ED with acute abdominal pain, radiating to the left iliac fossa. Some blood is seen in his urine dip

E

2

A 63-year-old man is referred to the outpatient clinic because of long standing rectal (PR) bleeding

D

2

C

3

An 85-year old man is referred to the outpatient clinic because of a change in stool colour and abdominal pain

G

A 10-week pregnant woman presents to the ED with acute abdominal pain, radiating to the left iliac fossa. Some blood is seen in her urine dip

3

J

A 60-year old man is presenting to the Emergency Department with a history of sudden onset abdominal pain. He has been taking medication for his acute gout this week

A

A 57-year old man presents to the outpatients clinic with ongoing haematuria

4

A 60-year old man has a follow-up CT for his abdominal aortic aneurysm repair. On this CT a longstanding renal cortical cyst seems to have slightly changed in appearance

D

A 5-year old child is brought to the Emergency Department because of severe acute abdominal pain. He becomes pale during the painful episodes. There is a palpable mass in his right iliac fossa

C

5

A 20-year old woman presents to ED with Right Iliac Fossa (RIF) pain. Dipstick confirmed urinary tract infection (UTI)

C

4

5

Topic: Topic:

Imaging modality of choice/Focus: Diagnostic imaging of the urogenital system

Imaging modality of choice/Focus: Diagnostic imaging of the female pelvis

Options: Options: A

Abdominal plain film

A

Erect Chest X-ray (XR)

B

Abdominal Ultrasound

B

Abdominal plain radiograph

C

Transvaginal Ultrasound

C

Renal Ultrasound

D

Abdominal and Transvaginal Ultrasound

D

Contrast enhanced ultrasound

E

Unenhanced CT

E

Unenhanced CT

F

Contrast enhanced CT

F

Contrast enhanced CT

G

Colposcopy

G

CT Urogram

H

Unenhanced MRI

H

Unenhanced MRI

I

Contrast enhanced MRI

I

Abdominal Contrast enhanced MR

J

PET-CT

J

Renal ultrasound plus cystoscopy

Lead in:

For each of the patients below, select the most appropriate option from the list above. The same option can be used more than once

Lead in:

For each of the patients below, select the most appropriate option from the list above. The same option can be used more than once

Top Tips for Image Interpretation

7

Stem:

Stem:

1

A 65-year old woman presents to the outpatient clinic with postmenopausal vaginal bleeding

C

1

A 25-year old man presents to the outpatients clinic with acute chest pain after coughing

B

2

A 60-year old woman presents with bloating, no obvious palpable mass within the abdomen and pelvis

D

2

G

3

A 60-year old woman presents with a well defined palpable abdominal mass, ascites and significantly raised CA-125

I

A 55-year old woman presents with acute chest pain and respiratory distress. She has been flying from Japan to the UK and has just arrived at the airport

3

A 70-year old man presents with blood in his sputum for a week

B

4

A 28-year old woman presents with what appears to be a single recurrence from previous vaginal cancer at the level of the rectovaginal pouch. She is considered for total pelvic exenteration

J

4

A 71-year old man presents with an ongoing cough. He has had multiple courses of antibiotics and his lower base consolidation on plain film and clinical exam is unchanged

F

5

A 25-week pregnant woman presents with right sided acute abdominal pain and is known to have a small dermoid cyst of her right ovary. Ultrasound scan was performed but remained inconclusive

H

Topic:

Imaging modality of choice/Focus: Diagnostic Imaging of the chest

Options: A

Erect Chest Radiograph

B

Plain Chest Radiograph

C

Ultrasound

D

Radionuclide lung scan, ie ventilation-perfusion scan (V/Q scan)

E

Unenhanced CT scan

F

Contrast enhanced CT scan

G

CT Pulmonary Angiogram

H

Unenhanced MRI scan

I

Contrast enhanced MRI scan

J

PET-CT scan

Lead in:

For each of the patients below, select the most appropriate option from the list above. The same option can be used more than once

MCQs 1. What is the most optimal diagnostic imaging modality to assess a patient with suspected acute cholecystitis? a. XR b. Ultrasound (US) c. Unenhanced CT d. Contrast enhanced CT e. MRI Answer: b 2. Enlarged mediastinum on plain chest radiograph is a sign of: a. Pneumonia b. Aortic aneurysm c. Infrahilar adenopathy d. Foreign body aspiration e. Cardiac failure Answer: b 3. On an abdominal CT which window settings would exclude free air? a. Soft tissue b. Bony window c. Lung window d. Soft tissue and bony window e. Soft tissue and lung window

8

M. Dujardin et al.

Answer: e 4. A CT pulmonary angiogram: a. Is a non contrast scan b. Is the best test to be used in pregnancy to rule out Pulmonary Emboli (PE) c. Excludes aortic dissection d. Can show pulmonary infarctions e. Uses a double contrast dose Answer: d 5. Lung metastases are best detected by using which of the following imaging modality: a. US b. Contrast enhanced US c. CT scan d. Unenhanced MRI e. Contrast enhanced MRI Answer: c Techniques and key features for image interpretation Pitfalls/Mistakes to avoid – Always check patient details (name, date of birth, date of exam and patient identifier…) before even looking at an image. Then look for gender and age specific features in the image (breast soft tissue shadows, any osteoarthritis, growth plates..) and see if this image is likely to match with the patient details. Only then proceed to image interpretation. – Check image quality and check whether left/right or other markers are placed appropriately. – An image can never be interpreted appropriately without clinical details. – Always look at the entire image and not only at the organ you were interested in. – Seek help from a radiologist whenever in doubt, it takes expert experience to adequately interpret images. This is especially the case when interpreting images of poor image quality.

When choosing the optimal imaging technique, issues to take into account are: – What is the best radiological technique for this pathology? – Is there a one-stop-shop-exam possible? – Always weigh the risk/ benefit of an exam.

– Take cost effectiveness into account. – Availability and length of a technique? For example, a woman presenting with some discomfort in the right upper quadrant, where gallbladder stones are suspected and potentially a mild associated cholecystitis, several techniques would be able to demonstrate such gallbladder stones: abdominal X-ray (XR), ultrasound (US), computed tomography (CT) and magnetic resonance imaging (MRI) would all reliably enable the identification of these stones. While an abdominal XR (involving radiation) would show us the gallbladder stones, it will not inform us regarding any possible mild gallbladder inflammation. An abdominal CT will inform us of cholecystitis, but not all gallstones are visible on CT and this usually involves the use of iodine contrast (risk of contrast use should be considered especially if the patient has a history of allergy or renal impairment) and radiation dose. While an unenhanced MR would show us the gallbladder stones, the gallbladder wall, any potential inflammation and the entire biliary tract beautifully and without radiation exposure, it is more costly than the other techniques and is usually not readily available. Therefore, the use of MRI needs to be reserved for the more challenging cases. Ultrasound (US) has the advantage of avoiding radiation exposure and is very widely available. It can easily depict the echogenic gallstone. It is less costly than both CT and MR and has a high sensitivity and specificity to detect early cholecystitis (more than CT). Therefore in this scenario, US is the first line imaging test of choice. Should there be any further suspected complications like a complex abscess, CT can be performed should the US remain unclear (more easily available and cheaper compared to MRI). In the case of suspected obstructive common bile duct stones, MR is the method of choice (as it is much more robust in demonstrating this compared to CT). An MR exam can easily take up to 30 min or more to perform, while a CT takes seconds. Therefore MR is rarely the method of choice in circumstances where time is of the essence. In particular in the case of a high impact road traffic accident, one usually opts for a full body CT (head, neck, abdomen and pelvis) as a one-stop-shop exam. Rather than making individual elaborate X-rays of the spine first, followed by X-rays of the skull and neck, US of the abdomen to exclude liver and splenic laceration followed by a head CT to rule out an intracranial bleed. While in a pregnant woman radiation should be avoided if at all possible, when there is a relevant suspicion of pulmonary embolism, CT pulmonary angiogram can be chosen after multidisciplinary consideration and after patient consent in view of the mortality of pulmonary embolism in pregnancy.

Top Tips for Image Interpretation

9

– Has the patient got any contraindications to any of the techniques available? Patient safety is of course mandatory and needs to be taken into account in every patient. Therefore, it seems reasonable to always apply as small a dose of ionising radiation as achievable. Individual patient circumstances may direct or limit the choice of investigation even further. Think for instance of contrast use (e.g. gadolinium based MR contrast are to be avoided in patients with glomerular filtration rate (eGFR) < 30 ml/min to avoid nephrogenic systemic fibrosis, a potentially lethal disease thought to be directly caused by the gadolinium agent; it is common practice to avoid iodine based contrast in renal impairment to avoid nephrotoxicity and to cautiously apply iodine contrast in patients on metformin ( an oral drug used in diabetes mellitus which is usually stopped and only restarted after rechecking kidney function two days post contrast administration).

Fig. 1 Chest X-ray (CXR)-systematic reporting

X-ray (XR)/Radiograph – – – –

Uses ionising radiation First line radiology exam Low cost imaging technique High availability

Technique: An XR is a projection radiograph. Usually, a chest film is made posteroanterior (PA), where the X-ray beam (source of radiation) penetrates the patient’s chest from posterior and the patient stands closely facing the X-ray detector plate. Dense structures like bone block the X-ray beam to a large extent, resulting in a radio opaque (white) appearance in a classic X-ray display, like for instance the clavicles and ribs. As the lungs contain air and are highly penetrable, the X-ray beam arrives at the detector plate nearly intact, which results in the translucent (black) appearance in the film. Depending on their penetrability by the X-ray beam, all tissues within the field of view end up with a particular grey level on the detector plate. Typical of an XR is the superposition of information within the image. As the soft tissue of breast is superimposed to the lung base, usually this results in some reduction in translucency (more white) compared to the mid lung field. Figure 1 Chest X-ray. Figure 2 What do we need to look at in a chest XR? Try to identify. The aortic knuckle, right atrium, left ventricle, the hilum, scapula, clavicle, and first rib. We need to subsequently look at

Fig. 2 What do we need to look at in a chest XR?

– Upper mediastinum (enlargement may be a sign of underlying tumour) – Cardiomediastinal outline (any cardiomegaly? mediastinal mass?) – Lung fields (any consolidation in keeping with pneumonia? lesion? pleural effusion?) – Bony structures (fractures? metastasis?)

10

M. Dujardin et al.

– Other soft tissues (breasts, …) – Anything else? e.g. Intracardiac Device

D D

Identify the Intracardiac device? Trace the leads that connect the device to the heart and name the structures within the heart that the device is connected to? Answer: Cardiac resynchronisation therapy pacemaker device, connected to the right ventricle (RV), coronary sinus (stimulating the left ventricle to contract at the same time as the RV); the third lead is connected to the right atrium.

F

Pitfalls/Mistakes to avoid How to look at a chest radiograph? • Appreciate the clinical details (Infective? Smoker? Haemoptysis? …) • Most people will recommend a free global search through the image. • This may be followed by a structure by structure directed search (mediastinum, heart, lung-fields, bony structures, other soft tissues) • Are there any former images? • Comparison with former images when available gives crucial information as for instance a lesion stable in appearance over 2 years is likely benign. • Allow sufficient time to read an image, however longer viewing times (minutes) may increase false positive findings.

Key points for image interpretation A benign lung lesion – Usually does not change on a subsequent chest film. – Is well delineated.

Pitfalls/Mistakes to avoid Is a finding a nipple shadow or a lesion?: – Look at the opposite site, nipple shadows are usually symmetrical. – Find a previous chest XR to compare with, nipple shadows can usually be found in a previous radiograph. – If uncertainty persists, the chest XR can be repeated with nipple markers.

R

C

Fig. 3 What do we need to look at in an abdominal XR?

Figure 3 What do we need to look at in an abdominal XR?

Key points for image interpretation What to look for in an abdominal XR – Any subdiaphragmatic free air? (This is in keeping with a visceral perforation in the absence of recent abdominal surgery). – Organ outline? For instance kidneys, liver can be outlined and rarely the contours of a mass can be seen on XR. – Psoas muscle outline (Loss of psoas muscle outline on an abdominal XR suggests retroperitoneal disease, either mass or inflammation) – Any gaseous distention of either small or large bowel? These may be signs of obstruction when associated with absent air within distal bowel loops and in the presence of significant air-fluid levels. – Air fluid levels in the bowel? (More than 3 air fluid levels of more than 2 cm each suggest obstruction). – Any calcifications (renal stones? Gallbladder stones) – Bony structures (Any metastasis? Fractures?) – Other soft tissues (outline of breast shadows, skin folds…)

Top Tips for Image Interpretation

11

What can we learn from the abdominal XR in Fig. 3? – There is no air below the diaphragm (this would be a sign of visceral perforation). – No air fluid levels and no dilated bowel loops (these features would suggest obstruction). – There is still air within the distal rectum, which is reassuring. – There is a lot of faecal loading. – This is a female (breast shadows). – A small radio-opaque well delineated calcification is seen in the left hemi pelvis in keeping with a small phlebolith. Figure 4 What do we see in this abdominal film? The liver shadow (L), small bowel loops (S) are pushed upwards by a round large radio-opacity (M) obscuring the central abdomen. Through this mass the psoas lines can still be outlined which suggests this is not a retroperitoneal mass. The distal rectum presents with a fecaloma (F). Figure 5. How does this CT (coronal reconstruction of a contrast enhanced CT) in this woman with abdominal swelling and pain help visualisation of the underlying pathology? Answer: The CT revealed a large complex solid and cystic mass compressing the bladder, ovarian in origin causing this central opacity on the abdominal XR (Fig. 4).

L

Fig. 5 Does this CT (coronal reconstruction of a contrast enhanced CT) in this lady with abdominal swelling and pain help you visualize the underlying pathology? S

M

F

Fig. 4 What do we see in this abdominal film?

Because of the stained glass appearance of the lesion, this is likely mucinous in origin. The small bowel loops and stomach are contrast filled (oral contrast is given as well as IV contrast) on the CT and the stomach is seen on the XR as a typical gas bubble. Ascending colon is seen containing faeces. Figure 6 What do we need to look at in a bony XR?

Key points for image interpretation How to look at a bony plain film. We need to look at. – – – – –

Bony anatomy (any congenital anomalies?) Bony aspect in general, any osteopenic features? Focal lesions? Cortical delineation: is the cortex well aligned? Soft tissue: any foreign body? Soft tissue mass?

12

M. Dujardin et al.

– Periosteal reaction? (periosteal thickening can be a sign of either infection or malignancy.) The image in Fig. 6 teaches us the following: – This is an adult, since there are no epiphyseal growth plates visible. – The clinical information is that of a fall, complains of pain at the level of the external malleolus and reduced mobility. – The cortex of the tibia is well aligned. – Soft tissues are swollen at the level of the external malleolus. – There is an obvious cortical interruption at the level of the distal fibula in keeping with an acute distal fibula fracture. Ultrasound (US) – – – – – – –

No use of ionising radiation Patient friendly technique First line radiology exam Low cost imaging technique High availability Operator dependent Difficulty making a diagnosis based on a single picture since this is a hands-on exam

The general principle is that the echogenicity of the US image depends on how much of the US waves that have been sent out by the transducer reaches the same transducer after having been reflected by the examined structure of interest (How much of the echo is sent back to the transducer determines the echogenicity and this is structure specific). Water does not reflect the US waves and therefore an increased reflection posterior to a purely watery structure is seen: this is called posterior acoustic enhancement. At this early stage a transvaginal US can give more detail. Bone and calcifications as well as air reflect the ultrasound waves intensely and this causes an absence of reflections posterior to such a structure, a phenomenon that is called acoustic shadowing. Therefore, immediately posterior to a bony or calcified structure, evaluation using ultrasound is suboptimal. Figure 7 Transabdominal US in a 10-week pregnancy. Posterior enhancement is seen as a white shadow behind the gestational sac containing the 5 cm foetus. The placenta can be delineated anterior. On the right acoustic shadowing is seen as a dark shadow posterior to bowel gas.

S Fig. 6 What do we need to look at in a bony XR?

Top Tips for Image Interpretation

13

Fig. 7 Transabdominal US in a 10 week pregnancy

Key points for image interpretation Ultrasound – is a hands-on technique. – Posterior acoustic enhancement (white shadow) is the hallmark of a cyst or fluid filled structure and is caused because the structure posterior to the cyst reflects more acoustic waves as the cyst lets them pass freely. – acoustic shadowing is in keeping with a calcified or bony structure or air and is seen as a dark shadow as these particular structures reflect the acoustic wave and as such the tissues posterior to this receive and reflect limited or no acoustic waves.

Computed Tomography (CT) – Uses ionising radiation. – Use of iodine based contrast should be clinically relevant and the risk/benefits of the contrast use are to be appreciated on an individual basis (potential allergies

and potential nephrotoxicity especially in the renal impaired). – Cross sectional images and reconstructions are obtained. – High availability. The knowledge of the patient’s clinical signs and the clinical question is mandatory since – This is helpful for image interpretation. – This allows the appropriate scanning technique to be chosen. Various scan techniques and various contrast injection protocols are used: only the use of the appropriate scan technique and injection protocol will allow an answer to the relevant clinical question. What type of contrast phase is this? Figures 8, 9 and 10. Figure 8 No contrast in arteries or veins in the image. The renal cortex is not enhanced. Figure 9 Contrast is seen in the aorta and the renal cortex is well enhanced, while the liver lacks enhancement in this early phase. The spleen demonstrates its particular arterial phase tiger-stripe pattern.

14

M. Dujardin et al.

Figure 10 A similar contrast enhancement is seen in aorta and IVC and the liver is well enhanced. In case of a brain or limb scan, just look at the vessels to depict whether there has been contrast administered or not. Which window setting should be chosen?

C

The CT images (Figs. 11, 12, 13 and 14) illustrate the typical windows (relevant choice of upper and lower range of Houndsfield Units and level setting allow some structures to be visualized and analyzed in detail, while other structures remain obscured. Each structure has its own optimal window setting). Which structures are easily evaluated and which are obscured by the following level and window settings?

Fig. 8 No contrast in artery or veins in the CT image

A C

S

Fig. 9 Contrast is seen in the aorta (A) and the renal cortex is well enhanced (C), while the liver lacks enhancement in this early phase Fig. 11 Bony window of skull (CT)

A

Fig. 10 A similar contrast enhancement is seen in aorta (A) and IVC and the liver is well enhanced

Fig. 12 Bony window of liver (CT)

Top Tips for Image Interpretation

15

– The right horizontal fissure is well seen and within the upper aspect of the right lower lung there is a small bulla. – Soft tissue is poorly evaluated using this window setting. Figure 14 Mediastinal or soft tissue window (CT) around [Level (Center) 50 HU, Window 350 HU].

Fig. 13 Lung window (CT)

– Allows optimal evaluation of vessels, soft tissues. – Note that bony details and particularly the lung details are less optimally evaluated. – Note the hypodense saddle embolus in the pulmonary artery and the pulmonary embolus in the left main pulmonary artery. In order to allow significant conspicuity of a pulmonary embolus, a particular ‘pulmonary arterial’ contrast phase must be performed (CT pulmonary angiogram). In this phase, the pulmonary artery presents with denser contrast compared to the descending and ascending aorta. Pitfalls/Mistakes to avoid CT – Always check images in all relevant window settings (soft tissue, bone, lung window), as otherwise significant pathology may remain obscured.

MRI

Fig. 14 Mediastinal or Soft tissue window (CT)

Figures 11 and 12 Bony window (CT). around [Level (Center) 300 HU, Window 2000 HU]. – Allows optimal delineation of bony trabeculation and delineation of bony cortex as well as heavily calcified structures. – Note that the lung details and soft tissue details are not well evaluated. Figure 13 Lung window (CT) around [Level (Center) -600 HU, Window 1600 HU]. – Allows fine detail evaluation of the lung fields.

– No use of ionising radiation – Contrast is not nephrotoxic but is not recommended in patients with renal impairment and a GFR of below 30 ml/min due to potential risk of developing Nephrogenic Systemic Fibrosis (NSF), a systemic, potentially lethal condition that is associated with the use of gadolinium based contrast agents in patients with renal impairment only. – Multiplanar cross sectional images are obtained – Excellent soft tissue contrast – Lower availability compared to CT – Longer scan time compared to CT – Equipment cost is high – Patients need to be MR-safe (no ferromagnetic metal implant, e.g. no pacemaker, although recent research, development and innovation have produced MRI compatible intracardiac devices so some patients can have MRI scan after a period of time following the device implantation. You must check whether the device is MRI compatible with the device expert and inform the MRI department.

16

M. Dujardin et al.

H B

Fig. 15 MRI-How do I recognise a T2 and a T1 weighted image?

– Similar to CT the clinical question allows the relevant sequences, planes and area to be chosen (a dedicated surface coil is placed over the area of interest). Figure 15 MRI-How to recognize a T2 and a T1 weighted image? Look for a water filled structure within the image that is not a vessel. In a T1-weighted image such watery structure as CSF, gallbladder or bladder is hypointense. This is a sagittal T1-weighted unenhanced image at mid uterine level. The bladder is hypointense. Note the abnormal hyperintensity filling the endometrial cavity which is in keeping with blood (hematoma) within the endometrial cavity in a patient with a stenosis of the external os from radiotherapy. Figure 16 T2 weighted MRI image of hematoma in the endometrial cavity. Same patient as the T1-weighted image in Fig. 15. Again, look for a water filled structure within the image that is not a vessel and in the pelvis preferably the bladder (B). In a T2-weighted image such watery structure like CSF, gallbladder or bladder fluid is hyperintense. On the T2 – weighted image the typical hypointense junction zone can be distinguished from the myometrium. Again, abnormal heterogeneous fluid is seen within the endometrial cavity in keeping with a hematoma (H).

Fig. 16 T2 weighted MRI image of hematoma in the endometrial cavity

are hyperintense. Arterial and venous phase images can be made similar to CT. Figure 17 is a high resolution pelvic axial image through the uterus showing two simple ovarian fluid filled cysts (hypointense content) (C). A contrast-enhancing complex structure was seen anterior to the second ovarian cyst which presents with a thick enhancing wall (the inflamed fallopian tube), in a febrile patient with extensive left abdominal pain.

C

C

How to recognise a contrast enhanced (T1) image? In a contrast enhanced (T1-weighted) image vessels (in the brain), and vessels as well as organs (renal cortex, spleen…)

Fig. 17 A high resolution pelvic axial image through the uterus showing two simple ovarian fluid filled cysts (hypointense content) (C)

Top Tips for Image Interpretation

17

Fig. 18 Pyosalpinx

Figure 18 A diffusion-weighted image shows two areas of restricted diffusion in an inflamed fallopian tube: pyosalpinx.

Key points for image interpretation MR Look for a fluid filled structure which is not a vessel in the image, e.g. CSF. If the fluid is hyperintense the image is T2-weighted. If the fluid is hypointense the image is T1-weighted. In a T1-weighted contrast enhanced image the vessels are enhanced. Diffusion weighted imaging can help detect malignancy, cerebral infarction and pus. Mammography – Screening technique for the detection of breast cancer – US and fine needle biopsy allow further assessment of a positive finding – On occasion MR mammography provides additional preoperative information or demonstrates response to chemotherapy

Fig. 19 Dense mammogram revealing a few small calcifications

Figure 19 Dense mammogram revealing a few small calcifications. Dense mammography is usually followed by ultrasound (Fig. 20). Within the fibro glandular tissue a small lesion is seen which is not a simple cyst since it does not present with any posterior acoustic enhancement. The lesion is not very well delineated. Biopsy showed this was a malignant lesion. Figure 21 MR-mammography axial image highlighting the silicon within both breast implants. While the left implant is intact the right implant demonstrates the typical linguini sign in keeping with an intracapsular breast implant rupture. Angiography – Uses ionising radiation

18

M. Dujardin et al.

Fig. 20 Within the fibro glandular tissue a small lesion is seen which is not a simple cyst since it does not present with any posterior acoustic enhancement. The lesion is not very well delineated. Biopsy showed this was a malignant lesion Fig. 22 Renal artery stenosis

Fig. 21 MR- mammography axial image highlighting the silicon within both breast implant

– Gold standard technique for vessel evaluation but parenchymal evaluation is poor compared to other techniques as US, CT and MR – Use of iodine based contrast similar to CT – Intra-arterial (IA) or Intravenous (IV) contrast administration – Invasive technique due to catheter insertion in large vessels – Therapeutic procedures (stents, coils placement) can be added Figures 22 and 23 renal artery stenosis.

Fig. 23 Renal artery stenosis

Intra-arterial digital subtraction angiography (IA DSA) of a patient with a high grade renal artery stenosis. After the renal artery stent has been inserted, angiography demonstrates patency and restored vessel calibre. Figure 24 Renal artery stent (CT). Post contrast axial CT image in the same patient at a later stage shows the stent. Vessel detail at CT is inferior

Top Tips for Image Interpretation

19

slice the heart in a “vertical long axis” fashion. What are the findings of the scan? Diagnosis? Figure 25 Nuclear cardiology- to diagnose ischemia. Answer:

Fig. 24 Renal artery stent (CT)

compared to the IVDSA, but right renal cortical scarring is well demonstrated and two simple cortical cysts are seen. Nuclear Imaging PET in oncology Positron emission tomography (PET) is a technique where a radionuclide or tracer-usually positron-emitting fluorodeoxyglucose (FDG), a glucose analogue- is injected intravenously. A gamma ray detector will then produce a functional image showing metabolic activity. Often this technique is combined with a CT scan obtained during the same visit, in order to cross compare both metabolic activity and anatomical detail. PET scans are particularly helpful for determining how far the cancer has spread and assessment of treatment response.

Findings: Poor perfusion to the septum, apex and reduced perfusion to the inferior wall in the stress images. There was very little improvement in the perfusion to the inferior wall at rest, with no improvement in perfusion to the septum/apex at rest. Diagnosis: Myocardial infarction, with little reversible ischemia. Stress induced reversible ischemia is an imaging correlation of angina due to increased workload of the heart, typically relieved by rest within 5 min. On the other hand, the regions of the heart permanently scarred by an MI are associated with perfusion abnormalities that are FIXED-present to the same extent both at rest and stress.

Key points for image interpretation – Viability assessment enables us to tell whether a region of the heart that is not moving well is still salvageable with revascularization. – This can be done with either stress echocardiography or cardiac MRI which has the distinct advantage of demonstrating scar accurately. – Nuclear imaging can also be used to diagnose viability but it involves radiation.

Cases and key features for image interpretation A 65-year old man presents with shortness of breath, swollen feet and tiredness. What is the abnormality on the Chest X-ray (CXR)? What is the Diagnosis? (3*). Figure 26 CXR of a breathless patient.

Nuclear imaging in cardiology Answer: A 70 year old man has prolonged chest pain radiating to the neck, jaws and left arm, not relieved by nitroglycerin spray. His ECG showed ST-elevation. He did well after receiving reperfusion therapy but several months afterwards, he experienced more angina. This is a myocardial perfusion scan performed using Technetium. Technetium highlights well perfused areas of the heart. The brighter the better myocardial perfusion. Stress images and rest images are intercalated (top row stress image, the adjacent row rest image). The top 2 rows look at the short axis of the heart. The 3rd and 4th row slice the heart in a “horizontal long axis” fashion. The bottom 2 rows

The Chest XR reveals an increased cardiothoracic index or CTR of above 0.5 (cardiomegaly). The heart is enlarged and the hila are prominent on a vascular basis. Diagnosis: Heart Failure A 50- year old woman presents with retrosternal pain, predominantly in the early afternoon. A CXR reveals a retro cardiac mass (M) with an obvious air fluid level. What is the diagnosis? (3*) Figure 27 CXR of a patient with chest pain

Fig. 25 Nuclear cardiology- to diagnose ischemia

20 M. Dujardin et al.

Top Tips for Image Interpretation

21

Fig. 26 CXR of a breathless patient

Fig. 28 CXR of a patient with chest pain after a road traffic accident

A 22-year old male has a sharp pain in his left hemithorax after a road traffic accident. What is the diagnosis? (3*) Figure 28 CXR of a patient with chest pain after a road traffic accident. A chest X-ray demonstrates a vertical subpleural line at the level of the left lung base, and lateral to this lacks normal pulmonary markings. Within the soft tissues adjacent there is surgical emphysema. Monitoring material is superimposed. Answer: Fig. 27 CXR of a patient with chest pain

Diagnosis: left pneumothorax Answer: Large hiatus hernia. Pitfalls/Mistakes to avoid – The medial border of the scapula can easily be mistaken for a pneumothorax line. Always follow the suspicious line upwards and downwards to avoid this mistake. – A pneumothorax always presents with absent lung markings lateral to the pneumothorax line.

A 70-year old man, with a fever following hip surgery had this Chest X-ray. What is the main abnormality? (4*) Figure 29 post hip surgery CXR ordered because of fever Answer: Findings: Cardiomegaly Right central line is well positioned

22

M. Dujardin et al.

Fig. 29 Post hip surgery CXR ordered because of fever

Bilateral extensive pleural plaques suggest asbestosis contact in history No pneumonia Small left pleural effusion.

Fig. 30 Previous CT chest of the patient whose CXR was shown in Fig. 29

Key points for image interpretation – A central line should have its tip in superior vena cava, but not as far down as the right atrium (RA).

A previous chest CT is at our disposal. Figure 30 Previous CT chest of the patient whose CXR was shown in Fig. 29 A hiatus hernia is seen on the CT and in retrospect retrocardiac gas can be seen on the chest radiograph. The pleural plaques are obvious on the CT. Pitfalls/Mistakes to avoid – Always compare with previous imaging whenever available – We ended our search prematurely due to the most conspicuous abnormality (the pleural extensive plaques). Beware that many patients have more than one abnormality.

Fig. 31 CXR of a patient who coughs

Answer: Findings:

A 60-year old man presents with a 6 month history of cough. What is the main finding on the Chest X-ray? (4*) Figure 31 CXR of a patient who coughs.

COPD features (horizontal ribs, more translucent lungs and flattening of the diaphragm).

Top Tips for Image Interpretation

23

Fig. 32 CT scan of the patient who coughed (with an abnormal CXR in Fig. 31)

A right infrahilar mass is not obviously seen. Figure 32 CT scan of the patient who coughed (with an abnormal CXR in Fig. 31). Contrast enhanced CT chest shows a suspicious spiculated mass in the posterior upper aspect of the right lower lobe in keeping with primary lung cancer. A 35 year old man presents with urinary blockage and pain in the right flank. What can be seen? (3*) Figure 33 plain abdominal film of a 35 year old male who presents with urinary blockage and pain in the right flank.

Fig. 33 Plain abdominal film of a 35 year old male who presents with urinary blockage and pain in the right flank

Answer: A plain abdominal film reveals the outline of the right kidney and several renal calcifications. The catheter in the penis is seen on the plain film. Ultrasound demonstrated a 9 mm (longitudinal section) and a further 7 mm echogenic stone, without hydronephrosis. Ultrasound is an excellent technique to demonstrate renal stones, but the entire ureteric tract cannot be evaluated using this technique. Figure 34 Unenhanced axial image of a CT KUB (kidney urinary bladder CT) clearly demonstrates the dense right nephrolithiasis, and allows ureteric evaluation in the same scan. A 68 year old man presented with mild abdominal pain. He had diabetes and previous major vascular surgery. Besides a right dynamic hip screw, what surgery does this abdominal plain film suggest? (3*). Figure 35 Besides a right dynamic hip screw, what previous operation does this abdominal plain film suggest?

Fig. 34 Unenhanced axial image of a CT KUB (kidney urinary bladder CT) clearly demonstrates the dense right nephrolithiasis, and allows ureteric evaluation in the same scan

Answer: Aortobifemoral graft. Figure 36 A CT angiogram is performed to exclude any leakage at the level of the aortic stent. Axial slice demonstrates the patent stent without any contrast leakage towards the surrounding old aneurysm. See the beautiful coronal reconstruction (Fig. 37). Diagnosis: aortobifemoral graft, no leakage (as no contrast is seen outside the graft). A 25 year old pregnant woman presents with a 2 day history of extensive headache. Because of radiation

24

M. Dujardin et al.

Fig. 35 Besides a right dynamic hip screw, what surgery does this abdominal plain film suggest?

Fig. 37 Coronal reconstruction [CT scan of aortic stent] Fig. 36 An CT angiogram is performed to exclude any leakage at the level of the aortic stent

Top Tips for Image Interpretation

25

Fig. 38 Severe Headache in a pregnant 25 year old patient

protection issues an MR is performed and not a CT. Besides the standard T1 and T2 weighted sequences, a 3D time-of-flight MR (3D TOF MR) angiography is performed (non contrast angiography). What is the diagnosis? (5*). Figures 38 and 39.

Fig. 39 Severe Headache in a pregnant 25 year old patient

Answer: Diagnosis: superior sagittal sinus thrombosis. Acute venous infarction with high signal on DWI (diffusion weighted image) showing an area of restricted diffusion on the left, and low “ADC values” (corresponding apparent diffusion coefficient map shows an area of low signal posteriorly on the left showing that this is indeed a true area of restricted or decreased diffusion) due to the superior sagittal sinus thrombosis. A 50 year old man presented with left arm pain during exercise. ECG showed ST elevation. After primary percutaneous coronary intervention (PPCI), his left arm pain resolved. A few weeks later he complained of dyspnea on exertion. ECG showed pathological Q waves (3 mm deep) in V3-V4 and II, III and aVF. Echocardiography was non-diagnostic (poor images). Contrast enhanced CMR (cardiac magnetic resonance) was performed including cine imaging to evaluate cardiac function and delayed enhancement imaging to detect scar tissue. What is the diagnosis? (5*). Look at Video 1 first. Can you see an akinetic segment of the left ventricle? Akinesia of the inferior segment of the apex is seen on the cine images.

Video 1 Cine of akinetic inferior apical segment [See online supplementary digital material] (▶ Cine of akinetic inferior apical segment)

26

M. Dujardin et al.

Fig. 40 Cardiac MRI to evaluate left ventricular function and scar size Video 2 Cine of hypokinetic inferior segment [See online supplementary digital material] (▶ Cine of hypokinetic inferior segment)

Now look at Fig. 40 and examine the inferior wall for scar size. Scar appears white, and if it is transmural then the akinetic area is not viable. Figure 40 Cardiac MRI to evaluate left ventricular function and scar size. Answer: Diagnosis: The akinetic area was associated with transmural delayed enhancement (hyperintense area in the inferior apical segment) in keeping with transmural non-viable post ischemic scar. A 76 year-old man presents with stable angina, dyspnea and palpitations during exercise. ECG demonstrated sinus tachycardia with 2 mm ST depression inferiorly when the patient had angina. Contrast enhanced CMR (cardiac magnetic resonance) is performed including cine imaging to evaluate cardiac function and delayed enhancement imaging to detect scar tissue. (5*). Hypokinesia of the inferior segment of the apex is seen on the cine images. Figure 41 Cardiac MRI to diagnose viability. What is the diagnosis?

Fig. 41 Cardiac MRI to diagnose viability

Top Tips for Image Interpretation

27

Answer: Diagnosis: Delayed enhancement was noted in the endocardium (hyperintense area in the inferior segment at mid left ventricle level). The epicardial myocardium within this segment is still viable (dark on the delayed enhancement images). Hypokinesia of the inferior segment of the apex is seen on the cine images. This suggests viable myocardium whilst akinetic segments are only viable if there is not too much scar (e.g. full thickness scar would imply non- viable myocardium). Presence of viable myocardium in this 76 year old man is expected as he has angina. A subendocardial infarct with ongoing angina means the patient may benefit from revascularization. A 40 year old man presents with headache and drowsiness following a fall. He was known to abuse alcohol. An unenhanced CT of the head is performed. What is the diagnosis? (3*). Figure 42 CT head after head injury.

Fig. 43 CT head for severe onset severe headache after sex

Answer: Hyperdense acute frontotemporal subdural hematoma causing edema (loss of normal cortical alignment on the left compared to the right) and discrete bulging of the midline towards the right. A 42-year old woman presents with a sudden onset of severe postcoital headache. An urgent unenhanced CT is performed. What is the diagnosis? (4*). Figure 43 CT head for severe onset severe headache after sex. Answer:

Fig. 42 CT head after head injury

Hyperdense acute ‘starfish’ appearance of subarachnoid blood caused by an acute cerebral aneurysm rupture. A 65-year old woman presents with acute onset of breathlessness. On examination 3rd and 4th heart sounds are heard as well as inspiratory crackles throughout both lung fields. Figure 44 is the CXR immediately after admission to the acute medical unit when she was gasping for breath and Fig. 45 is after treatment. What is the most likely diagnosis? (3*). Figure 44. Figure 45.

28

M. Dujardin et al.

Answer: heart failure Note the typical batwing appearance of pulmonary edema and bilateral pleural effusion, successfully treated with IV diuretics and oxygen therapy.

Conclusion The most important take-home message is that images are a reflection of the true pathology. Recognize the limitations and strengths of different imaging approaches to help you make accurate diagnoses efficiently.

Recommended Further Reading Fig. 44 The CXR immediately after admission on the acute medical unit when she was gasping for breath

Fig. 45 Post successful treatment

1. Dilsizian V, Pohost GM. Cardiac CT, PET and MR. 1st ed. Oxford: Blackwell Publishing; 2006.

Dermatology Shernaz Walton, Katherine Walton, Vanessa Smith, Nikolaos Tyrogalas, and Javed Mohungoo

Learning Outcomes: Structure and Function of Skin: • Describe the different layers of the skin. • Describe the other components of the skin and mention their functions. • Describe the functions of the skin and explain its role in thermoregulation and as a protective barrier. • Describe the different immune mechanisms causing skin disease.

• Apply knowledge of the above to assess a patient, devise a differential diagnosis, commence investigation, and provide a plan for initiating care and its continued management. • Look for comorbidities, such as arthritis, cardiovascular risk, and features of the metabolic syndrome. • Include the mucous membranes, hair, and nails in your clinical examination where relevant. • Be familiar with the common hair and nail disorders and their differential diagnoses and management. Benign and Malignant Lesions:

Inflammatory Skin Diseases: • Describe the pathophysiology and common presentations of eczematous eruptions, psoriasis, and acneiform eruptions. • Describe the morphology and clinical features of the different types of eczema, psoriasis, acne, rosacea, drug eruptions and collagen vascular disease. • Have a knowledge of dermatological emergencies and their management.

S. Walton (&) . J. Mohungoo Department of Dermatology, Spire Hull and East Riding Hospital, Anlaby, England e-mail: [email protected] J. Mohungoo e-mail: [email protected] K. Walton Brook Green Medical Center, London, England e-mail: [email protected] V. Smith Department of Dermatology, Northwest Anglia Foundation Trust, Peterborough, England e-mail: [email protected] N. Tyrogalas Department of Dermatology, Calderdale, Huddersfield NHS Foundation Trust, Huddersfield, England

• Describe the clinical features, and management of the following benign lesions: Viral warts, Seborrheic keratosis, Dermatofibroma and Keratoacanthoma. • Diagnose benign lesions and consider potentially more serious differential diagnoses. • Describe the clinical features and pathophysiology of benign, premalignant, and malignant non-melanocytic and melanocytic skin lesions. • Explain the determinants of prognosis for melanoma recognizing its different presentations and stages with its life-threatening nature. • Describe the significance of the BRAF gene in the management of Malignant Melanoma. • Apply the above knowledge to assess a patient presenting with a pigmented lesion, using a holistic approach, including full skin examination. • Describe the clinical features and pathophysiology of premalignant lesions: actinic keratosis and Bowen’s disease. • Describe the clinical features and pathophysiology of basal cell carcinoma and squamous cell carcinoma. • Utilize the above knowledge to assess a patient, formulate a differential diagnosis, investigate, and plan treatment for non-melanoma and melanoma skin cancers. • Explain how Ultraviolet Light damages the skin.

© The Author(s), under exclusive license to Springer Nature Switzerland AG 2023 K. Wong et al. (eds.), Practical Guide to Visualizing Medicine, https://doi.org/10.1007/978-3-031-24465-0_3

29

30

S. Walton et al.

Pre-study Questions: EMQ Theme—Inflammatory Skin rash Focus—Diagnosis. Options: A. B. C. D. E. F. G. H. I. J.

Psoriasis Atopic Eczema Contact Dermatitis Lichen Planus Scabies Acne Vulgaris Acne Rosacea Discoid Lupus Erythematosus Venous (stasis) eczema Erythema nodosum.

Lead in: A 30-year-old hairdresser presents with a rash of 18 months duration. Stems. Presents with a silvery scaly rash on elbows and knees

A

A positive patch test confirms the diagnosis

C

Rhinophyma is a complication of which condition

G

Fig. 1 Rash on hands with sharp cut off above wrists. Full and informed patient consent has been obtained to reproduce the image in commercial publication

MCQ 1. A 45-year-old male printer gave a 6-month history of hand dermatitis (Fig. 1). A few months previously he had started to use the solvent trichloroethylene in his job. Patch testing was negative. On substituting a different solvent, the eruption cleared. What is the diagnosis? A. B. C. D. E.

Allergic contact eczema Irritant contact eczema Psoriasis Atopic eczema Infected eczema.

Answer: B 2. A 15-year-old schoolboy gave a three-month history of an intensely itchy papular eruption affecting the hands,

Fig. 2 Burrows in finger webs. Reprinted by permission from Springer Nature: Diagnosis and Treatment of Scabies by Dr. Edgardo Chouela et al., © 2012

wrists, and penis. Several lesions were excoriated. Treatment with a potent topical steroid was of little benefit. His mother and sister had also recently developed itchy lesions. Close examination showed burrows in the skin (Fig. 2).

Dermatology

31

Fig. 3 Blisters in trigeminal area. Reprinted by permission from Springer Nature: Trigeminal Neuralgia Associated with Herpes Zoster by M. A. Piradov et al., © 2019

What is the diagnosis? A. B. C. D. E.

Eczema Scabies Allergic contact eczema Lichen planus Pompholyx.

Answer: B. 3. A 30-year-old man develops a painful blistering eruption in the trigeminal nerve distribution area of his face. It is extremely painful. He gives a history of recent contact with his 5-year-old son who had chicken pox (Fig. 3). What is your diagnosis? A. Erysipelas B. Sunburn

Fig. 4 Scaly plaques on knees. Full and informed patient consent has been obtained to reproduce the image in commercial publication

C. Herpes Zoster D. Chicken Pox E. Drug reaction. Answer: C. 4. A 30-year-old man is referred to the dermatology department by his GP with these scaly plaques on both knees and elbows (Fig. 4). There is a family history of a similar rash in his father and brother. He is otherwise well in himself.

32

S. Walton et al.

What is the diagnosis?

Answer:

A. B. C. D. E.

(A) Basement membrane (B) Artery (C) Subcutaneous fat (D) Arrector pili muscle (E) Vein (F) Eccrine sweat gland

Eczema Psoriasis Lichenification sec. to kneeling Lichen planus Discoid Lupus Erythematosus.

(G) Hair follicle (H) Sebaceous gland (I) Hair shaft (J) Epidermis (K) Dermis (L) Subcutis

Answer: B. Figure 6 shows the histology of the epidermis, including its four layers.

Basic Structure and Function of Skin with Immuno-Pathogenetic Mechanisms Causing Skin Disease

Name the principal cell of the epidermis. (1*) Answer:

(a) Structure and Function of Skin (b) Immune Mechanisms Causing Skin Disease (c) Pathogenetic Mechanisms in Psoriasis (a) Structure and Function of Skin (1*) The diagram (Fig. 5) shows the structure of the skin. Name the components labeled A–L. (1*) Fig. 5 Components of human skin. Adapted and sourced from Anatomy, Skin, Sudoriferous Gland (authors: Patel BC et al.). Creative Commons Attribution 4.0 International License (http:// creativecommons.org/licenses/by/ 4.0/), which permits use, duplication, adaptation, distribution and reproduction. Permission granted by Cynna Plantz from StatPearls Publishing on 31 May 2021

The keratinocyte, which produces a protein, keratin. Describe the structure of the stratum basale including its composite cells. (1*)

Dermatology

33

Fig. 6 Normal skin histology

Answer: Keratinocytes comprise most of the stratum basale, which are attached to the basement membrane by hemidesmosomes. The stratum basale also contains melanocytes, responsible for the synthesis of melanin. The melanin produced is transferred to adjacent keratinocytes via dendritic cells. Merkel cells are also found in the stratum basale and are connected to peripheral nerves. Although their exact function is unknown, they are thought to play a role in sensation.

Figure 7 shows the histology of the epidermis and dermis. Can you name the structures A–D? (1*) Answer: (A) (B) (C) (D)

Rete ridges Connective tissue Papillary dermis Reticular dermis

Explain the process of keratinocyte maturation. (1*)

Which component of the skin do rete ridges belong to? (1*)

Answer:

Answer:

Keratinocytes are formed in the stratum basale, where they divide, producing daughter cells. These cells then migrate upwards, where they become interconnected by desmosomes, forming the stratum spinosum. In the stratum granulosum, keratinocytes become flattened, and their nuclei begin to degenerate. Once they reach the stratum corneum they become corneocytes, flattened cells without nuclei, which are filled with keratin.

The epidermis. Name the cell population present in the epidermis. (1*) Answer: • Keratinocytes (they form the stratum corneum following differentiation). • Melanocytes (produce melanin in response to UV radiation which is taken up by maturing keratinocytes).

34

S. Walton et al.

Fig. 7 Histology of skin

• Langerhans cells (participate in the immune response). • Merkel cells and tactile discs (specialized receptors sensitive to touch).

The subcutis.

Name the cell population in the dermis. (1*)

What does it consist of? (1*)

Answer:

Answer:

• Mesenchymal cells (stem cells, can differentiate to produce all connective tissue cell types). • Fibroblasts (produce collagen fibers and ground substance, participate in tissue repair and healing). • Melanocytes. • Mast cells (release histamine and heparin). • Adipocytes. • Macrophages, neutrophils, lymphocytes, and eosinophils.

• Adipose tissue (fat). • Blood vessels. • Nerves.

Can you name the other functions of the skin? (1*)

Describe the connective tissue of the dermis. (1*)

Answer:

Answer:

• Acts as a barrier, providing protection from pathogens, physical agents, and mechanical and thermal injury. • Prevents loss of moisture. • Acts as a sensory organ. • Reduces the penetration of UV radiation. • Role in Vitamin D synthesis. • Forms part of the immune system.

70% of the dermis is composed of collagen fibers, which provide strength. Elasticity of the skin is provided by elastin fibers, which are loosely arranged in the dermis. Fibroblasts within the dermis synthesize the collagen and elastin. The dermis also contains a ground substance made up of glycosaminoglycans. What layer lies deep to the dermis? (1*)

Answer:

Figure 8 shows how the skin plays an important role in thermoregulation.

Figure 9 are melanocytes found within the stratum basale.

Dermatology

35

Fig. 8 Thermoregulation. © 2022 Illustration drawn by K Walton, author of section 1

Can you describe the principal function of Langerhans cells? (1*) Answer: Langerhans cells play an important role in the skin’s immunological system, where they act as antigen-presenting cells. Which other cells make up the skin’s immunological system? (1*) Answer: Fig. 9 Melanocytes in skin. Permission granted by Hull York Medical School (HYMS)

T-lymphocytes, mast cells, macrophages, and keratinocytes. What are cytokines? (1*)

What is the function of a melanocyte? (1*)

Answer:

Answer:

Cytokines are soluble molecules that act as messengers between cells, mediating the inflammatory response.

Melanocytes produce melanin, responsible for skin pigmentation. They absorb UV light reducing the amount of UV penetration into the skin, helping to protect against skin cancer. What is a Langerhans cell? (1*)

Which cells within the skin produce cytokines? (1*) Answer: T-lymphocytes, Langerhans cells, keratinocytes, fibroblasts, endothelial cells, mast cells and macrophages.

Answer: (b) Immune Mechanisms Causing Skin Disease (3*) Langerhans cells are dendritic cells and are characterized by a specific organelle within their cytoplasm, known as the Birbeck granule.

The four types of hypersensitivity reaction are illustrated in Fig. 10.

36

S. Walton et al.

Fig. 10 Hypersensitivity reactions © 2021 Illustration drawn by K Walton, author of section 1

See if you can use this diagram to describe the different components of the four reactions. (3*) Answer: Type I (immediate): IgE is attached to the surface of mast cells. Contact with the antigen promotes degranulation and the release of inflammatory mediators, including histamine. Urticaria or anaphylaxis results within minutes. Type II (antibody dependent cytotoxicity): Antibodies (IgG/IgM) produced in response to the antigen cause subsequent cytotoxicity either via killer T-cells or complement fixation and activation. Type III (immune complex disease): Antigen and antibody form immune complexes which circulate in the blood and are deposited in the walls of small vessels, joints, and glomeruli. Vascular damage then results from subsequent complement fixation, platelet aggregation and release of inflammatory mediators.

Type IV (cell-mediated or delayed): First exposure to the antigen activates T-helper cells which increase in number. Secondary contact with the antigen causes cytokine release which attracts more macrophages and T-cells to the site. Skin lesions appear because of tissue damage 42–78 h after exposure to the antigen. Now see if you can match the skin rashes in Figs. 11, 12, 13 and 14 with the hypersensitivity reactions. (3*) For each figure, state the diagnosis and the type of hypersensitivity reaction responsible. Answer: Figure 11: Leukocytoclastic vasculitis, Type III hypersensitivity reaction. Figure 12: Urticaria, Type I hypersensitivity reaction. Figure 13: Bullous pemphigoid, Type II hypersensitivity reaction. Figure 14: Allergic contact eczema (shoe leather), Type IV hypersensitivity reaction.

Dermatology

Fig. 11 Purpuric rash. Full and informed patient consent has been obtained to reproduce the image in commercial publication

Fig. 12 Wheals on skin. Full and informed patient consent has been obtained to reproduce the image in a commercial publication

37

Fig. 13 Blisters on skin. Full and informed patient consent has been obtained to reproduce the image in a commercial publication

Fig. 14 Eczematous rash on foot. Full and informed patient consent has been obtained to reproduce the image in a commercial publication

38

S. Walton et al.

Fig. 15 Genes and cytokines in Psoriasis. Permission granted by Professor Jonathan Barker, St. John’s Institute of Dermatology (King’s College)

(c) Pathogenetic Mechanisms in Psoriasis (5*) Can you describe the role of identified genes and autoimmunity as seen in the diagram (Fig. 15)? (5*) Answer: There are about 9 genes identified in Psoriasis with the main gene, known as Psors1 located on chromosome 6 in the Human Leukocyte Antigen Complex (HLA-C) region which is responsible for adaptive immunity. Key to Genes and Cytokines in diagram [1]: ERAP-1 ! is located on chromosome 5q15 and regulates peptides which are attached to major histocompatibility complex (MHC) molecules. PSORS1 (HLA-C) ! on chromosome 6p2. The gene also attaches to Class I MHC molecules, and is the main gene associated with psoriasis. TRAF3IP2 ! Chromosome 6q21. This protein is essential for IL-17 and Th17-mediated inflammation which occurs in psoriasis.

NFKBIA ! Chromosome 14q13. It controls cytokine production/release and cell survival in the absence of inflammation. REL ! Chromosome 2p16. It is important in regulating cellular responses in the absence of inflammation. TYK2 ! Chromosome 19p13. It regulates interferon signaling and may contribute to initiation and/ or maintenance of progression of Psoriasis IL28RA (relates to interferon), ZAP70 (relates to T cell signaling), RNF114 (located on Chromosome 20q13) ! all play a role in regulating immune responses. Genes encoding the interleukin-23 and interleukin-12B indicate an increased risk of developing psoriasis. Cytokines play a role in regulating inflammatory responses. Drugs are now developed to target these cytokines and their receptors to treat inflammatory conditions such as Psoriasis. All the new biological therapies target TNFa and cytokine receptors and are a major development in the treatment of Psoriasis.

Dermatology

39

Inflammatory Skin Disease

What is your diagnosis? (2*)

(a) (b) (c) (d) (e) (f) (g) (h) (i) (j) (k)

Psoriasis.

Psoriasis Eczema Lichen Planus Acne Vulgaris Acne Rosacea Discoid Lupus Erythematosus Systemic Lupus Erythematosus Dermatomyositis Systemic Sclerosis Erythema multiforme Erythema nodosum.

(a) Psoriasis

Which other areas of the body would you examine? (2*) Answer: The entire skin, scalp, and nails. This patient has 40% of his body surface affected. What treatment would you suggest? (3*) Answer:

Bilateral scaly erythematous plaques with a silvery scale symmetrically distributed on extensor surfaces (elbows).

Topical treatment with a combined Vit. D analogue and topical steroid (Dovobet ointment or Enstilar foam) once a day for 5 weeks and commence a course of Phototherapy. Vit D analogues act by reducing the turnover of keratinocytes within the epidermis. Another patient is referred complaining of pain and swelling in the distal interphalangeal joints (arthropathy) of both hands associated with early morning stiffness lasting 1.5 h and which started 2 years before seeing her GP.

What is an Auspitz Sign? (2*)

Describe what you see in Fig. 17. (3*)

Answer:

Answer:

After removing layer after layer of scale, the underlying skin shows bleeding points.

Pitting of nails. Psoriatic plaques on the fingers. Swelling of the terminal interphalangeal joints.

A 37-year-old patient presents with this rash in Fig. 16. How would you describe it? (2*) Answer:

Fig. 16 Scaly plaques on elbow. Permission granted by Hull York Medical School (HYMS) Fig. 17 Skin rash, nail dystrophy and swollen joints. Full and informed patient consent has been obtained to reproduce the image in commercial publication

40

What are the other nail changes associated with Psoriasis? (3*) Answer: Onycholysis (nail plate lifts off from its nail bed); nail plate then loses its pink color and looks white. This then predisposes to secondary infection. Salmon patches (“oil drop” sign). Beau’s lines (horizontal lines in nails due to inflammation in the proximal nail matrix causing the nail to stop growing) —can also occur after a systemic illness as seen in Fig. 18. Leukonychia (areas of white marks in nails). Crumbly nails due to involvement of nail bed. What treatment would you consider for this patient? (3*) Answer: After excluding modest to high intake of alcohol, after preliminary blood tests (full blood count, biochemical profile, lipids, Pro-collagen peptide-P3NP) and Chest X-ray (to

S. Walton et al.

exclude pulmonary fibrosis) commence Methotrexate therapy in secondary care. If the patient has deranged liver function, with a negative hepatitis screen and no history of alcohol, what would be your diagnosis of the liver condition? (4*) Answer: Non-alcoholic fatty liver. What would the therapeutic options be? (4*) Answer: Continue Methotrexate under close monitoring of liver function or consider Ciclosporin if patient’s kidney function and blood pressure are normal using shared care guidelines with the general practitioner (GP) for no longer than 6 months. The same patient presents 6 months later in the medical assessment unit with this rash and is unwell as seen in Fig. 19. What is your diagnosis? (4*) Answer: Generalized Pustular Psoriasis. What would you do? (4*)

Fig. 18 Beau’s line on fingernail. Full and informed patient consent has been obtained to reproduce the image in a commercial publication

Fig. 19 Pustular rash on skin. Permission granted by Hull York Medical School (HYMS)

Dermatology

Answer: Admit the patient and refer urgently to a dermatologist as this is dermatological emergency.

41

• Alcohol—Heavy intake can worsen Psoriasis. • Smoking—There is a link between smoking and palmo-plantar psoriasis, especially in women. Immunological Factors:

If a patient does not respond to phototherapy, methotrexate or cyclosporin what would you consider next? (5*) Answer: Biologic therapy with anti-TNF a agent (refer section 1 for mode of action), once screening tests including tests for latent Tuberculosis are confirmed as negative.

Psoriasis is mediated by T-cells. T helper cells (Th1) cells predominate and are activated by interleukins, (IL12, 23), Tumor Necrosis Factor a (TNFa) and interferon. All these mediators are now targets of various biological therapies in the treatment of Psoriasis What are the comorbidities associated with Psoriasis? (5*)

What causes Psoriasis? (5*)

Answer:

Genetic factors:

The Metabolic Syndrome:

There are nine different susceptibility genes to psoriasis (PSORS1-9). PSORS1, located on a region of the major histocompatibility complex on chromosome 6p2, is the main gene identified in psoriasis, and accounts for up to 50% of genetic susceptibility to the disease [2]. Other autoimmune and inflammatory diseases such as inflammatory bowel disease, type 1 diabetes, multiple sclerosis, and atopic dermatitis share these genes pointing towards similar mechanisms that are involved in a variety of inflammatory diseases (refer section 1).

This term includes obesity, arterial hypertension, abnormal glucose tolerance, and abnormal blood lipids. This is the most important comorbidity of psoriasis. Patients with psoriasis are at an increased risk of developing type 2 diabetes. Cardiovascular disease: Patients with severe disease seem to have at least a twofold or threefold increase in mortality from cardiovascular disease and double the risk of a myocardial infarction.

Environmental Triggers: Depression and Anxiety: Psoriasis can be triggered by several environmental factors as follows: • Trauma—Psoriasis can occur in scratch marks and areas of skin that have been traumatized. This is also known as ‘Koebner Phenomenon’. This phenomenon is not unique to psoriasis and can occur in lichen Planus and warts. • HIV—Psoriasis can occur for the first time in patients who have undergone seroconversion and can also make existing psoriasis to worsen. • Infection—Streptococcal infection is implicated in guttate psoriasis and sometimes precedes the skin condition. • Drugs—Beta Blockers, lithium and antimalarial drugs can exacerbate psoriasis and are best avoided in patients predisposed to psoriasis. Psoriasis can also appear for the first time if these drugs are given to these individuals. • Obesity is linked with Psoriasis and weight loss can improve the condition and the response to drugs used in treatment. • Stress—there is a strong association between stress and Psoriasis with stress is known to exacerbate the condition.

This is common in any chronic skin condition such as severe acne, eczema and psoriasis. This stems from social isolation and stigmatization. This can lead to suicidal ideation and also make treatment less effective in controlling the disease. Coping strategies help to allow patients to take control of their condition. (b) Eczema Atopic Eczema A 3-year-old child presents with this itchy rash in Fig. 20 and is fretful due to lack of sleep. How would you describe the skin rash? (2*) Answer: Face: vesicular, papular rash with erythema and edema. Hand: Excoriations and crusting due to dried up exudates.

42

S. Walton et al.

Fig. 20 Rash on face and hands. Permission granted by Hull York Medical School (HYMS) and parental consent obtained for commercial publication

Fig. 21 Skin histology showing spongiosis. Permission granted by Hull York Medical School (HYMS)

What other history would you elicit? (3*) Answer: Ask if other siblings or 1st Degree relatives have a history of eczema, asthma, or hay fever. Do they have any pets, especially dogs and cats? Does the child keep other family members awake due to nocturnal itching and lack of sleep? Is the rash related to foods or worse during upper respiratory infections and teething? What tests would you do? (3*) Answer:

Answer:

Serum Immunoglobulin E (IgE) and radioallergosorbent test (RAST) to house dust mite, cat, dog, grass, and food battery, if indicated. Skin swab for bacterial culture if any signs of oozing or yellowish crusting. Another patient with atopic eczema encounters a person who has an active cold sore. What is the complication seen in Fig. 22? (4*)

Atopic Eczema.

Answer:

What are the histopathological features in Fig. 21 responsible for the clinical picture? (2*)

Kaposi’s varicelliform eruption (eczema herpeticum) due to disseminated Herpes simplex virus infection.

Answer:

How would you treat this complication? (4*)

Spongiosis (separation of epidermal cells due to edema between cells) # Vesicles and on rupture leave dried up exudate (crust)

Answer:

What is your diagnosis? (2*)

Patient will need to be admitted as it is a dermatological emergency and treated with topical and systemic acyclovir.

Dermatology

43

Fig. 22 Generalized vesicular rash. Full and informed patient consent has been obtained to reproduce the image in a commercial publication

His topical steroids will need to be stopped temporarily (refer section “Dermatological Emergencies”). If the patient with atopic eczema is not controlled with topical steroids once the above complication is treated, what would be your approach? (5*) Answer: Consider second line treatment in secondary care with either phototherapy or immunosuppression with Azathioprine or Ciclosporin for a short period, no longer than 6 months.

Fig. 23 Rash on hairline, around eyes and swelling of face. Permission granted by Hull York Medical School (HYMS)

Answer:

Allergic Contact Eczema

Patch test.

A 56-year-old lady in Fig. 23 develops swelling on her face within 24 h following the application of a cosmetic on her scalp. What is the active ingredient in the cosmetic? (2*)

What is the sign of a positive patch test? (4*)

Answer:

Vesiculation and erythema at the site where the PPD has been applied.

Answer:

Paraphenylenediamine (PPD) in her hair dye. What is your diagnosis? (2*)

What is the mechanism causing allergic contact eczema? (4*)

Answer:

Answer:

Allergic contact eczema.

It is type IV hypersensitivity with the memory of previous exposure to the same antigen retained in memory T cells. With each subsequent exposure to the same antigen, the interval to the onset of symptoms is shortened.

What test in Fig. 24 has been done to confirm your diagnosis? (3*)

44

S. Walton et al.

Fig. 25 Violaceous flat-topped papules on wrist. Permission granted by Hull York Medical School (HYMS)

Which other area of the body would you like to examine? (2*) Answer: The rest of the skin, especially the back and the mouth, genitalia, and nails. What do you see on the buccal mucosa in (Fig. 26)? (3*) Answer: White lacy streaks known as Wickham’s striae.

Fig. 24 Positive patch test reaction to hair dye. Permission granted by Hull York Medical School (HYMS)

How would you describe the changes on his glans penis as seen in Fig. 27? (3*)

How would you treat this condition? (4*) Answer: Withdraw the offending antigen or cosmetic and treat with topical or systemic steroids depending on severity. (c) Lichen Planus A 23-year-old man presents with an intensely itchy eruption as seen in Fig. 25. How would you describe the skin rash? (2*) Answer: Flat topped shiny violaceous papules, some coalescing into plaques.

Fig. 26 White streaks on buccal mucosa. Permission granted by Hull York Medical School (HYMS). Full and informed patient consent has been obtained to reproduce the image in commercial publication

Dermatology Fig. 27 Plaque on glans penis. Permission granted by Hull York Medical School (HYMS)

45

What clue may suggest that the eruption is drug-induced? (5*) Answer: Absence of mucous membrane involvement. (d) Acne Vulgaris A 16-year-old schoolboy presents with a 2-year history of a rash on his face as seen in Fig. 29. Can you describe the skin changes? (2*) Answer: Erythematous papules, nodules, cysts, comedones and scars. What is your diagnosis? (2*)

Answer: Answer: Erythematous, raised plaques. Acne Vulgaris. Describe the typical nail changes seen in Fig. 28. (4*) Answer: Pterygium formation on the nail. List some of the drugs that cause lichen planus—like drug eruptions. (5*) Answer: Gold, Penicillamine, Streptomycin, Tetracycline, Chloroquine, Mepacrine, Quinine, Thiazides, Captopril, Phenothiazines.

Fig. 28 Nail fold type scarring. Permission granted by Hull York Medical School (HYMS)

Fig. 29 Nodules and cysts on face. Permission granted by Hull York Medical School (HYMS). Full and informed patient consent has been obtained to reproduce the image in commercial publication

46

S. Walton et al.

Which skin appendage is implicated in the causation of acne? (2*) Answer: Sebaceous gland. What does the sebaceous gland do? (2*) Answer: Produces sebum (grease) which causes seborrhea (greasiness) and then the Cutibacterium acnes (formerly called Propionibacterium acnes) flourishes in this environment causing the breakdown of free fatty acids in the sebum which in turn causes inflammation. How do the clinical findings correlate with the pathogenesis as seen in Fig. 30? (2*) Answer: Depicted in Fig. 30. How would you develop a treatment plan for a patient with acne? (3*) Answer: You would first grade the acne severity and treatment would depend on the acne severity as seen in Fig. 31. Physiological and mild acne Moderate acne

Severe acne

Topical therapy Topical therapy + Oral antibiotics or hormonal regimen Topical therapy + Hormonal regimen or Isotretinoin

How does Isotretinoin work? (4*)

Fig. 30 Clinico—pathologic correlation in acne. Reproduced with permission from Walton S et al. Topical treatments for acne-recent additions. Dermatology in practice 2002;10(1):6–8 © Hayward Medical Communications. Copyright granted by Hayley Mayes, Project Manager-Publications on 31 October 2011

It reduces inflammation through the above effects. Which drugs cause acne? (4*) Answer: Corticosteroids, anabolic steroids, androgens, antiepileptic drugs. If acne is resistant to treatment and patient also has hirsutism, what test would you do and why? (5*)

Answer: Answer: It reduces sebum production. It reduces comedone production by reducing pilosebaceous ductal hyperkeratosis. It reduces colonization of the duct with Cutibacterium (Propionibacterium) acnes.

Free Androgen index (FAI) followed by ovarian ultrasound to exclude Polycystic Ovary Syndrome (PCOS). If PCOS is confirmed, what would you use and why? (5*)

Dermatology

47

Physiological

Mild

Moderate

Severe

Fig. 31 Severity of acne. Reproduced with permission from Walton S et al. Topical treatments for acne-recent additions. Dermatology in practice 2002;10(1):6–8 © Hayward Medical Communications. Project Manager-Publications on 31 October 2011

Answer: Metformin as these patients also have insulin resistance. Dianette® (35 mg of ethinyl estradiol and 2 mg of cyproterone acetate) as it contains cyproterone acetate which is an anti-androgen or Yasmin® (drospirenone ethinyl estradiol) which is anti-androgenic or Spironolactone for its anti-androgenic effect with adequate contraception in females (Metformin can be combined with any of the anti-androgens listed above) (e) Acne Rosacea A 60-year-old lady presents with a red face and itchy, sore eyes as seen in Fig. 32 Describe the clinical features. (2*) Answer: Skin:

Eyes:

Redness, flushing, telangiectasia (prominent vessels), papules and pustules affecting cheeks, nose, forehead, and chin. Bilateral blepharitis and conjunctivitis.

Fig. 32 Redness on face and sore eyes. Full and informed patient consent has been obtained to reproduce the image in a commercial publication

How does this condition differ from Acne Vulgaris? (3*)

The patient in Fig. 33 has other complications of Rosacea. What do you see? (4*)

Answer:

Answer:

It affects the older age group and lacks comedones (blackheads).

Redness and thickening of the nose, known as Rhinophyma. Redness and inflammation involving his right eye.

48

S. Walton et al.

redness and rhinophyma. However, isotretinoin may exacerbate the eye complications of rosacea by causing dryness and patients may benefit from the use of artificial tears. Topical therapy has its place in the treatment of rosacea. Metronidazole applied topically reduces inflammatory lesions. Ivermectin 1% cream may also be applied once daily. Topical corticosteroids should not be used as they may cause skin atrophy, vasodilatation, and worsen telangiectasia. The treatment of skin changes such as rhinophyma (thickened and bulbous nose) may benefit from surgical intervention. (f) Collagen Vascular Diseases Discoid Lupus Erythematosus A 68-year-old man in Fig. 34 presents with a scaly rash predominantly on the face and gives a history of exacerbation on sun exposure. Can you describe the rash? (2*) Answer: Fig. 33 Red rash on face with thickened red nose. Permission granted by Hull York Medical School (HYMS)

What are the ocular manifestations seen in this condition? (4*) Chronic staphylococcal blepharitis, chalazion, conjunctivitis, episcleritis and iritis. What treatment strategies would you adopt? (4*) Control rather than cure of the disease should be the aim of treatment. This needs to be explained to the patient to improve compliance with treatment. Triggers of the disease e.g. exposure to sudden temperature variations, excessive sun exposure, ingestion of caffeine containing drinks, and excessive alcohol intake should be avoided. Doxycycline and Lymecycline are effective and compliance is improved due to once daily dosage. Patients who are allergic to tetracyclines should be prescribed erythromycin or its derivatives. Low dose oral Isotretinon has been effective for the inflammatory component of the disease such as pustules,

Scaly, erythematous plaques with scarring due to skin atrophy on sun exposed areas. Can you describe the pathological process? (2*) Answer: The scaling is due to hyperkeratosis and if you lift the scale off, the under surface of the scale demonstrates ‘carpet tag’ sign which are spicules of keratotic follicular plugs. The inflammation results in erythema and the vacuolation of the basal layer causes damage to anchoring collagen fibrils and leads to scarring. Systemic Lupus Erythematosus (SLE) A 30-year-old lady in Fig. 35 presents in the medical assessment unit, acutely ill with a facial rash, systemic symptoms and blood and protein in her urine. Which feature in her skin presentation helps you with a diagnosis? (3*)

Dermatology

Fig. 34 Scaly rash with atrophic scarring on face. Permission granted by Hull York Medical School (HYMS). Full and informed patient consent has been obtained to reproduce the image in commercial publication

49

Fig. 35 Erythematous rash in photosensitive distribution. Permission granted by Hull York Medical School (HYMS). Full and informed patient consent has been obtained to reproduce the image in commercial publication

Answer: Erythema, edema and telangiectasia on face and V of neck suggesting a photosensitive distribution. What is the diagnosis? (3*) Answer: Systemic Lupus Erythematosus.

• Anti-RO and anti-LA—positive. 30% of patients with SLE and cutaneous involvement show positivity to both anti- RO and anti-LA antibodies. Anti-RO are prevalent among many autoimmune diseases, such as systemic lupus erythematosus (SLE), SS/SLE overlap syndrome, subacute cutaneous LE (SCLE), whilst anti-LA is present in patients with Sjögren’s syndrome (SS) [3]. • Anti-phospholipid antibodies—positive if associated with antiphospholipid syndrome.

What tests would help you with the diagnosis? (4*) What is the renal complication in this patient? (4*) Answer: Answer: Routine Blood Tests: • Full Blood count may show low lymphocyte numbers. • Complement levels may show low levels of C3 and C4 complements. Serum Antibody tests: • Antinuclear antibodies—positive. • Antibodies to double stranded DNA—positive.

Lupus nephritis is a possible complication. It occurs when antibodies are deposited in the renal glomeruli, causing renal disease. What are the findings in this patient which suggest glomerular disease? (4*)

50

S. Walton et al.

Proteinuria, hematuria, reduced glomerular filtration, hypoproteinemia, and edema. What would you do next? (4*) Answer: Refer the patient to a Renal Physician for a full renal assessment and a renal biopsy. Dermatomyositis A 55-year-old man is referred by his GP with a facial and hand rash and complains of difficulty in climbing stairs and combing his hair. What are the skin features in Figs. 36 and 37 that help you with a diagnosis? (2*)

Fig. 37 Scaly papules on hands. Permission granted by Hull York Medical School (HYMS). Full and informed patient consent has been obtained to reproduce the image in commercial publication

Answer: Figure 36: Heliotrope erythema and edema around the eye and on the neck. Figure 37: Scaly, erythematous papules on proximal hands (Gottron’s papules). What tests would you do to determine the cause of his muscle weakness? (3*) Muscle Enzymes-Creatine Phosphokinase (CPK): A raised CPK indicates muscle damage suggesting the presence of myopathy. CPK is normal or slightly raised in patients with cutaneous involvement alone. Electromyogram (EMG): The EMG involves the insertion of a small needle into a clinically weak muscle. This is usually performed in the department of neurophysiology in a secondary care setting. MRI scan of affected muscles shows signs of inflammation. Muscle biopsy: The specimen is best taken from a weak muscle identified on an EMG first. The MRI scan has now replaced this interventional procedure as a lot of hospitals do not have a neurophysiology department. Fig. 36 Swelling and redness around eyes and neck. Permission granted by Hull York Medical School (HYMS). Full and informed patient consent has been obtained to reproduce the image in commercial publication

The same patient develops colon cancer a year later. Is there an increased risk of internal malignancy? (5*)

Dermatology

51

Answer: Though the incidence of internal cancers is high in these patients there are no studies to date suggesting a causal link rather than that associated with advancing age. Systemic Sclerosis A 50-year-old lady presents with difficulty in swallowing and frostbite on her fingers. What are the features in this patient which suggest a possible diagnosis as seen in Figs. 38, 39 and 40? (3*) Answer: Pursed mouth. Ischemic fingers with bone resorption.

Fig. 39 Tumid fingers with tight skin. Permission granted by Hull York Medical School (HYMS). Full and informed patient consent has been obtained to reproduce the image in commercial publication

What is the diagnosis? (3*)

Fig. 40 Micro infarcts, ulceration, and loss of digits. Permission granted by Hull York Medical School (HYMS). Full and informed patient consent has been obtained to reproduce the image in commercial publication

Answer: Systemic sclerosis. (g) Erythema Multiforme

Fig. 38 Pursed mouth. Permission granted by Hull York Medical School (HYMS). Full and informed patient consent has been obtained to reproduce the image in commercial publication

A 23 year old lady who is subject to frequent cold sores develops an eruption on her skin and presents to the acute medical assessment unit. This is the 3rd time she has developed this rash. She develops new lesions every few weeks and which usually follow an attack of cold sore infection. Can you describe the rash as seen in Fig. 41? (2*)

52

S. Walton et al.

Fig. 41 Target lesions on hands. Permission granted by Dr. Koshy Johnson, retired GP, Hull. Full and informed patient consent has been obtained to reproduce the image in commercial publication

Answer:

When the condition is severe and affects mucous membranes (oral, conjunctival and genital), what is your diagnosis? (4*)

Target lesions with a central vesicle on the hands. Answer: What is your diagnosis? (2*) Answer:

Steven–Johnson Syndrome (covered in section “Inflammatory Skin Disease” under dermatological emergencies).

Erythema multiforme.

(h) Erythema Nodosum

Why did this patient develop the rash after her cold sore? (2*)

A 16-year-old girl develops these painful nodules on her legs in crops of 2–3, days after a sore throat as seen in Fig. 42.

Answer:

What is your diagnosis? (2*)

Circulating immune complexes which result from several causes (in this case from the Herpes simplex antigen–antibody complex) are deposited in blood vessels. Cell mediated immunity may be involved. This leads to necrotic epidermis with edema, an inflammatory infiltrate and vasodilatation in the dermis.

Answer: Erythema Nodosum. Can you describe the clinical findings? (2*) Answer:

What are the other causes of Erythema multiforme? (3*) Erythematous, edematous, tender, subcutaneous nodules. Answer: What is the process responsible for the clinical picture? (2*) Idiopathic (50%). Viral (Herpes Simplex, Hepatitis B, Orf, Mycoplasma, viral vaccines) Bacterial (Streptococcus). Fungal (Coccidioidomycosis). Drugs (Penicillin, barbiturate, Sulphonamide). Other (Lupus erythematosus, pregnancy, malignancy).

Answer: Microscopic appearance is a panniculitis (inflammation of the subcutaneous fat) that primarily affects the fibrous septae between the fat lobules. The fibrous septae are expanded by granulomatous inflammation, neutrophils, eosinophils, and fibroblast proliferation.

Dermatology

53

Answer: Active therapy is rarely needed as spontaneous remission is usual, although a non-steroidal anti-inflammatory (NSAID) drug may help.

Dermatological Emergencies (a) Erythroderma (b) Drug reactions, including Steven Johnson Syndrome (SJS) and Toxic Epidermal Necrolysis (TEN) (c) Severe urticaria and angioedema (d) Henoch-Schönlein Purpura (e) Pemphigus (f) Bullous Pemphigoid (g) Eczema herpeticum (Disseminated Herpes Simplex Virus Infection). (a) Erythroderma A 66-year-old man presents as unwell and feverish. This rash is spread over his entire body surface as seen in Fig. 43. How would you describe his overall condition? (2*) Fig. 42 Painful nodules on shins. Permission granted by Hull York Medical School (HYMS)

What mechanism sets off this process? (2*) Answer: It results from circulating immune complex deposition in vessels of the subcutis.

Answer: Erythroderma. Erythroderma is defined as erythema (redness) with a variable amount of scale that affects at least 90% of his body surface area.

Can you name some of the causes of Erythema nodosum? (3*) Answer: Idiopathic (20%). Bacterial (Streptococci, Tuberculosis(TB), Leprosy, Yersinia, Mycoplasma, Rickettsia). Fungal (Coccidioidomycosis). Viral (Cat scratch fever). Drugs (Sulphonamides, oral contraceptives). Systemic diseases (Inflammatory bowel disease sarcoidosis, Behcet’s Disease, malignancy-rare). How would you treat this condition? (3*)

Fig. 43 Red, scaly skin. Permission granted by Dr. Koshy Johnson, retired GP, Hull. Full and informed patient consent has been obtained to reproduce the image in commercial publication

54

Fig. 44 Measles-like rash on body. Permission granted by Dr. Koshy Johnson, retired GP, Hull. Full and informed patient consent has been obtained to reproduce the image in commercial publication

(b) Drug Reactions Including Steven Johnson Syndrome (SJS) and Toxic Epidermal Necrolysis (TEN) A 66-year-old man presents with a rash. He had started amoxicillin for a sore throat 1 week prior to presentation. Clinical appearances were as illustrated in Fig. 44.

S. Walton et al.

Fig. 45 Skin peeling off in sheets with mucous membrane involvement. Permission granted by Hull York Medical School (HYMS). Full and informed patient’s next of kin consent has been obtained to reproduce the image in commercial publication

(Cross reference: Section “Dermatological Emergencies”—exanthems). However, your patient’s condition rapidly deteriorates, and he develops the skin and mucous membrane changes as seen in Fig. 45. What is the diagnosis now? (4*)

What is the likely cause for the rash? (3*)

Answer:

Answer:

Stevens Johnson Syndrome/Toxic Epidermal Necrolysis. Stevens Johnson Syndrome (SJS) and Toxic Epidermal Necrolysis (TEN) are variants of the same condition. TEN is the more serious variant of the two. This is rare and causes skin loss like extensive burns and mucosal loss which can be fatal despite adequate treatment. It appears to be of increased incidence amongst those with HIV. The condition is typically caused by medication and a wide range of drugs have been implicated. Common

When amoxicillin is given in the presence of Epstein Barr Virus (EBV) it causes a rash. Typically, it is an itchy morbilliform (measles like) rash. It may occur when any beta-lactam antibiotic such as ampicillin, amoxicillin and cephalosporin is given in the presence of EBV. It is not a true allergy as there will be no future reaction in the absence of EBV infection.

Dermatology

55

causes include antibacterial sulfonamides, antifungals, antivirals, allopurinol, NSAIDs and anticonvulsants. Onset is between 1 and 30 days. Often there is a prodromal illness with flu-like symptoms. Once the rash occurs the condition deteriorates rapidly over hours before reaching its maximal effect at around day 4.

0–1: 32% 2: 12.1% 3: 35.3% 4: 58.3% 5 or more: >90%.

What is Nikolsky’s sign? (4*)

What is the difference between TEN and SJS? (4*)

Stop causative agent. Admit patient to intensive care or burn unit. Maintain fluid balance and fluid replacement. Temperature maintenance. Analgesia. Skin care: Greasy emollient, antiseptics, non-adherent dressings. Eye care: Ophthalmology review, eye drops. Oral hygiene. Respiratory support may be necessary. Urinary catheter. Monitor for infection and treat appropriately if required (not recommended empirically). Thromboprophylaxis. The role of intravenous immunoglobulin (IVIG), immunosuppression and systemic corticosteroids is not well established.

Answer:

(c) Urticaria and Angioedema

Skin detachment involving 30% is termed TEN. There are cases of overlap in between.

A 23-year-old shop assistant presents with acute swelling around her eyes that has developed over just a few minutes.

Answer: Applying pressure on unblistered skin resulting in separating the epidermis from the dermis. What is the significance of this and in which other condition is it typically seen? (4*) Answer: It occurs when the pathology is above the dermo-epidermal junction and is typically seen with pemphigus vulgaris and toxic epidermal necrolysis.

How would you manage the patient? (5*)

What features can you see in the photograph (Fig. 46)? (2*) Are you aware of any specific TEN/SJS scoring system that may be used as a predictor of mortality? (5*) Answer: Scorten. At the time of presentation, one point is scored for each of the following indicators [4]: • • • • • • •

Age >40 years Presence of malignancy Heart rate >120 Initial percentage of epidermal detachment >10% Serum urea level >10 mmol/L Serum glucose level >14 mmol/L Serum bicarbonate level 350), and HIV immunocompromised patients (CD4 count 4 mm

50%

Block dissection of local lymph nodes is done for tumors > than 1 mm in thickness if malignant cells are present in a sentinel node biopsy which also helps to stage the disease. Interferon a and chemotherapy are used in patients if there is secondary spread after CT staging. There have been recent trials providing evidence that new agents (e.g., the monoclonal antibody ipilimumab against anti-cytotoxic T-lymphocyte antigen 4 (CLA4) and small molecule kinase inhibitors, especially in BRAF mutant positive patients are beneficial. What is the BRAF gene? (5*) Answer: BRAF is a human gene. Mutations in the BRAF gene can appear later in life and cause malignancy, as an oncogene. Fig. 102 Ulceration. Permission granted by Hull York Medical School (HYMS)

What is the significance of the BRAF gene? (5*)

84

S. Walton et al.

Answer:

Answer:

It is present in 50% of patients with a malignant melanoma.

A firm, pea sized nodule in the skin.

It has led to the development of specific inhibitors of mutated B-Raf protein for anti-cancer use e.g., Vemurafenib —(zelboraf) that interrupts the B-Raf/ mitogen-activated extracellular signal-regulated kinase (MEK) step on the B-Raf/MEK/ERK pathway if B-Raf carries the V600E mutation (around 45–50% of skin melanoma). This agent has demonstrated significant improvement in response rate, time to progression and survival and has recently received license for the treatment of patients with melanoma with BRaf V600 mutation. So far for metastatic disease, immunotherapy and BRAF + MEK inhibitors, allow a 5-year survival of no more than 60% [6].

Is this benign or malignant? (2*) Answer: Benign. What treatment would you suggest? (3*) Answer: Excise if causing symptoms. (f) Keratoacanthoma (KA)

(e) Dermatofibroma

This lesion represents a KA (Fig. 104).

This presentation is common in General Practice (Fig. 103). Sometimes these patients are referred to a dermatologist for confirmation of diagnosis.

What is the characteristic clinical feature? (3*)

What is the diagnosis? (2*)

The hard crater is surrounded by a soft edge.

Answer:

Is this benign or malignant? (3*)

Dermatofibroma.

Answer:

How would you describe the clinical manifestation? (2*)

Benign but needs to be excised as it can resemble a squamous cell carcinoma both clinically and histologically. Some regress spontaneously.

Answer:

(g) Bowen’s In-Situ)

Disease

(Intraepidermal

Fig. 104 Nodule with central hard core on scalp. Full and informed patient consent has been obtained to reproduce the image in a commercial publication

Fig. 103 Firm pink nodule on skin. Full and informed patient consent has been obtained to reproduce the image in a commercial publication

Carcinoma

Dermatology

85 Fig. 106 Irregular hard keratotic nodule on sole of foot. Full and informed patient consent has been obtained to reproduce the image in a commercial publication

The biopsy report suggests that this is Bowen’s disease with no invasive component. How would you treat it? (4*) Answer: Fig. 105 Scaly plaque on lower leg. Full and informed patient consent has been obtained to reproduce the image in a commercial publication

This condition is common on the legs of elderly women (Fig. 105).

If the lesion is solitary and small: Excise completely. If there are multiple lesions: Cryotherapy or topical 5-Fluorouracil cream or 5% Imiquimod cream. (h) Squamous Cell Carcinoma (SCC)

How would you describe it? (3*) Answer:

A 78-year-old man develops a large nodule on the sole of his foot which has grown rapidly in the last 4 months (Fig. 106).

A scaly, erythematous plaque on the lower leg. How would you describe the lesion? (3*) What change has occurred with it? (3*) Answer: Answer: A nodule has developed within the plaque.

A large irregular keratotic nodule with ill-defined edges showing surface ulceration and telangiectasia.

What would you do next and why? (4*)

What is the diagnosis? Why has it ulcerated? (3*)

Answer:

Answer:

You would do a biopsy of the nodule and send it for histological examination to exclude a change to invasive squamous cell carcinoma.

Squamous cell carcinoma. Due to rapid growth, the tumor has outstripped its blood supply leading to ischemic necrosis.

86

S. Walton et al.

Fig. 107 Several keratotic nodules on scalp. Full and informed patient consent has been obtained to reproduce the image in a commercial publication

Can you identify the underlying predisposing condition? (4*) Answer: You can see small keratoses on the sides of the sole. This patient was given arsenic in a tonic as a young child. The keratoses represent ‘arsenical keratoses’. Can you identify the predisposing cause in the next patient who is a gardener (Fig. 107)? (4*) Answer: Actinic keratosis due to cumulative sun exposure. The large lesion may need to be examined further by referral to a specialist on a 2-week cancer pathway. How would you treat this condition? (4*)

Fig. 108 Crateriform ulcer on face. Full and informed patient consent has been obtained to reproduce the image in a commercial publication

Would this metastasize? (3*) No: It can however invade locally and is a very slow growing tumor. Which are further examples of a BCC as seen in Figs. 109, 110, 111 and 112? (4*) Answer: Yes or No. Figure 109. Yes: Translucent cystic BCC. Figure 110. Yes: Pigmented BCC. Figure 111. Yes: Superficial BCC. Figure 112. No: Fibro epithelial polyp (skin tag). How would you treat a BCC? (4*)

Answer: If seen in a GP practice: refer on a 2-week pathway to a dermatologist or plastic surgeon. Treatment: Excision with a 3–5 mm margin of the large lesion. The smaller lesions can be treated with cryotherapy. (i) Basal Cell Carcinoma (BCC) How would you describe a typical BCC as seen in Fig. 108? (3*) Answer: An ulcerated lesion with a translucent telangiectatic edge, likened to a ‘string of pearls’.

Fig. 109 Yes: Translucent cystic BCC. Full and informed patient consent has been obtained to reproduce the image in a commercial publication

Dermatology

Fig. 110 Yes: Pigmented BCC. Full and informed patient consent has been obtained to reproduce the image in a commercial publication

87

Fig. 113 Patient receiving treatment for a Basal Cell Carcinoma (BCC). Permission granted by Hull York Medical School (HYMS). Full and informed patient consent has been obtained to reproduce the image in a commercial publication

Answer: Treatment of choice: Excision with a 3 mm margin. Superficial BCC: Cryotherapy, topical 5% Imiquimod cream or Photodynamic Therapy. In elderly frail patients: Radiotherapy after biopsy confirmation. What is the treatment as shown in Fig. 113)? (5*) Fig. 111 Yes: Superficial BCC. Full and informed patient consent has been obtained to reproduce the image in a commercial publication

Fig. 112 No: Fibro epithelial polyp (skin tag). Full and informed patient consent has been obtained to reproduce the image in a commercial publication

Answer: Photodynamic therapy: a photosensitizer, 5-aminolevulinic acid or its methyl ester is applied as a cream on the BCC and after 4 h, it is irradiated with visible red light or laser light. What is the mechanism of action? (5*) Answer: The reaction between the aminolevulinic acid (photosensitizer), the light and oxygen in the tumor cells forms Reactive Oxygen Species (ROS) which acts on the cell membrane of the malignant cell and causes targeted cell death.

88

S. Walton et al.

Which BCCs is this treatment not suitable for? (5*)

What is the diagnosis? (5*)

Answer:

Answer:

BCCs greater than 2 mm in depth and morphoeic BCCs.

Frontal fibrosing alopecia (FFA).

Hair and Nails (a) (b) (c) (d) (e) (f)

Frontal fibrosing alopecia (FFA) Alopecia areata Acne keloidalis/folliculitis nuchae Koilonychia Periungual pyogenic granuloma Pseudomonas superinfection.

(a) A 58-year-old lady was referred by her GP complaining of hair loss (Fig. 114). She has noticed that her hairline at the front and sides has receded over the past couple of years. She has not experienced any symptoms. She does not tie her hair in a ponytail. On examination you note that she has hair loss in a fronto-temporal pattern, the skin where the hairline has receded is shiny and there are no hair follicles present. At the hairline there is scattered perifollicular inflammation. A punch biopsy of the scalp at the hairline shows a dense perifollicular lymphocytic infiltrate and in other areas follicular and sebaceous gland destruction and fibrosis.

Fig. 114 Receding frontal hairline in a female. Adapted and sourced from Frontal Fibrosing Alopecia (authors: N Litaiem, S Idoudi). Creative Commons Attribution 4.0 International License (http:// creativecommons.org/licenses/by/4.0/), which permits use, duplication, adaptation, distribution and reproduction. Permission granted by Cynna Plantz from StatPearls Publishing on 30 May 2021

The characteristic features of progressive hair loss in a fronto-temporal distribution in a postmenopausal woman with signs of scarring alopecia point towards the diagnosis of frontal fibrosing alopecia. It can often involve the eyebrows and it can be pruritic. It is a rare, but emerging, type of lymphocyte-mediated scarring alopecia which is thought by some to be a variant of lichen planopilaris (another type of scarring alopecia) with similar histological appearances. Diagnosis is made by clinical and biopsy findings. Management of FFA varies and commonly used agents include oral Hydroxychloroquine or Corticosteroids (topical, intralesional and oral). Topical Minoxidil can be used alone or in combination. (b) You review a 29-year-old Asian patient with hair loss in your GP surgery. He has noticed patches of hair loss on his scalp and beard. He tells you that this has happened once before in different areas of his scalp and that it resolved spontaneously after a few months, but the new hair was initially gray. He is on long term thyroxine replacement. There is no family history of skin or hair disease, but his sister has type 1 diabetes mellitus. On examination, there is patchy hair loss, and the hair follicles are intact, there is no visible inflammation and there are “exclamation mark” hairs at the border of the hairline surrounding the patches (Figs. 115, 116 and 117).

Fig. 115 Patch of alopecia on scalp. Permission granted by Col. Dr. Manas Chatterjee, Command Hospital, Kolkata. Full and informed patient consent has been obtained to reproduce the image in a commercial publication

Dermatology

89

Fig. 118 Scarring nodules on occipital scalp. Reprinted from D@anderm. Copyright Permission granted by Niels Kren Veien ([email protected]) on 22 June 2022 Fig. 116 Exclamation Mark Hair. Permission granted by Col. Dr. Manas Chatterjee, Command Hospital, Kolkata. Full and informed patient consent has been obtained to reproduce the image in a commercial publication

Fig. 117 Patch of hair loss in beard area. Permission granted by Professor Shital Poojary, K J Somaiya Medical College, Mumbai. Full and informed patient consent has been obtained to reproduce the image in a commercial publication

What is the diagnosis? (5*) Answer: Alopecia areata. The exact cause is not known but targeting of follicular and melanocyte-associated antigens has been implicated in its pathogenesis. An autoimmune cause is also suggested by reported associations with other autoimmune conditions such as autoimmune thyroid disease, vitiligo, type 1 diabetes mellitus and inflammatory bowel disease. There is also a strong association with atopy. The condition tends to recur, and it is asymptomatic. It can affect any hair-bearing area, but the scalp is favored.

The patchy form is the commonest, but more extensive forms such as alopecia totalis (total loss of scalp hair) and alopecia universalis (total body hair loss) also occur. It can follow a variable course and it can remit spontaneously or become chronic or intermittent chronic. The absence of scarring and destruction of follicles, the presence of ‘exclamation mark’ hairs at the edges, the regrowth of gray or white hair and an increased number of terminal catagen and telogen hairs are all characteristics of alopecia areata. History and clinical examination are usually sufficient to diagnose alopecia areata. The condition can regress spontaneously. Intralesional or topical steroids are first line treatment for patchy alopecia areata. Systemic pulsed corticosteroids can be used for rapidly progressive disease and topical immunotherapy with diphenylcyclopropenone can also be effective. (c) This 40-year-old patient is referred to your clinic complaining of hair loss. He gives a history of developing scarring and hair loss in an occipital distribution when he was 20 following shaving his head with a razor. Apart from occasional pruritus there have been no other symptoms. On examination of his occipital scalp, there is severe papulo-nodular scarring around the hair follicles (Fig. 118). How can you explain these findings? (5*) Answer: This patient suffers from Acne keloidalis or Folliculitis keloidalis.

90

S. Walton et al.

Fig. 119 Spoon shaped nails. Reprinted from D@anderm. Copyright Permission granted by Niels Kren Veien ([email protected]) on 22 June 2022

Acne keloidalis usually involves the occipital area and starts with inflammatory papules that involve hair follicles, which can become purulent and later scar and coalesce to form lesions resembling keloids. It sometimes extends to the vertex of the scalp. Treatment is usually with a long-acting tetracycline (e.g., Doxycycline or Minocycline). Hair laser removal has also been used in certain cases. (d) This 24-year-old lady who is known to be a vegetarian developed some nail changes in which her nails were becoming more ‘spoon-like’ as shown in Fig. 119. She is otherwise well but does also complain of heavy menses. Examination of her nails revealed spoon shaped nails. What is the cause of her nail changes? (4*) Answer: Koilonychia is a term used to describe spoon shaped nails. The most common cause is usually iron deficiency anemia and, in this case, due to her low intake of iron and heavy menses. Familial cases have also been described and rarer causes include occupational, or medication related. (e) The next patient presents to your clinic with a lesion on her finger nail (Fig. 120). This has developed over a few months. She tells you that she had several nail infections in that nail following manicure at a beauty parlor where she also used sunbeds. She has psoriasis for which she takes acitretin. There is a family history of melanoma. What is the most likely diagnosis? How should you investigate this to establish the diagnosis? (5*)

Fig. 120 Bleeding nodule on nail. Reprinted from D@anderm. Copyright Permission granted by Niels Kren Veien (veien@dadlnet. dk) on 15 June 2022

Answer: The most likely diagnosis is periungual pyogenic granuloma secondary to repetitive trauma and infection. Systemic retinoid and antiretroviral therapy can predispose to periungual pyogenic granulomas. Paronychia can sometimes resemble a pyogenic granuloma. Amelanotic subungual melanoma can ulcerate and present with a similar appearance and given the family history of melanoma and the use of sunbeds it would be essential to exclude it with a biopsy. (f) A 25-year-old patient comes to see you in your surgery complaining of the nail problem seen in Fig. 121. This started a few months ago when part of the nail bed took on a yellowish color, which later changed to green. Other fingernails show “oil drop” changes. He has been suffering from a backache. Past medical history

Fig. 121 Lifting of nail plate with green discoloration. Reprinted from D@anderm. Copyright Permission granted by Niels Kren Veien ([email protected]) on 15 June 2022

Dermatology

includes resistant toenail infections and long-term treatment with a tetracycline for acne. How are his nail signs and symptoms explained? (5*)

91 5. Lichterfeld M, Qurishi N, Hoffmann C, et al. Treatment of HIV-1-associated Kaposi’s sarcoma with pegylated liposomal doxorubicin and HAART simultaneously induces effective tumor remission and CD4+ T cell recovery. Infection. 2005;33(3):140–7. 6. Ottaviano M, Giunta EF, Tortora M, et al. BRAF gene and melanoma: back to the future. Int J Mol Sci. 2021;22(7):3474.

Answer: The diagnosis is onycholysis secondary to psoriasis with pseudomonas superinfection. Psoriatic arthropathy is common in psoriasis patients with nail changes, which could explain his backache. The clue about tetracycline is a red flag as some tetracyclines are associated with nail discoloration.

References 1. Genetic Analysis of Psoriasis Consortium & The Wellcome Trust Case Control Consortium. A genome-wide association study identifies new psoriasis susceptibility loci and an interaction between HLA-C and ERAP1. Nat Genet. 2010;42:985–90. https:// doi.org/10.1038/ng.694. 2. Smith CH, Barker JN. Psoriasis and its management. BMJ. 2006;333(7564):380–4. 3. Franceschini F, Cavazzana I. Anti-Ro/SSA and La/SSB antibodies. Autoimmunity. 2005;38(1):55–63. 4. Bastuji-Garin S, Fouchard N, Bertocchi M, et al. SCORTEN: a severity-of-illness score for toxic epidermal necrolysis. J Invest Dermatol. 2000;115(2):149–53.

Further Reading 1. Zaidi Z, Walton S, Hussain I, Wahid Z. A manual of dermatology. 2nd ed. New Delhi, London, Philadelphia, Panama: The Health Sciences Publisher/Jaypee Brothers Publishers; 2015. 2. Chiang NYZ, Verbov J. Dermatology: a handbook for medical students and junior doctors. 3rd ed. London: British Association of Dermatologists; 2020. 3. Verbov J. Dermatological disorders in illustrated textbook of pediatrics. 5th ed. Amsterdam, Netherlands: Elsevier; 2018. 4. Ottaviano M, Giunta EF, Tortora M, et al. BRAF gene and melanoma: back to the future. Int J Mol Sci. 2021;22(7):3474. 5. Thiboutot DM, Dreno B, Abanmi A, Alexis AF, Araviiskaia E, Barona Cabal MI, et al. Practical management of acne for clinicians: an international consensus from the global alliance to improve outcomes in acne. J Am Acad Dermatol. 2018;78(2 Suppl 1):S1–23. 6. Zaidi Z, Hussain K, Sudhakaran S. Treatment of skin diseases—a practical guide. [online] Rd.Springer.com. 2019. 7. Chowdhury MM, Griffiths TW, Finlay AY. Dermatology training-the essentials, 1st ed. Wiley-Blackwell, USA, UK;2022.

Endocrinology Thozhukat Sathyapalan, Ammar Wakil, David Hepburn, and Stephen L. Atkin

Learning Outcomes . Describe the diagnostic process for diabetes with reference to the WHO diagnostic criteria. . Recognise the features of diabetic nephropathy. . Recognise the fundoscopic findings of diabetes retinopathy. . Recognise the clinical features of diabetic foot ulceration. . Recognise the clinical features of hyperlipidemia. . Recognise the clinical features of pituitary tumours. . Describe how to investigate pituitary disorders. . Recognise the clinical features of autoimmune endocrine disorders. . Recognise the clinical features hyperthyroidism and hypothyroidism. . Describe the investigations for hyperthyroidism. . Recognise the clinical features of hyperparathyroidism. . Recognise the clinical features and investigations for diagnosing Addison’s Disease. . Recognise the clinical features of primary hyperaldosteronism. . Recognise the clinical features of polycystic ovary syndrome.

. Recognise the clinical features of acromegaly and outline the possible treatments. . Recognise the medical complications of obesity. Pre-study Questions EMQ Options: Molecule 1. 2. 3. 4. 5. 6. 7. 8. 9.

Thyroglobulin Monoiodotyrosine Triiodothyronine (T3) Thyroxine (T4) Thyroid Stimulating Hormone TSH receptor antibody Thyrotropin Releasing Hormone Thyroid peroxidase Calcitonin.

Lead in For each of the following scenarios choose the most appropriate investigation from the list above.

T. Sathyapalan (&) . D. Hepburn Academic Diabetes, Endocrinology and Metabolism, Hull York Medical School, University of Hull, Hull, England e-mail: [email protected] D. Hepburn e-mail: [email protected] A. Wakil Endocrine Department, Central Coast Local Health District, Gosford, NSW, Australia S. L. Atkin Royal College of Surgeons of Ireland Bahrain, Busaiteen, Kingdom of Bahrain e-mail: [email protected]

A. A 52-year-old man who had thyroidectomy four years ago for differentiated thyroid cancer. He is now attending thyroid cancer clinic for monitoring recurrence. (Thyroglobulin) B. A 62-year-old woman presented with symptoms of hypothyroidism. Which molecule stimulates iodine uptake by thyroid gland. (Thyroid Stimulating Hormone). C. A 44-year-old man presented with typical symptoms and signs of hyperthyroidism. Which is the predominant hormone secreted by the thyroid gland. (Thyroxine (T4)).

© The Author(s), under exclusive license to Springer Nature Switzerland AG 2023 K. Wong et al. (eds.), Practical Guide to Visualizing Medicine, https://doi.org/10.1007/978-3-031-24465-0_4

93

94

D. A 72-year-old woman presented with symptoms and signs of hypothyroidism with hypothyroid biochemistry. Presence of antibodies against this compound indicates autoimmune thyroid disease. (Thyroid peroxidase). E. A 34-year-old woman presented with symptoms of hyperthyroidism. Which is the predominant thyroid hormone that acts on nuclear receptors of target tissues to exert the main biological effects. (Triiodothyronine-T3). F. A 64-year-old man who had thyroidectomy two years ago for medullary thyroid cancer. He is now attending thyroid cancer clinic for monitoring recurrence. (Calcitonin). G. A 32-year-old woman presented with symptoms and signs of hyperthyroidism with hyperthyroid biochemistry. You are planning to perform a test for the biomarker of Grave’s disease. (TSH receptor antibody). H. Biomarker used to monitor recurrence of differentiated thyroid cancer after thyroidectomy. (Thyroglobulin). I. Stimulates iodine uptake by thyroid gland. (Thyroid Stimulating Hormone). J. Predominant hormone secreted by the thyroid gland. (Thyroxine-T4). K. Presence of antibodies against this compound indicates autoimmune thyroid disease. (Thyroid peroxidase). L. Predominant thyroid hormone that acts on nuclear receptors of target tissues to exert the main biological effects. (Triiodothyronine -T3). M. Raised in medullary carcinoma thyroid. (Calcitonin). N. Biomarker of Grave’s disease. (TSH receptor antibody).

Fig. 1 Hyperpigmentation on left hand with normal hand for comparison. Permission granted by Hull York Medical School (HYMS)

T. Sathyapalan et al.

MCQ 1. Which of the following is a symptom of thyrotoxicosis? A. Cold intolerance B. Weight loss C. Constipation D. Dry skin E. Depression Answer: B 2. Thyrotoxicosis typically presents with weight loss, heat intolerance, tremor, palpitations, diarrhoea, irritability, and insomnia whereas some of the symptoms of hypothyroidism are weight gain, cold intolerance, constipation, dry and flaky skin and depression. Which of the following biochemical abnormality occurs commonly in Addison’s crisis? A. Hypokalemia B. Hyperchloremia C. Metabolic alkalosis D. Hyponatremia E. Hypocalcaemia Answer: D The common biochemical abnormalities in Addison’s crisis are hyponatremia, hypochloremia, low serum bicarbonate, metabolic acidosis, hyperkalemia, hypercalcaemia and raised serum urea. These changes result predominantly from insufficient mineralocorticoid (aldosterone). Cases 1. The hand on the left in the picture (Fig. 1) belongs to a 28-year-old Caucasian man, who has dizziness on standing, low blood pressure, weight loss and tiredness. The hand on the right is for comparison only.

Endocrinology

What test can determine the underlying cause for the changes in the left hand? (2*) What is the cause of the changes ? (2*) Answer: ACTH stimulation test Increased secretion of proopiomelanocortin (a precursor of both ACTH and melanocyte stimulating hormone). Hyperpigmentation in Addison’s disease is caused by an increased production of a-melanocyte-stimulating-hormone (aMSH). Both aMSH and ACTH originate from the pro-hormone peptide pro-opiomelanocortin (POMC). In primary adrenal insufficiency POMC production is strongly increased in response to the fall in cortisol levels with concomitant release of aMSH which stimulates the melanocytes resulting in hyperpigmentation. This hyperpigmentation usually occurs in palmar creases, skin folds and gingival mucosa. ACTH stimulation tests are used to diagnose Addison’s disease. Synthetic ACTH is administered, and response of cortisol is determined. In Addison’s disease there is suboptimal cortisol response to ACTH since adrenal gland is hypo functioning. 2. Figure 2 is a picture of a 45-year-old man who has history of snoring during sleep, headache, tiredness, and daytime somnolence. How can the patient be screened for the underlying cause and how is the diagnosis confirmed ? (2*) Answer: Screening by Epworth sleepiness scale.

Fig. 2 Obese gentleman. Permission granted by Hull York Medical School (HYMS)

95

Polysomnography. Obesity is one of the important risk factors for obstructive sleep apnoea. The presence of obstructive sleep apnoea is screened using Epworth sleepiness scale and confirmed by polysomnography. Various endocrine conditions including untreated hypothyroidism, Cushing’s disease and acromegaly are risk factors for obstructive sleep apnoea. 3. A 54-year-old woman presented with a history of excessive thirst, sweating and headache. She underwent an oral glucose tolerance test that revealed a fasting plasma glucose of 7.0 mmol/L and a two hour postprandial plasma glucose of 13.2 mmol/L. What is the abnormality shown on the oral glucose tolerance test (OGTT) compared to reference ranges in Table 1? (2*) Table 1 Reference range Oral glucose tolerance test (OGTT). Courtesy of Prof. T Sathyapalan Fasting

2h

Normal

11.1 mmol/L

Answer: Diabetic range OGTT. Figure 3 belongs to the patient described in question 3. She was diagnosed with acromegaly and underwent a transsphenoidal surgery. Following surgery what parameters can be used to diagnose disease activity? (4*)

Fig. 3 Facial features in a patient with Acromegaly. Permission granted by Hull York Medical School (HYMS)

96

Fig. 4 Facial hair. Permission granted by Hull York Medical School (HYMS)

If tests showed continuing disease activity what are the other treatment options that can be used therapeutically? (3*) Answer: Insulin-like growth factor 1 (IGF-1) and OGTT with growth hormone levels. Somatostatin analogues and radiotherapy. 4. A 23-year-old woman presented with a history of increased facial hair, obesity and irregular menstrual periods since the age of menarche (13 years old). Her blood pressure was normal. Her face and ovarian ultrasound are shown in Figs. 4 and 5 respectively. She was found to have biochemical hyperandrogenism. What is the likely diagnosis? (3*) What are the medical options available to reduce facial hair (give two examples)? (3*) Answer: Polycystic ovary syndrome should be considered at the top of the differential diagnosis. It is however important to exclude other diseases that are associated with hyperandrogenism e.g., Congenital Adrenal Hyperplasia, Cushing’s syndrome. Antiandrogens: Spironolactone, Cyproterone acetate and Flutamide.

T. Sathyapalan et al.

Fig. 5 Ovarian ultrasound. Permission granted by Hull York Medical School (HYMS)

5. A 30-year-old woman whose picture is shown in Fig. 6 had been feeling warm for a few weeks. Her examination revealed a pulse rate of 120 beats per minute and she was agitated on examination. She was diagnosed with Graves’ thyrotoxicosis. What are the clinical features that distinguish Graves’ disease from other causes of hyperthyroidism? (2*) Answer: Orbitopathy, dermopathy and diffuse goitre with bruit. 6. A 56-year-old man presented with a history of diarrhoea of a few months duration. He recently started to have difficulty climbing the stairs. Examination showed only mild proximal myopathy. His biochemistry showed: Sodium (Na+): 138 mmol/L, Potassium (K+): 3.6 mmol/L, Urea: 4.7 mmol/L, Creatinine: 85 umol/L, Normal liver function tests. Adjusted calcium: 2.20 mmol/L, 25 Hydroxy Vitamin D: 15 nmol/L (normal range above 75 nmol/L), parathyroid hormone: 90 ng/L (normal range: 7-53). What is the most likely diagnosis? (1*) Secondary hyperparathyroidism due to vitamin D deficiency. 7. A 27-year-old man presented with a 4 month history of a throbbing headache. His blood pressure was elevated

Endocrinology

97

Fig. 7 MRI scan MRI scan showing a pituitary macroadenoma, marked with an arrow. Full and informed patient consent has been obtained to reproduce the image in a commercial publication

Answer: Pituitary Macroadenoma If the patient is to undergo surgery, what medical therapy might they need prior to surgery? Surgery on the pituitary can cause disruption in the secretion of ACTH and cause adrenal insufficiency rarely can be fatal. Hence corticosteroid replacement might be needed. Fig. 6 Graves’ Thyrotoxicosis. Permission granted by Hull York Medical School (HYMS)

175/100 mm Hg. His biochemistry revealed: Na+: 135 mmol/L, K+: 3.2 mmol/L, Urea: 3.2 mmol/L, Creatinine: 55 umol/L, normal liver function tests. What could be the cause for his hypertension and what initial screening test should be requested? (3*) Hyperaldosteronism Random aldosterone to renin ratio Primary hyperaldosteronism results from increase in secretion of aldosterone by adrenal cortex and results in reduction in renin due to negative feedback inhibition. Increased aldosterone will result in hypertension and hypokalemia.

9. Figure 8 is a retinal photograph of a 22-year-old male. What abnormality is shown? (1*) Answer: No abnormality (normal retina).

10. Figure 9 is a retinal photograph belongs to a 56-year-old patient with 10-year history of type 2 diabetes mellitus. Which diabetic retinopathy stage is shown in the retinal photograph and what is the next step in the management? (2*) Answer: New vessels formation.

8. This is an MRI of a 29-year-old woman who presented with secondary amenorrhoea. What is shown in the MRI (Fig. 7)? (3*)

Urgent referral to ophthalmology for laser treatment to prevent blindness.

98

T. Sathyapalan et al.

Fig. 10 Diabetic foot. Full and informed patient consent has been obtained to reproduce the image in a commercial publication Fig. 8 Retinal Photograph. Full and informed patient consent has been obtained to reproduce the image in a commercial publication

11. A patient with type 1 diabetes of 33-year duration presented after a beach holiday with an ulcer on the plantar aspect of the right foot (Fig. 10). What 3 factors contributed to the development of the ulcer? (3*) Answer: Poor peripheral circulation, lack of pain sensation in the foot and heat exposure caused by a combination of large vessel atheroma, small vessel disease and peripheral neuropathy.

12. The picture at the top shows a normal renal histology (Fig. 11). The picture at the bottom is the histology specimen of a renal biopsy taken from a patient with a long-standing history of type 1 diabetes, retinopathy and a recent onset generalised swelling (Fig. 12). What is the correct pathological process describing these changes and how can it be screened? (2*) Answer: Fig. 9 Retinal Photograph in a Diabetic. Full and informed patient consent has been obtained to reproduce the image in a commercial publication

Glomerulosclerosis.

Endocrinology

99

Fig. 13 Glucose response curve. Courtesy of Prof. T Sathyapalan

Fig. 11 Normal renal histology. Courtesy of Prof. T Sathyapalan

Fig. 14 Foot X-ray in a patient with diabetes. Full and informed patient consent has been obtained to reproduce the image in a commercial publication

Fig. 12 Abnormal renal histology. Courtesy of Prof. T Sathyapalan

When did bone pathology (see arrow in Fig. 14) start? (4*) What is the implication of this in clinical practice? (4*)

Urinary albumin: creatinine ratio. Answer: 13. Figure 13 shows the glucose response after a single subcutaneous injection of insulin. Which type of insulin has this glucose response curve? (3*)

At least 3 weeks ago.

Answer:

15. The picture in Fig. 15 is of a hand of a 39-year-old male who has a history of angina. There are similar recent onset lesions on the elbows and knees that appeared simultaneously. His fasting blood tests showed a normal plasma glucose but elevated serum cholesterol and triglyceride. What is the underlying primary lipid disorder and what is the underlying genetic abnormality? (4*)

Rapid acting insulin analogue. 14. Figure 14 is a foot x ray of a patient with a long history of type 2 diabetes and a slowly healing ulcer on the plantar aspect of the head of the first metatarsal of the right foot.

Osteomyelitis may be missed by a plain x-ray within the first 3 weeks.

100

T. Sathyapalan et al.

Fig. 16 Skin depigmentation. Permission granted by Hull York Medical School (HYMS)

16. Figure 16 is a picture of a 30-year-old female who presented with a patch of alopecia areata and skin depigmentation. Name 4 hormone deficiencies this patient is at risk of developing in the future. (2*) Answer: Fig. 15 Eruptive Xanthomata. Full and informed patient consent has been obtained to reproduce the image in a commercial publication

Answer: Type III hyperlipidaemia (familial dysbetalipoproteinemia). Presence of two apo E2 alleles. Familial dysbetalipoproteinemia (type III hyperlipoproteinemia) is a genetic disorder characterised by the accumulation of remnants of triglyceride-rich lipoproteins in the plasma. The remnant particles are susceptible to uptake by macrophages in peripheral tissues, which may become cholesterol-laden foam cells and lead to atherosclerosis. The most common feature associated with this condition is the presence of xanthomas, which are deposits of lipids in the skin and appear as multiple yellow papules on or just beneath the skin. Xanthomas may form on different parts of the body including the hands, elbows, knees, knuckles, arms, legs, and buttocks. Genotyping in these patients will show the presence of the apo E2 homozygous variant (E2/E2).

Oestrogen, cortisol, insulin, and thyroid hormones. Alopecia areata and vitiligo are two cutaneous associations of autoimmune disorders. They are associated with Addison’s disease, type 1 diabetes, hypo/hyperthyroidism and premature ovarian insufficiency.

17. Figure 17 a picture of a 60-year-old woman who had been complaining of lethargy, constipation, somnolence and feeling cold for a long duration. What is the likely diagnosis and how can you confirm the diagnosis? (1*) Answer: Myxoedema. Thyroid Function Test (TFT). The common symptoms of hypothyroidism are weight gain, cold intolerance, lethargy and Constipation.

Endocrinology

101

Fig. 17 Puffy face with coarse features. Permission granted by Hull York Medical School (HYMS)

The facial features of myxoedema are characterised by generalised puffiness, macroglossia, ptosis and periorbital oedema. The thyroid function tests will show low thyroid hormones (freeT4 and freeT3) and raised TSH due to loss of feedback control to pituitary gland.

18. A 31-year-old man presented to his General Practitioner with polyuria and polydipsia of a few weeks duration. On examination he had the skin lesion shown in the photograph in Fig. 18 and he also had fine inspiratory crackles on chest auscultation. He was not on any medication. What is the abnormality shown in the photograph? (3*) What is the mechanism of polyuria? (3*) Answer: Lupus pernio. Hypercalcaemia. Lupus pernio is a cutaneous manifestation of sarcoidosis. Polyuria and polydipsia occurring in sarcoidosis results from various processes including occurrence of primary or vasopressin-sensitive diabetes insipidus by discrete sarcoid lesions in the hypothalamic region, and the occurrence of polyuria with renal failure and specific tubular lesions that prevent the concentration of urine, due to hypercalcemia and unresponsive to vasopressin.

Fig. 18 Bluish red nose. Permission granted by Hull York Medical School (HYMS)

19. A 33-year-old woman presented with tiredness and weight loss for a few months. She underwent a short Synacthen® test. The result of the test is shown below: 9:00 AM Cortisol 89 nmol/L. 9:30 AM Cortisol 150 nmol/L. 9:00 AM Adrenocorticotropic hormone (ACTH): 1000 ng/L (reference range: 10–40). What is the likely underlying cause of this patient’s symptoms? (3*) What other hormones could be lacking in this condition? (3*) Answer: Primary Adrenocortical insufficiency. Aldosterone and adrenal androgens; dehydroepiandrosterone sulphate (DHEA-S). There is a suboptimal response of cortisol after short Synacthen® test.

102

T. Sathyapalan et al.

What is the underlying condition? What is the clinical implication for the nodule? (3*) Answer: Toxic adenoma. Very unlikely to be neoplastic. There is an intense bright spot in the region of right lobe of thyroid gland suggesting increased uptake suggestive of toxic adenoma. Toxic adenomas are unlikely to be malignant.

Fig. 19 Thyroid uptake scan. Full and informed patient consent has been obtained to reproduce the image in a commercial publication

21. A 44-year-old patient underwent pituitary surgery for a non-functioning pituitary adenoma. He then underwent an insulin tolerance test 12 weeks after surgery to determine his pituitary reserve and the result is shown in Table 2. What is the interpretation of the insulin tolerance test? (4*) Answer:

Normal response of cortisol after short Synacthen® test is >540 nmol/L. There is also an increase in ACTH at baseline due to loss of negative feedback inhibition of cortisol to pituitary gland suggesting primary adrenal insufficiency. Adrenal gland also produces other hormones. Aldosterone is produced by Zona glomerulosa, cortisol is produced by Zona fasciculata and adrenal androgens including dehydroepiandrosterone sulphate (DHEA-S) is produced by Zona reticulata of adrenal cortex.

20. A 45-year-old woman presented with heat intolerance and loss of weight of 6 months duration. Neck examination revealed a solitary thyroid nodule. Her thyroid function tests showed free T4: 28 pmol/L (normal range 9–24), Free T3: 34 pmol/L (normal range 2.5–5.3), TSH: 0.46 s is highly suggestive of the diagnosis of long QT syndrome c. How is QTc measured? (4*) Answer: QT interval is measured in lead II or V5-6, from the start of Q wave till the end of T wave. Bazett's Formula QTc = QT (in ms) / √RR (s). d. What are the genetics of long QT syndrome? (5*)

8. This boy was born at 30 weeks of gestation. The first weight measurement plotted on the the growth chart was taken by the community midwife. The health visitor saw him at 18 weeks postnatal age and plotted his weight on the growth chart (Fig. 12). a. What can explain this discrepancy in his weight plots? (2*)

120

S. Gupta and S. Besarovic

Fig. 12 Weight plotted on growth chart Courtesy of Dr. S Gupta

Answer:

Answer:

1. Genuine weight loss, which will need urgent review and investigations 2. Health visitor not correcting postnatal age for prematurity

1. Gestational correction is continued for 1 year for babies born 32–36 weeks. 2. For babies born before 32 weeks the plots should be corrected until 2 years postnatal age

b. At what gestational age will a baby be called preterm? (2*)

d. How could the discrepancy noted in the growth chart be correctly plotted? (4*)

Answer: Answer: Babies born between 37 completed weeks and up to 42 weeks of gestation are considered to be born at term. Hence any baby born before 37 completed weeks of gestation (up to 36 weeks and 6 days) will be preterm. For calculation of prematurity, gestation age of baby is subtracted from 40. Hence the baby described in this case is 10 weeks premature (40–30). c. How long is the gestational correction taken in account while plotting anthropometric parameters on a growth chart? (3*)

The weight should have been plotted at the postnatal age and then prematurity shown by drawing a dashed arrow going backwards on the chart to the corrected age for prematurity, as shown in Fig. 13. The weight centile measurement is read at the tip of arrow. Every observer should plot the measurements in such way to avoid confusion.

Pediatrics

121

Fig. 13 Growth chart showing weight corrected for prematurity Courtesy of Dr. S Gupta

9. A 4-year-old boy from middle eastern origin was referred by his gp due to concerns about poor growth and bowing of legs. these were the x-rays of his knees (Fig. 14). a. What findings are shown in the x-rays? (3*) Answer: 1. Cupping and fraying of the metaphyses 2. Widening of the growth plate 3. These findings suggest rickets (vitamin D deficiency is the commonest cause) b. What laboratory findings may you see for this child? (3*) Fig. 14 X ray of both knees. Full and informed parental consent has been obtained to reproduce the image in a commercial publication

Answer: 1. Normal or low plasma calcium 2. Low plasma phosphate

122

S. Gupta and S. Besarovic

3. Raised parathyroid hormone 4. Raised alkaline phosphatase 5. Low25 OH and 1, 25 (OH)2 vitamin D levels c. What are the causes of rickets? (5*) Answer: 1. Nutritional deficiency of vitamin D 2. Vitamin D dependent rickets (Autosomal recessive or vitamin D receptor problem) 3. Hypophosphatemic rickets (X linked dominant or autosomal recessive) 4. Renal failure including Fanconi syndrome 5. Osteopenia of prematurity d. How would you treat nutritional rickets? (4*)

Fig. 15 Chest X-ray of premature baby with respiratory distress Permission granted by Hull York Medical School (HYMS)

Answer: 1. High dose vitamin D (2,000–5,000 units/day) for 10– 12 weeks given orally 2. This should be followed by daily vitamin D intake of 400–800 units 10. A baby boy born at 28 weeks of gestation developed respiratory distress after 24 h of birth needing support with nasal CPAP. His chest radiograph is shown in Fig. 15. a. What does the x-ray show? (3*) Answer: 1. Widespread radio-opacity. 2. Air bronchogram on the left side

3. Lack of surfactant leads to failure of attainment of functional reserve capacity (FRC) and the alveoli become atelectatic d. Which group of infants are most at risk for developing RDS? (4*) Answer: 1. Prematurity, especially babies born below 28 weeks’ gestation 2. Maternal diabetes 3. Meconium aspiration at birth 4. Multiple births 5. Cesarean section births

b. What is the most likely diagnosis? (3*)

11. This was an incidental finding on a routine chest x-ray (Fig. 16).

Answer:

a. What does the X-ray show? (2*)

Respiratory distress syndrome (RDS); was historically called hyaline membrane disease.

Answer:

c. What is the pathogenesis of this condition? (5*)

1. Dextrocardia 2. Stomach air bubble on the right indicating situs inversus

Answer:

b. What further investigation is needed? (4*)

1. Surfactant deficiency is the primary cause of RDS 2. Surfactant is secreted in the type II alveolar cells and helps to reduce surface tension within pulmonary alveoli and hence aids alveolar expansion

Answer: 1. An Ultrasound examination to assess abdominal organs to confirm situs inversus and to detect splenic abnormalities

Pediatrics

123

Fig. 17 Inspection findings of perineum. Full and informed parental consent has been obtained to reproduce the image in a commercial publication Fig. 16 Chest X-ray showing position of vital structures. Full and informed parental consent has been obtained to reproduce the image in a commercial publication

2. Echocardiogram to identify congenital heart disease

5. Gastrointestinal anomalies such as malrotation, duodenal atresia or tracheo-esophageal fistula c. What do you understand by ‘high’ imperforate anus? (5*)

c. What is the prognosis? (5*) Answer: Answer: 1. Dextrocardia with associated situs inversus and normally related great arteries is unlikely to have any complications 2. Dextrocardia without situs inversus could be associated with severe cardiac malformations and asplenia which increases the risk of serious bacterial infections 12. This was the finding on routine neonatal check on postnatal ward (Fig. 17). a. What does the picture show? (3*)

1. Levator ani, puborectalis, external and internal anal sphincter form a sphincter complex around the anorectal region 2. If, during embryonic development, the rectum has descended below the sphincter complex such anomaly is called low imperforate anus. If rectum is above the sphincter complex it is high imperforate anus 3. Girls with high lesions usually have cloaca, in which rectum, vagina and urethra all empty in a common channel 13. A 13-year-old girl presents with tall stature and is found to have following signs on ophthalmology examination (Fig. 18).

Answer: a. What does the picture show? (3*) Imperforate anus. Answer: b. What anomalies may be associated with imperforate anus? (4*)

Ectopia lentis (subluxation of lens)

Answer:

b. What other clinical signs you may find in her? (4*)

1. Renal malformations 2. Uterine anomalies like bicornuate uterus 3. Spinal cord anomalies like sacral teratoma, spinal dysraphism and tethered cord 4. Congenital cardiac defect

Answer: 1. Increased arm span to height ratio 2. Arachnodactyly 3. Pectus excavatum or carinatum

124

Fig. 18 Eye abnormality in a child with tall stature. Permission granted by Aravind Eye Hospital and Postgraduate Institute, Madurai, India

4. Scoliosis 5. High arched palate and dental overcrowding. 6. Cardiac anomalies–aortic root dilatation and aortic regurgitation c. What are the differential diagnoses? (4*) Answer: 1. Marfan syndrome 2. Homocystinuria d. What investigations may be helpful? (5*)

S. Gupta and S. Besarovic

Fig. 19 Abnormal findings on retinal examination. Permission granted by Aravind Eye Hospital and Postgraduate Institute, Madurai, India

b. What is the inheritance and prognosis of this condition? (5*) Answer: 1. Autosomal recessive, autosomal dominant or X-linked recessive 2. Progressive loss of peripheral vision 3. Gradual reduction in retinal function as assessed by electroretinography (ERG) 4. Only supportive treatment is available c. What other conditions are you aware of where you may find pigmentary changes of the retina? (5*)

Answer: Answer: 1. Urinary organic acid screening to rule out homocystinuria 2. Echocardiogram 3. Genetic studies to look for mutation in fibrillin −1 (FBN1) gene on chromosome 15. 14. A 9-year-old boy presents with impairment of night vision and found to have following sign on ophthalmology examination (Fig. 19). a. What does the picture show? (4*) Answer: 1. This eye shows advanced presentation of retinitis pigmentosa, attenuated blood vessels and consecutive optic atrophy, evident as waxy pallor of the optic nerve head 2. The macula also shows some pigmentary degenerative changes. The disorder is almost always bilateral

1. Mucopolysaccharidoses (Hurler, Hunter and Sanfilippo syndrome) 2. Gangliosidosis (Batten–Mayou disease). 3. Mitochondrial disorder (Kearns-Sayre syndrome) 4. Laurence-Moon and Bardet-Biedl syndromes 5. Abetalipoproteinemia (BassenKornzweig syndrome) 6. Refsum disease It is important to distinguish secondary pigmentary changes from retinitis pigmentosa as some of the above-mentioned conditions are treatable.

15. The retinal image at 36 weeks post-conception age of a preterm baby born at 26 weeks’ gestation is shown in Fig. 20. a. What does the picture show? (4*)

Pediatrics

125

4. This line may grow into a ridge (as shown in picture) 5. Abnormal neo-vascularization (under the influence of vascular endothelial growth factors) may occur over retina and into the vitreous 6. Cicatrization and traction on retina may lead to retinal detachment c. What are the risk factors implicated as contributors for development of ROP? (5*) Answer:

Fig. 20 Retinal abnormality in a preterm baby Permission granted by Aravind Eye Hospital and Postgraduate Institute, Madurai, India

1. Prematurity and low birth weight 2. High inspired oxygen concentration produces free radical injury 3. Respiratory distress, hypercarbia and acidosis 4. Blood transfusion

Answer:

d. What is the treatment of this condition? (5*)

1. Dilated venules and tortuous arterioles. 2. A fibrovascular ridge in zone 2 of retina

Answer:

These appearances are typical of retinopathy of prematurity (ROP) b. What is the pathogenesis of this condition? (5*) Answer: 1. Retinal vascularization starts at 16 weeks of gestation and proceeds from optic disc towards periphery of retina 2. Between 36- and 40-weeks vascularization of retina is complete 3. Any interruption to this process leads to termination of vascularization (visible as a demarcation line in the retina)

In selected cases laser photocoagulation of the avascular retina is indicated to prevent progression of ROP.

Further Readings 1. Tasker RC, McClure RJ, Acerini CL. Oxford Handbook of Paediatrics. 3rd ed. Oxford: Oxford University Press; 2020. 2. Lissauer T, Carroll W. Illustrated Textbook of Pediatrics. 6th ed. Amsterdam: Elsevier; 2021. 3. Marcdante K, Kliegman RM. Nelson Essentials of Pediatrics. 8th ed. St. Philadelphia: Elsevier-Health Sciences Division; 2018. 4. Johnson S. A Clinical Handbook on Child Development Pediatrics. Amsterdam: Elsevier; 2012.

Cardiology Kenneth Wong, Abdullah Abdullah, and Renjith Antony

Learning Outcomes

Images and Online E supplement (Videos)

• • • • • •

EMQs (extended matching questions)

Systematic reporting of ECG Recognize common disturbances of rhythm Discuss management of acute heart failure Outline the consequences of common heart valve disease Better understand heart murmurs List causes of dyspnea and chest pain.

EMQ: Area: Causes of Chest Pain Options:

Resources to help achieve these learning outcomes EMQs to guide learning of causes of chest pain and dyspnea. Section 1: Top tips for ECG interpretation (a) Palpitations (b) Syncope (c) Dyspnea Section 2: Visualizing graphy made Easy

Heart

(a) (b) (c) (d) (e) (f) (g) (h) (i) (j)

Stable angina ST elevation myocardial infarction (MI) Non-ST elevation acute coronary syndrome Acute pericarditis Aortic dissection Pulmonary embolus Chest infection Gastro-oesophageal reflux disease Costochondritis Herpes zoster

Murmurs-EchocardioLead in:

MCQs to test learning outcomes

For each of the following presentations of chest pain, choose the most likely cause from the list above.

Supplementary Information The online version contains supplementary material available at https://doi.org/10.1007/978-3-031-24465-0_7. The videos can be accessed individually by clicking the DOI link in the accompanying figure caption or by scanning this link with the SN More Media App.

Stems:

K. Wong (&) . A. Abdullah Department of Cardiology, Blackpool Teaching Hospitals NHS Foundation Trust, Blackpool, FY3 8NR, UK e-mail: [email protected]

1. A 40 year old female patient complains of severe sharp left sided chest pain worse on inspiration, better leaning forward. A pericardial rub is audible. ECG shows widespread saddle shape ST elevation and PR depression (3*) (Fig. 1).

A. Abdullah e-mail: [email protected]

Answer: (d) acute pericarditis

R. Antony Department of Cardiology, Hull University Teaching Hospitals NHS Trust, UK (HUTH), Hull, HU3 2JZ, UK e-mail: [email protected]

2. A 69 year old woman who develops sudden onset of severe left sided chest pain worse on inspiration associated with shortness of breath and hemoptysis. She

© The Author(s), under exclusive license to Springer Nature Switzerland AG 2023 K. Wong et al. (eds.), Practical Guide to Visualizing Medicine, https://doi.org/10.1007/978-3-031-24465-0_7

127

128

K. Wong et al.

Fig. 1 ECG of a patient with chest pain worse on inspiration, better leaning forward

recently returned to the UK from India. Her left leg is swollen and painful (2*).

Stems:

EMQ:

1. A 60 year old man presents with dyspnea which has gradually got worse in the last few days associated with a cough producing green sputum. He has a 70 pack-year smoking history. Widespread wheeze is audible in both lungs (2*).

Area: Causes of Dyspnea

Answer: (g) infective exacerbation of COPD

Options:

2. A 70 year old woman presents with gradually worsening dyspnea. On examination HR 90 beats/min., regular. There is a loud pansystolic murmur, loudest in the apex, radiating to the axilla (3*).

Answer: (f) Pulmonary embolus

(a) (b) (c) (d) (e) (f) (g)

Left ventricular failure Arrhythmia Mitral regurgitation Aortic stenosis Mitral stenosis Pulmonary hypertension Infective exacerbation of Chronic Obstructive Pulmonary Disease (COPD) (h) Acute asthma (i) Pneumothorax (j) Anemia

Lead in: For each of the following presentations of dyspnea, choose the most likely cause from the list above.

Answer: (c) mitral regurgitation 3. A 51 year old man presents with sudden onset of shortness of breath waking him up from sleep. He had an anterior MI 1 year ago and normally gets breathless after walking 50 yards. On examination, inspiratory crackles are audible throughout both lungs, 3rd and 4th heart sounds are noted (2*). Figure 2 Chest X-ray (radiograph) of patient who woke up breathless. Answer: (a) left ventricular failure

Cardiology

Fig. 2 CXR of patient who woke up breathless

Section 1: Top tips for ECG interpretation Always check the patients’ name and date, then look at the scale (is it 25 mm/second, as usual, on the x-axis? 10 mm/mV on the y axis?) Systematic reporting: (a) What is the heart rate? (300/number of big squares or 1500/number of small squares between RR) (b) What is the heart rhythm? (c) QRS axis (normal QRS axis ranges from −30 to +90 degree) Each limb lead “looks at” the heart from a different angle (See schematic image in Fig. 3).

129

Thus, I and aVL look at the lateral wall, and leads II, III, aVF look at the inferior wall of the left ventricle. If the cardiac depolarization vector (or QRS axis) travels towards a lead, then the QRS complex is predominantly positive. If the QRS axis travels away from a lead, then it is predominantly negative. Normal QRS axis ranges from −30 to +90. Left axis deviation means the cardiac depolarization vector (QRS axis) travels in a direction beyond −30 degree. Right axis deviation means the QRS axis travels in a direction beyond +90 degree. You often see right axis deviation if the patient has a hypertrophied right ventricle A quick scan of I and II often gives you the correct conclusion. If the QRS complex of both leads I and II are predominantly positive, as in Fig. 1, then the QRS complex is normal. If lead I is predominantly negative, consider if the patient has right axis deviation. If lead II is predominantly negative, you may hypothesize that the patient has left axis deviation. But, you should confirm the hypothesis by doing the following test. You should test your hypothesis by examining whether the QRS complexes are predominantly positive in the leads where you reckon the cardiac vector (QRS axis) is travelling towards. e.g. If lead I is predominantly negative, then you would hypothesize that the patient has RIGHT axis deviation. The QRS axis has to be beyond + 90, but it can also be beyond minus 90 degrees—ie extreme left. So examine leads II, III and AVF. If they are positive, then your patient has right axis deviation. If however, they are also negative, then your patient’s QRS axis must be pointing beyond minus 90 degree to about −150. In this scenario, you would expect aVR to be ……… POSITIVE. Yes, you have got it. If you haven’t, then read the above again. It is rewarding to master QRS axis assessment! Now we move onto examine the rest of the ECG: (d) The intervals:

Fig. 3 ECG QRS axis

PR interval (120–200 ms is normal: check that the ECG is recorded at 25 mm/s. If so, normal PR interval is between 3 and 5 small squares). QRS duration ( 3.5 mV (or 35 mm if ECG is recorded at 10 mm/mV). ST elevation—a medical emergency-recognising this quickly and prompt revascularisation by primary percutaneous coronary intervention (PPCI) can save the amount of myocardium getting infarcted. You may save the patient’s life. ST depression—one of the most common causes is myocardial ischemia. If possible, compare the ECG when the patient had symptoms of angina with an ECG when the patient did not have angina. The likelihood of ischemia is higher if there is “dynamic ST depression”—i.e. ST depression with pain, and no ST depression when painfree. T wave abnormalities (inversion-seen in ischemic heart disease (IHD) / flat-seen in IHD or hypokalemia/peakedsuspect significant hyperkalemia, usually with potassium 6.5 or more).

Pitfalls/Mistakes to avoid ST elevation or depression is most commonly due to MI/ acute coronary syndrome, but consider differential diagnoses, e.g. do not misdiagnose acute pericarditis (Fig. 1) as ST elevation MI; ST depression can be caused by LVH with strain. Always take into account the patient’s symptoms. Remember to interpret ECGs (as all other images/tests) in the clinical context.

R V3, V4 (mirror image of V3 and V4 on the right side of the chest) looks at the right ventricle Section 1A: Palpitations Palpitations (awareness of heartbeat) is one of the commonest presenting complaints to the Cardiology clinic. Indeed, many patients would present to the Emergency room with palpitations in association with chest pain, dyspnea, or syncope. A good history really helps visualize the pathology. For instance, if a patient experiences sudden onset of fast and irregular palpitations before the onset of chest pain, that would suggest a diagnosis of atrial fibrillation with rapid ventricular response as a cause of the patient’s myocardial ischemia (possibly unmasking underlying coronary artery disease). This section is an excellent opportunity for you to put into practice what you have learnt about systematic ECG reporting. Causes of palpitations Cardiac dysrhythmias (rhythm disturbances, sometimes described as “arrhythmias”) can be broadly classified as 1. Tachycardia (Broad QRS complex suggesting ventricular in origin, or narrow complex (supraventricular), and can be regular or irregular), or 2. Bradycardia (which includes asystole). But very frequently, patients have normal ECGs (sinus rhythm) even during the episodes of palpitations recorded during ambulatory monitoring. Another common finding is ectopic beat (extra beat which comes too early). The patient may sense the beat after the compensatory pause following the ectopic beat. The Frank–Starling mechanism would help you understand that the more the ventricle fills during the longer diastolic time interval, the greater the stroke volume. After reading chapter Introduction: Cultivation of Clinical Reasoning (Introduction), you would appreciate the distinction between diagnosis, its causes and consequences. People often talk about “rhythm diagnosis” on ECGs, but we

Cardiology

must remember the abnormal rhythm is really a “consequence” of the diagnosis. So, whenever possible, you should try to make a definitive diagnosis that causes the dysrhythmia e.g. myocardial infarction (MI) causing Ventricular tachycardia (VT). Clearly in the context of a cardiac arrest, for instance due to Ventricular fibrillation (VF), your priority is to defibrillate the patient first, so in that sense, making the rhythm diagnosis immediately and administering emergency treatment would be your first priority. For the student who is keen to use the diagnostic sieve, first described in chapter Introduction: Cultivation of Clinical Reasoning (Introduction chapter), we have outlined in Table 1 many of the common diagnoses behind the ECG dysrhythmias which are often seen in clinic or in the Cardiology wards, and some of the rarer causes which you should know about. You may be able to add more to the table. In fact, one might argue that the vast majority of cardiac problems, and indeed many common general medical problems (e.g. infection) are associated with cardiac dysrhythmias, even though palpitations might not be the presenting symptom. So, the causes listed in the table are not intended to be exhaustive, but serve to get you thinking about the merits and challenges of using such a tool to consider differential diagnoses (e.g. systematic versus the need to learn pattern recognition and rapidly see the “wood for the trees”). In any case, remember to interpret ECGs in the clinical context. Now is your chance to practise systematic reporting of ECGs. To keep the text short, I have only included the key abnormalities in the answers.

131

Answer: Atrial fibrillation (AF)- irregularly irregular, with no discernable P waves The patient also has left bundle branch block (broad QRS duration, RSR’ pattern in V6). 3rd ECG. This patient has suffered from palpitations (Fig. 6). What does the ECG show? Answer: Delta waves (slurred upstroke of the QRS complex eg lead V3), short PR interval ( 30 g/L, lactic dehydrogenase > 200 U/L). There is no evidence of malignancy and direct pleural infection is not confirmed (negative microbiology), so in the context of the history the working diagnosis is a parapneumonic pleural effusion secondary to pneumonia. This is a photograph of pleural fluid being removed with a chest drain (Fig. 15a and b). 10. a 75-year-old man complains that he is short of breath when walking for 2 or 3 min. This has been getting worse for about one year. His medical history is of well-controlled hypertension, treated with amlodipine. There is no history of other medical conditions and no family history of lung disease. He smoked tobacco until 20 years ago. He is a retired

Answer: Pleural effusions are categorised as either exudates or transudates based on biochemical analysis of the pleural fluid, so the next step is to perform pleural fluid aspiration under ultrasound guidance. Transudative effusions are usually bilateral because they are caused by systemic problems such as fluid overload, cardiac failure, or hypoproteinemia. Look for a history indicating cardiac or renal disease and examination findings of fluid overload. Exudates are often unilateral. Causes include pleural infection (empyema), inflammatory ‘parapneumonic’ effusions, tumour (primary or secondary), trauma (haemothorax), or connective tissue disease (which can also be bilateral). A history of trauma is usually obvious. Malignancy may be from primary pleural mesothelioma, lung cancer, or metastatic cancer. Look for a history of a previous cancer and recent systemic symptoms such as weight loss.

Fig. 15 a and b An intercostal chest drain has been inserted under ultrasound guidance in the ‘triangle of safety’ below the patient’s left axilla. It has been secured in place and connected to an underwater drainage bottle that allows fluid to escape but will not allow ingress of air. Full and informed patient consent has been obtained to reproduce the image in a commercial publication.

Respiratory Medicine

169

taxi driver. He keeps no pets or birds. On examination, there is early finger clubbing and bilateral fine crackles at both lung bases. A chest X-ray (Fig. 16) is performed, and he is referred to the hospital clinic where he undergoes CT scanning (Fig. 17), lung function testing (Fig. 18, Table 5) and a 6-min walk test (Table 6). What diagnoses would you consider in a patient with slowly progressive breathlessness? (3*) What is the likely diagnosis in this case? (3*) What risk factors for pulmonary fibrosis would you ask about? (3*) How would you interpret the lung function test results? (4*) Fig. 17 CT scan of chest

How would you manage this patient? (5*) What diagnoses would you consider in a patient with slowly progressive breathlessness? (3*) Answer: Chronic breathlessness can be caused by respiratory disease, heart disease, neuromuscular disease affecting the respiratory muscles, anaemia, obesity, chest wall deformity, physical deconditioning, or dysfunctional breathing, or a combination of these. The history and physical examination are key to discriminating these conditions.

What is the diagnosis in this case? (3*) Answer: In this patient with progressive breathlessness, the key findings are the presence of bilateral crackles and finger clubbing, suggesting either pulmonary fibrosis or bronchiectasis. The lack of a history of chronic sputum production and fine nature to the crackles indicate pulmonary fibrosis. The typical history in pulmonary fibrosis is of progressive worsening breathlessness on exertion, with or without a dry cough, typically in an older person (>50 years). On examination look for finger clubbing (present in 60%) and the typical fine, late inspiratory, ‘Velcro®-like’ crackles, usually bilateral and most pronounced at the lung bases. The patient’s chest X-ray shows bilateral reticular opacities (worse in the right lung in this case) consistent with an interstitial lung disease. The CT scan shows peripheral reticulation (lines) and honeycomb change consistent with a ‘usual interstitial pneumonia’ (UIP) pattern of lung fibrosis. In this patient, the UIP pattern of pulmonary fibrosis with no identified cause means the diagnosis is idiopathic pulmonary fibrosis (IPF) [4]. What risk factors for pulmonary fibrosis would you look for? (3*) Answer:

Fig. 16 Chest X-ray

Approximately 1 in 10 patients with pulmonary fibrosis will have an affected first degree relative. Smoking tobacco,

170

S. Hart et al.

either currently or in the past, increases pulmonary fibrosis risk (about 60% of patients with idiopathic pulmonary fibrosis have a history of smoking). Occupational exposure to asbestos, wood or metal dust are important factors to elicit in history. A history of inhaled organic antigen exposure (pigeons, caged birds, moulds) should lead to suspicion of hypersensitivity pneumonitis (extrinsic allergic alveolitis). Strong associations have been demonstrated between gastro-oesophageal reflux and pulmonary fibrosis, although esophageal dysmotility is probably equally common. A history of fibrogenic medication use should be specifically asked about, specifically bleomycin, amiodarone, or long-term nitrofurantoin. On examination, features of a connective tissue disease should be sought since pulmonary fibrosis commonly accompanies rheumatoid arthritis, scleroderma, and the anti-synthetase syndromes. Top tip: Fig. 18 Spirometry and flow-volume loop. The left panel shows the expiratory spirometry trace. The predicted FEV1 for the patient is shown by the intersection of the dotted green lines. X-axis, time (seconds); y-axis, volume expired (litres). The right panel shows the flow volume loop. The predicted expiratory loop is shown by the dotted green line. X-axis, volume (litres); y-axis, flow (litres/second) Table 5 Lung function test results. FEV1, forced expiratory volume in 1 s; FVC, forced vital capacity; FRC, functional residual capacity; RV, residual volume; TLC, total lung capacity; TLco, gas transfer factor for carbon monoxide; Kco, gas transfer coefficient (TLco/Va); Va, alveolar volume; SR, standardized residual

In patients with breathlessness do not forget to ask about exposure to caged birds How would you interpret the lung function test results? (4*)

Measured

Predicted

% Pred

SR

Spirometry FEV1

2.13

3.13

68

-2.62

FVC

2.67

3.62

74

-2.21

FEV1/FVC

80

81

FRC

2.52

2.87

88

−0.69

RV

1.19

1.75

68

−1.61

TLC

3.85

5.50

70

−2.74

RV/TLC

31

33

94

−0.35

Tlco

4.55

9.23

49

−4.00

Kco

1.24

1.68

74

Va

3.6

-0.20

Lung volumes

Gas transfer

Table 6 Six-minute walk test

6MWT

SpO2 (%)

0 min

93

HR (bpm) 88

1 min

88

99

2 min

87

97

3 min

86

105

4 min

89

109

5 min

89

115

6 min

86

115

Distance walked = 120 m

Respiratory Medicine

Answer: There is a restrictive pattern of spirometry with reduced FVC (and FEV1 reduced in proportion), low lung volumes (reduced TLC), and reduced transfer factor. This is typical of pulmonary fibrosis. There is oxygen desaturation on exercise which highlights the gas exchange problem at the alveolar level. How would you manage this patient? (5*) Diagnosis and management plan are agreed by an interstitial lung disease multidisciplinary team. Pulmonary rehabilitation comprises education, breathlessness management, and tailored exercise. Long-term home oxygen is only offered to patients who have severe resting hypoxaemia. In this case there is mild resting hypoxaemia, but exercise induced hypoxaemia may merit a trial of ambulatory oxygen. Anti-fibrotic drug therapy with pirfenidone or nintedanib should be offered; both have been shown to slow decline in lung function over time. Suitable patients should be referred for lung transplantation. Top tips: Lung function test interpretation Airways obstruction is characterised by FEV1/FVC ratio < 0.7 Inconsistent spirometry traces suggest that the patient is not providing a maximal effort

171

In COPD, the severity of airways obstruction is indicated by FEV1 expressed as percentage of predicted In asthma, spirometry and peak expiratory flow rate may be normal if symptoms are controlled Diffusing factor for carbon monoxide (TLco, transfer factor) is reduced in alveolar disease (e.g., pulmonary fibrosis) or alveolar destruction (e.g., emphysema), or V/Q mismatch (e.g., pulmonary hypertension) TLco is normal in patients with asthma TLco may be increased in the presence of alveolar haemorrhage.

References 1. Konstantinides SV, Meyer G. The 2019 ESC guidelines on the diagnosis and management of acute pulmonary embolism. Eur Heart J. 2019;40(42):3453–5. 2. MacDuff A, Arnold A, Harvey J, Group BPDG. Management of spontaneous pneumothorax: British thoracic society pleural disease guideline. Thorax. 2010;65 Suppl 2:ii18–31. 3. Hill AT, Sullivan AL, Chalmers JD, De Soyza A, Elborn SJ, Floto AR, et al. British thoracic society guideline for bronchiectasis in adults. Thorax. 2019;74(1):1–69. 4. Raghu G, Remy-Jardin M, Myers JL, Richeldi L, Ryerson CJ, Lederer DJ, et al. Diagnosis of idiopathic pulmonary fibrosis: an official ATS/ERS/JRS/ALAT clinical practice guideline. Am J Respir Crit Care Med. 2018;198(5):e44–e68.

Anesthesia and Intensive Care Marc Turnbull, Balaji Packianathaswamy, and Nabeel Siddiqui

Learning Outcomes • Describe the principles of pre-operative assessment of a patient prior to anesthesia. • Be able to assess a patient’s airway to predict difficulty with bag-mask ventilation or intubation. • Describe the steps in management of a patient’s airway including where airway management is unexpectedly difficult. • Describe some drugs commonly used in anesthesia. • Describe the management of some anesthetic emergencies. • Describe how pain can be managed in women in labor. • Describe the organ support available on ICU. Pre study Questions MCQs 1. A 46-year-old woman is anesthetized for an elective laparoscopic cholecystectomy. After 3 attempts at laryngoscopy, the vocal cords cannot be seen, and an endotracheal tube cannot be placed. After calling for assistance, what is the next course of action? A. An Attempt to intubate the patient using a different size laryngoscope blade B. Perform emergency cricothyroidotomy C. Attempt to intubate the patient using a fiberoptic scope M. Turnbull . B. Packianathaswamy (&) Department of Anesthesia, Hull University Teaching Hospitals NHS Trust, Hull, UK e-mail: [email protected] M. Turnbull e-mail: [email protected] N. Siddiqui Department of Anesthesia and Intensive Care, Royal Victoria Infirmary, Newcastle Upon Tyne, Tyne and Wear, UK e-mail: [email protected]

D. Declare a failed intubation and place a supraglottic airway device E. Adjust the patient's position and switch to a video laryngoscope Answer: D 2. A 76-year-old man attends the pre-operative assessment clinic one week before his elective total knee replacement. He complains of progressive shortness of breath on exertion over the last few years with occasional chest pain and dizziness on strenuous activity. On examination you note an ejection systolic murmur radiating to the carotids. What is the next course of action? A. Proceed with surgery but advise the patient that he will require spinal anesthesia B. Delay surgery and advice the patient that you need to arrange more investigations to assess the risk associated with anesthesia C. Cancel surgery and politely tell the patient that he is unfortunately not fit for this operation D. Proceed with surgery. Fully document your assessment and allow the anesthetist on the list to discuss the anesthetic plan with the patient E. Send the patient immediately to Accident and Emergency (A&E) Answer: B 3. A 55-year-old presents with community acquired pneumonia. On assessment his heart rate is 95, blood pressure 85/40, oxygen(O2) saturation 95 on 35% oxygen, Glasgow Coma scale (GCS) 15/15. He has been anuric for the last 2 h and has a partially compensated metabolic acidosis with a lactate of 3.5. He has been given appropriate antibiotics IV and 3 L of crystalloid but has not improved. What is the next best management step? A. give vasopressor support B. continue fluid resuscitation and review

© The Author(s), under exclusive license to Springer Nature Switzerland AG 2023 K. Wong et al. (eds.), Practical Guide to Visualizing Medicine, https://doi.org/10.1007/978-3-031-24465-0_9

173

174

M. Turnbull et al.

C. institute renal replacement therapy D. intubate and ventilate E. institute extracorporeal membrane oxygenation (ECMO)

condition. It also allows teams to prioritize patients when planning operating lists. What categories are included in the classification of intervention? (2*)

Answer: A Answer: 4. A 67-year-old man with an infective exacerbation of COPD requires intubation for worsening type 2 respiratory failure despite non-invasive ventilation. Complications of mechanical ventilation include (select all that apply): A. Pneumothorax B. Raised CSF pressure C. Interstitial emphysema D. Pulmonary infection E. Retinal haemorrhage Answer: A, C & D 5. A 44-year-old man requires mechanical ventilation for type 1 respiratory failure secondary to influenza pneumonia. After five days ventilation is becoming more difficult with oxygen requirement and peak inspiratory pressures rising. Features of adult respiratory distress syndrome (ARDS) include (select all that apply): A. A Severe hypoxemia B. B Severe Carbon Dioxide (CO2) retention C. C Reduced lung compliance D. D Reduced functional residual capacity (FRC) E. E Large shunt Answer: A, C, D & E

Anesthesia and Pain Management 1. A 66-year-old man is booked for an expedited (using the National Confidential Enquiry into Patient Outcome and Death (NCEPOD) classification) Hartmann’s procedure for an obstructing sigmoid tumor. What is the National Confidential Enquiry into Patient Outcome and Death (NCEPOD) classification of interventions? (1*) Answer: The NCEPOD classification asks within what timescale the operation should be completed based on the severity of the patient’s condition. This will impact the time available for assessment, investigation, and optimization of their

NCEPOD classification: • Immediate (class 1)—Immediate life, limb, or organ-saving procedures. Operate within minutes of decision. Do not delay resuscitation or perform simultaneous resuscitation during intervention. E.g., Abdominal Aortic Aneurysm (AAA) rupture, testicular torsion, acute limb compartment syndrome. • Urgent (class 2)—Potentially life-threatening condition, where limb/organ may become threatened, or for relief of acutely painful conditions. Operate within hours of decision. E.g., acute appendicitis, some fractures. • Expedited (class 3)—Condition not currently a threat to life but early treatment is required. Operate within days of decision. E.g., obstructing tumors. • Elective (class 4)—Planned and booked ahead of time. Timing to suit patients, hospital, and staff. e.g., knee replacement. What history should be obtained from the patient before surgery? See Fig. 1. (3*) Answer: The patient should be reviewed by an anesthetist in the pre-assessment clinic, on the ward, or, in very exceptional situations for immediate operations, in the anesthetic room prior to anesthesia. When assessing a patient, their identity (ID) and expected operation site or side should be confirmed. They should be asked about past medical history and review of systems completed. Asking about the history of acid reflux or gastro esophageal reflux may indicate a need to perform a rapid sequence induction or, for operations where use of a supraglottic airway device may be possible, to intubate the patient in the first place. Past surgical history and previous anesthetics, including personal or family history of anesthetic complications, helps to predict some difficulties occasionally encountered during anesthesia (to be covered later). The patient should be asked about any current acute illness unrelated to the reason for surgery.

Anesthesia and Intensive Care

175

Fig. 1 Anesthetic assessment for a fictional patient booked for Hartmann’s procedure

Exercise tolerance gives an indication of the patient’s overall cardiovascular fitness and ability to tolerate the physiological stress of surgery. Medication history should be considered including medications taken today. Allergies and intolerances must be confirmed. Social history, smoking, and alcohol history should be checked. Starvation time is important to consider. Any dental work or loose teeth may be damaged during airway manipulation or may affect the ease of airway management. What other information should be obtained about this patient? (4*)

The patient’s medical notes and previous anesthetic charts should be reviewed. Investigations will depend on the patient’s condition or co-morbidities. The patient’s weight may be required for calculation of drug doses or programming of infusion pumps. 2. 60-year-old woman has been booked for wide local excision for breast cancer and undergoes an assessment by the anesthetist (Fig. 2). She has asthma. She uses a Fostair (beclomethasone and formoterol) and Tiotropium inhaler regularly. She carries a salbutamol inhaler which she uses a few times per month. She is not limited by shortness of breath. What further information would you like to know regarding her asthma? (1*)

Answer: Answer: As well as taking a history, an examination should be performed. A brief cardiovascular and respiratory examination is generally indicated. A full airway assessment should be performed for all patients, regardless of the planned mode of anesthesia.

When considering any comorbidity, it is important to consider how severe or well controlled it is. For this patient, how well controlled her asthma is currently? Usual inhalers and pattern of use, shown in Fig. 3.

176

M. Turnbull et al.

Fig. 2 Anesthetic assessment for a fictional patient booked for wide local excision of breast cancer

When her last exacerbation occurred, and usual frequency of exacerbations. Any recent change in symptoms may indicate a current or impending exacerbation. Hospital admissions related to asthma include a need for non-invasive or mechanical ventilation. Limitation in activity related to asthma symptoms. Is her asthma triggered by non-steroidal anti-inflammatory drugs (NSAIDS)? Her current peak flow is related to her usual peak flow.

• ASA 3—severe systemic disease which limits activity. • ASA 4—severe systemic disease which is a constant threat to life. • ASA 5—moribund patient not expected to survive 24 h with or without surgery. • ASA 6—organ donor. What is this patient’s ASA grade? (3*) Answer:

Describe the American Society of Anesthesiologists (ASA) physical status classification? (2*)

This patient has a mild to moderate systemic illness which is well controlled. She is therefore classified as ASA 2.

Answer:

Explain the capnography trace in Fig. 4. obtained after the patient is connected to the ventilator. How would you treat this? (4*)

The ASA (American Society of Anesthesiologists) physical status classification is used to (crudely) predict perioperative risk and to allow easy communication of this risk between specialties. The classification is: • ASA 1—fit and well non-smoker. • ASA 2—mild to moderate but well controlled systemic illness which is not limiting.

Answer: This capnography trace shows an upward slope rather than a plateau on expiration caused by unequal emptying of alveoli. This is likely to be caused by bronchospasm. In this case the patient should be assessed taking the ABC approach to rule

Anesthesia and Intensive Care

177

out other causes including anaphylaxis, circuit blockage and misplaced airway. Inhaled fraction of oxygen should be increased, help sought, and surgery stopped if ventilation is difficult, or the patient is hypoxic. Bronchospasm in an anesthetized patient may be improved by increasing depth of anesthesia (particularly, sevoflurane); nebulized or IV salbutamol; nebulized ipratropium; nebulized, IM or IV adrenaline; IV magnesium; IV ketamine; IV amiodarone; and IV hydrocortisone. 3. A 35-year-old man is booked for a general anesthetic for an urgent laparoscopic appendicectomy. What features from a patient’s history might indicate potential difficult bag-mask ventilation or difficult intubation? (1*) Answer:

Fig. 3 Inhalers. Left—Fostair (beclomethasone and formoterol) inhaler. Right—Tiotropium inhaler

Airway difficulty is predicted from the patient’s history and from examination. Difficulties may arise from an inability to maintain oxygenation through bag-mask ventilation, difficulty in placement of a supraglottic airway device, or problems siting an endotracheal (ET) tube. Problems with ET tube siting can be further subdivided into difficulty visualizing the glottic opening and difficulty passing the ET tube. History of difficult airway or failed intubation may be reported by the patient, noted in their GP record or their hospital notes (Fig. 5). Patients may also carry a DAS (Difficult Airway Society) alert card or be registered on the DAS difficult airway database.

Fig. 4 Anesthetic monitor showing the pulse oximeter (oxygen saturation) trace in yellow above and end tidal (ET) carbon dioxide (capnography or etCO2) trace in white below

178

M. Turnbull et al.

Fig. 5 Patient alert sheet from the front of the medical notes in a fictional patient

Pre-existing conditions predisposing to airway difficulty include systemic disease, previous head and neck pathology, and many genetic syndromes. Systemic conditions include pregnancy, obesity, obstructive sleep apnea (OSA), rheumatoid arthritis, ankylosing spondylosis, diabetes mellitus, and angioedema. Pre-existing head and neck pathology may lead to airway difficulties including previous head and neck cancer treated with surgery or radiotherapy, thyroid disease, previous head or neck trauma, and treated cervical radiculopathy. Genetic syndromes associated with craniofacial abnormalities including Pierre Robin, Trisomy 21, Treacher Collins, and Klippel-Feil. Presenting pathology may also cause airway difficulty. This includes head and neck burns, tumors, infections, or trauma. How would you assess a patient’s airway? (2*)

Answer: All patients undergoing general or regional anesthesia should have their airway assessed. An airway assessment starts with a general examination of the patient’s body habitus and Body Mass Index (BMI). Their overall facial structure is then considered, looking for anatomical variations, craniofacial deformity, and facial hair. Mouth opening, Mallampati (MP) score (Fig. 6), and dentition are then assessed by asking the patient to open their mouth as wide as possible and protrude their tongue while in the sitting position. MP scoring is graded from one to four: MP 1—tonsillar pillars, soft palate, and entire uvula visible MP2—soft palate and a proportion of the uvular visible

Anesthesia and Intensive Care

179

Fig. 6 Open mouth views showing Mallampati (MP) grades 2 to 4. a MP 2 soft palate and part of uvula visible. b MP 3 only base of uvula visible. c MP 4 only hard palate visible. (Image taken on authors as models)

MP3—only base of uvula visible MP4—soft palate and uvula not visible at all Neck structure, neck movement, thyromental distance, and sternomental distance are assessed by asking the patient to maximally extend their neck. Thyromental distance can then be measured from the thyroid protuberance to the tip of the jaw. Sternomental distance is measured from the sternal notch to the tip of the jaw. Jaw movement can be further assessed by asking the patient to protrude their lower jaw forward in front of their upper, or to attempt to bite their top lip. Jaw movement is graded: A. if the patient can move their lower central incisors ahead of the upper B. if they can bring them level C. if they have an overbite even when trying to bring their lower jaw forward The patient’s nose should be assessed for nostril patency and septal deviation.

On general examination central obesity associated with a larger neck circumference may predict difficult bag-mask ventilation as does the presence of a beard. A small lower jaw or overbite is often predictive of difficult laryngoscopy. Being edentulous is likely to make bag-mask ventilation more difficult but may make insertion of a laryngoscope easier. MP grades one and two are reassuring whereas three and four can predict difficulty. Mouth opening is likely to allow laryngoscope insertion if more than 3 cm. Thyromental distance >6.5 cm and sternomental distance >12.5 cm are both reassuring. A receding lower jaw, or inability to protrude the lower jaw beyond the upper incisors, may suggest difficult laryngoscopy. Where, despite assessment, intubation difficulties arise this is termed unanticipated difficult intubation. Describe the findings for the patient in Fig. 7 (4*) What do you predict from this examination? (4*) Answer:

What features may suggest that intubation or bag-mask ventilation may be difficult when assessing a patient’s airway? (3*) Answer: No single finding has a high positive or negative predictive value for difficulty and factors leading to difficult bag mask ventilation differ from those predicting difficult laryngoscopy. When examining a patient’s airway, a combination of factors is generally considered together to predict if difficulty is likely to arise.

The patient pictured in Fig. 7 has no apparent craniofacial abnormality and appears to be of normal body habitus, but he has a large beard. He has a Mallampati grade of 2. Mouth opening appears to be adequate. Dentition is acceptable with slightly prominent central incisors. Neck extension is adequate with thyromental distance apparently greater than 6.5 cm. Jaw protrusion is difficult to assess here.

180

M. Turnbull et al.

Fig. 8 Airway adjuncts a and b

Answer:

Fig. 7 Findings on airway examination of a 35-year-old male booked for urgent laparoscopic appendicectomy. (Image taken on authors as models)

Laryngoscopy is not likely to be difficult, but bag-mask ventilation is likely to be challenging. 4. A 15-year-old child has been anesthetized for a laparoscopic inguinal hernia repair. On attempting to bag-mask ventilate the patient, the mask does not fog up and an end tidal CO2 trace is not visible. What maneuver might help improve the ability to mask-ventilate the patient? (1*) Answer: A head-tilt chin-lift or jaw thrust may help to move the base of the tongue away from the posterior wall of the oropharynx to allow passage of air into the airway. What is the equipment A and B pictured in Fig. 8? (2*)

These are airway adjuncts used to maintain patency of the airway of an obtunded or anesthetized patient. Image a shows oropharyngeal airways. These are sized by measuring from the angle of the jaw to the level of the central incisors or from the tragus of the ear to the corner of the mouth. They are inserted with the opening facing towards the roof of the mouth then turned inferiorly once over the tongue. Image b shows a nasopharyngeal airway. The size required is estimated based on the size of the patient’s nares. Sizes 6 or 7 should be suitable for smaller adults and 8 or 9 for larger adults. After insertion of an oropharyngeal airway bag-mask ventilation is possible and intubation is to be attempted. What airway management equipment is pictured in Fig. 9a and b? (3*) Answer: Image a shows a Mackintosh (left) and McCoy (right) laryngoscope for direct laryngoscopy. The McCoy blade uses a lever operated from the handle to tilt the epiglottis superiorly, to improve laryngoscopy view. Image b shows a Glidescope video laryngoscope which allows the user to indirectly view the glottis. A stylet may be required to direct the endotracheal tube anteriorly around the curve of the upper airway.

Anesthesia and Intensive Care

181

Fig. 9 a and b Equipment used for visualization of the vocal cords

a

c

b

d

a. b. c. d.

CL1—entire glottis seen at laryngoscopy CL2—only posterior glottis seen CL3—only epiglottis seen CL4—unable to see epiglottis

With reference to the DAS (Difficult Airway Society) guidelines for the management of unanticipated difficult intubation, how should further attempts at intubation be performed? (2*) Answer:

Fig. 10 Laryngoscopy views taken using a video laryngoscope representing Cormack-Lehane grades seen on direct laryngoscopy in a mannequin

5. A 65-year-old man is to undergo a discectomy for L5 disc herniation causing cauda equina symptoms. After positioning, preoxygenation and induction, your initial view at laryngoscopy is a Cormack-Lehane grade 4. What is the Cormack-Lehane grading system as seen in Fig. 10? (1*) Answer: Cormack-Lehane (CL) grading describes the view obtained on direct laryngoscopy.

The DAS guidelines on the management of unanticipated difficult intubation start with plan A and moves alphabetically through to plan D. In plan A, intubation via direct laryngoscopy, the anesthetist should initially optimize head and neck position. They should ensure adequate neuromuscular blockade, and consider external laryngeal manipulation or backwards, upwards, rightward pressure (BURP). Removal, or adjustment, of cricoid pressure may be required if already applied. A bougie may help to guide the endotracheal tube anteriorly into the larynx where some portion of the cords is visualized. If an initial attempt fails, use of different laryngoscopes including different size Macintosh, McCoy blade, or video laryngoscope may allow for successful intubation. Oxygenation should be maintained throughout attempts at airway management. What are the next steps if intubation is unsuccessful after further attempts? (3*)

182

M. Turnbull et al.

Answer: Within the DAS guideline, plan A allows for a maximum 3 attempts at direct laryngoscopy, plus 1 from a senior anesthetist, before moving to plan B, maintenance of oxygenation via a supraglottic airway device. The first task in plan B is to declare a failed intubation and seek help. Maintenance of oxygenation should be accomplished using a supraglottic airway device (SAD). It is advisable to start with a second-generation device (e.g., i-gel). If unable to oxygenate, the anesthetist should try different sizes or a different device. Again, a maximum of 3 attempts is permitted to prevent the exacerbation of airway edema with repeated manipulation. If able to oxygenate, one should consider whether to wake the patient, to continue the SAD or to intubate through the SAD. If not oxygenating with 3 attempts, the next step is to move to plan C. Communication retains its importance when moving to plan C. Declaring failed SAD ventilation allows the team to follow the anesthetist's thinking, and to plan the next steps. Help should be sought if not already enroute. One final attempt should be made to mask ventilate the patient with adjuncts. If ventilation is impossible, and the patient is not already paralyzed, a muscle relaxant, for example rocuronium, should be given. If able to ventilate, this should be continued until the patient can be woken up. If not ventilating, the only option is to move to plan D.

Fig. 11 eFONA kit. Size 10 scalpel, bougie and size 6.0 endotracheal tube

Describe how you would manage a ‘can’t intubate can’t oxygenate’ (CICO) scenario. (4*) Answer: A ‘can’t intubate can’t oxygenate’ scenario is covered in plan D of the DAS guideline for management of unanticipated difficult intubation. This is also referred to as emergency front of neck access (eFONA).The anesthetist should declare CICO. Again, this makes the situation clear to the team and indicates to the operating department practitioner (ODP) that they should get the eFONA kit. Help should have been summoned by this time, but should be urgently raised if not. The standard technique of scalpel cricothyroidotomy, or the scalpel, bougie, tube (Fig. 11) technique is recommended as it is quick and has a relatively high success rate compared to other emergency techniques. A laryngeal handshake (Fig. 12) is used to identify the cricothyroid membrane. If unable to palpate the cricothyroid membrane, the anesthetist would make a vertical incision and blunt dissect down to the larynx. A transverse stab is made through the cricothyroid membrane (Fig. 13a), and then the scalpel blade turned

Fig. 12 Laryngeal handshake. The thumb and middle finger stabilize the larynx while index finger palpates for the cricothyroid membrane

downwards and pushed laterally to create a triangular opening down to the airway (Fig. 13b). A bougie is next passed alongside the scalpel into the airway (Fig. 14a) before the blade is removed and a 6.0 internal diameter endotracheal tube (ETT) passed over the bougie (Fig. 14b). Ventilation is confirmed with the presence of end tidal CO2. 6. A 70-year-old male has been admitted for a TURP (trans-urethral resection of the prostate) under spinal

Anesthesia and Intensive Care

Fig. 13 a Horizontal stab through cricothyroid membrane. b Blade turned inferiorly and pushed laterally

Fig. 14 a Bougie passed into airway alongside scalpel blade b Blade removed and ETT passed into airway

183

184

M. Turnbull et al.

dural puncture headaches (PDPH). PDPH is caused by ongoing CSF leakage into the epidural space. Through what structures does the needle pass when doing a midline spinal injection? (3*) How does this differ for an epidural? (3*) What drugs are used in spinal anesthesia? (3*) Answer:

Fig. 15 Spinal needles

anesthesia. He reports a family history of malignant hyperthermia (MH) and is currently awaiting the results of a muscle biopsy. What are the contraindications to spinal anesthesia? (1*) Answer: Absolute contraindications to spinal anesthesia are: patient refusal, allergy to local anesthetic, and infection at site. Relative contraindications include: deranged coagulation, increased intra cerebral pressure (ICP), systemic infection or sepsis, aortic stenosis and other fixed cardiac output states, undiagnosed neurological disease or demyelinating central neurological disorders and previous back surgery.

A spinal needle will pass through skin, subcutaneous tissue, supraspinous ligament, interspinous ligament, ligamentum flavum, epidural space and fat, dura mater, subdural space, arachnoid mater into the subarachnoid space. When a midline approach is unsuccessful due to bony obstruction, a paramedian approach may be used. Here, the paraspinal muscles are traversed rather than supraspinous and interspinous ligament. When performing an epidural, a loss of resistance technique is used to place the needle tip within the epidural space, so that it does not pierce the dura mater. The loss of resistance technique will be explained in question 7. A combined spinal-epidural (CSE) technique can be performed to provide the dense, faster onset block of a spinal with the ongoing analgesia of an epidural catheter. In this technique either two separate injections are performed, or a spinal injection is performed through the epidural needle (needle-through-needle technique) before an epidural catheter is inserted. Spinal anesthesia is achieved using both isobaric and hyperbaric or ‘heavy’ local anesthetic agents. Hyperbaric agents are made denser than the surrounding CSF by the addition of glucose. This allows them to move with gravity so that the position of the drug within the spinal column, and therefore block level, can be adjusted by tilting the patient until the block is established. Prilocaine and bupivacaine (Fig. 16) are the most frequently used agents. Opiates, particularly diamorphine, morphine, and fentanyl, may also be included.

What types of needles are shown in Fig. 15? (1*). These are spinal needles used to sample cerebrospinal fluid (CSF) or to inject drugs into the CSF during anesthesia.

What drugs are implicated in malignant hyperthermia (MH)? (4*) Answer:

1. is a non-cutting, pencil point (Sprotte) needle. 2. is cutting (Quincke) needle. A pencil point needle requires an introducer to pierce the skin, and therefore may be more difficult to manipulate, but causes less tissue injury and reduces the incidence of post

MH is a life-threatening reaction to some anesthetic drugs in susceptible individuals. It can be triggered by any of the volatile inhalational anesthetic agents (Fig. 17), except nitrous oxide, and by suxamethonium. Triggering causes excitation–contraction coupling within skeletal muscles to

Anesthesia and Intensive Care

185

release of calcium from the sarcoplasmic reticulum after administration of a triggering agent (Fig. 17). How is MH managed in anesthesia? (5*) Answer: Prevention of MH is by avoidance of triggering agents. This can be achieved by using regional techniques and total intravenous anesthesia (TIVA). When using a TIVA technique, anesthesia is maintained by an intravenous infusion of propofol, rather than inhalational agents, usually combined with an infusion of a short acting opiate such as remifentanil. Where MH is triggered in an individual who is not known to be susceptible, treatment is by stopping the causative agent, administering dantrolene (Fig. 18), and providing organ support on intensive care.Dantrolene inhibits the uncontrolled release of calcium from the sarcoplasmic reticulum by binding the ryanodine receptor therefore controlling the metabolic derangement seen in MH.

Fig. 16 Hyperbaric (heavy) bupivacaine (left) and prilocaine (right) used in spinal anesthesia

become deregulated leading to increased muscle tone, increased heat generation, excessive oxygen consumption, increased CO2 production, and tachycardia. As the reaction progresses, metabolic and lactic acidosis develop. Muscle breakdown then begins, leading to rhabdomyolysis. Without treatment renal failure, hyperkalemia, cardiovascular collapse and disseminated intravascular coagulation develop, eventually leading to death. The most common defect leading to MH susceptibility is a mutation of the gene for the ryanodine receptor on chromosome 19 causing excessive

7. A 48-year-old previously fit and well woman has undergone her first general anesthetic. She required a laparotomy for a perforated duodenal ulcer under general anesthesia with a rapid sequence induction (RSI). She was induced with thiopentone and suxamethonium. After an uneventful surgical repair of the perforation the patient is not waking up. What is the purpose of a rapid sequence induction (RSI)? What are the components? (1*) Answer: RSI is intended to facilitate a secure airway in a rapid manner to minimize the risk of aspiration in patients at high risk of regurgitation of gastric content.

Fig. 17 Desflurane (left) and Sevoflurane (right) are often used to maintain anesthesia, but these vapors should be avoided in patients with known MH

186

M. Turnbull et al.

necessarily related to anesthesia can include stroke and seizure under anesthesia. Complications of anesthesia including hypoxia, hypotension and hypothermia can slow or prevent waking. Pharmacological causes are the most commonly encountered and include opiate excess, residual hypnotic drugs or inhalational agent, residual, or incompletely reversed muscle relaxation, and suxamethonium apnea. What causes suxamethonium apnea? (3*) Answer: Fig. 18 Dantrolene used in the treatment of MH. Each vial contains 20 mg which should be dissolved in 60 ml of water for injection. As the starting dose is 2.5 mg/kg, a 72 kg person would need at least 9 of these vials

In the traditionally described RSI, the patient is anesthetized on a tilting table with suction within reach. After pre oxygenation cricoid pressure is applied before induction with thiopentone and suxamethonium. The RSI can be modified in myriad ways. This can include the addition of an opiate, using other IV induction agents and replacing suxamethonium with rocuronium (Fig. 19). What is the differential diagnosis for a patient not waking from general anesthesia? (2*) Answer: Not waking from anesthesia can have several causes and sometimes more than one. Neurological causes not

Suxamethonium is metabolized by plasma cholinesterase. Where this is deficient or absent, the action of suxamethonium is prolonged from minutes to hours. Plasma cholinesterase activity can be reduced due to variations in the gene for plasma cholinesterase on chromosome 3. Acquired plasma cholinesterase deficiency can be caused by chronic liver disease, malignancy, pregnancy, severe malnutrition, burns, organophosphate poisoning and renal failure. Due to the risk of apnea, the standard syringe label for suxamethonium differs from that of other muscle relaxants (Fig. 20). How is suxamethonium apnea managed? (4*) Answer: Management in patients with known deficiency is by avoidance of suxamethonium and mivacurium, which is also partially reliant on plasma cholinesterase for its metabolism. Relatives of those with known genetic deficiency can also be screened. At first presentation, the treatment is mainly supportive with sedation and ventilator support maintained on ICU until return of neuromuscular function is confirmed by neuromuscular monitoring. 8. A 32-year-old woman in her first pregnancy has had labor induced at 37 weeks gestation due to gestational diabetes. Observations, full blood count and clotting screen taken 2 h earlier are normal. The obstetric team

Fig. 19 Drugs commonly used in modified RSI. Top to bottom: fentanyl, alfentanil, thiopentone, propofol, suxamethonium, rocuronium. Standardized colored labels indicate type of drug: blue—opiates, yellow—induction agents, red—muscle relaxants

Fig. 20 Suxamethonium is labeled with red and black (rather than the pure red of other muscle relaxants)

Anesthesia and Intensive Care

187

Answer: This is a Tuohy needle, loss of resistance syringe, epidural catheter (with feeding connector), and filter used for epidural analgesia. The loss of resistance technique involves applying forward pressure on the needle via a specialized low friction syringe (Fig. 22). When the needle tip is in ligaments, fluid cannot be injected and so the needle moves forward with pressure on the syringe. Once the needle tip reaches the epidural space, fluid can be injected and so the plunger is depressed rather than the needle moving, expanding the epidural space. A catheter can then be threaded into the epidural space to provide continuous analgesia. What drugs are generally used for epidural anesthesia? (3*) Answer: Drugs used are generally local anesthetic agents with adjuvants to enhance analgesia, reduce local anesthetic requirement, and minimize motor block. Bupivacaine levobupivacaine and ropivacaine are the local anesthetic agents used in labor with lidocaine being used to top-up for cesarean section. Fentanyl is the most often encountered adjuvant, with diamorphine and morphine administered for patients who undergo cesarean section. Clonidine is commonly used as an adjuvant outside of the obstetric setting. What are the potential complications of epidural anesthesia? (4*) Fig. 21 Entonox cylinder with demand valve

reports no concern for mother or baby. She would like to consider options for pain relief. How can analgesia be provided in labor for this patient? (1*) Answer: Non-pharmaceutical methods include breathing, relaxation, massage techniques, transcutaneous electrical nerve stimulation (TENS) (only in early labor) and warm water baths. Pharmaceutical methods are paracetamol, oral or IV, Entonox by inhalation (Fig. 21), pethidine IM, morphine or diamorphine IM or IV and epidural anesthesia. What equipment is pictured in Fig. 22? (2*)

Answer: Block failure, patchy block, missed segments or inadequate analgesia are commonly seen. Discomfort on insertion and localized bruising is to be expected. Headaches occur where the dura is accidentally punctured during epidural insertion. Local anesthetic toxicity can occur due to excessive dosing or increased systemic absorption of local anesthetic. Infection can lead to localized symptoms or, when more severe, epidural abscess and meningoencephalitis. Excessive bleeding can lead to epidural hematoma formation. Nerve damage can be caused by direct trauma from the Tuohy needle, hematoma, abscess or chemical arachnoiditis. Nerve damage can be temporary or permanent and can range from a single nerve to complete loss of function below the level of damage.

188

M. Turnbull et al.

Fig. 22 Epidural equipment

9. A 25-year-old fit and well woman in labor with her first pregnancy has had an epidural catheter inserted for analgesia. She is seen to have a seizure. The midwife suspects local anesthetic toxicity. What is the differential diagnosis of seizure in this patient? (1*) Answer: Seizures in pregnancy can be related to pre-existing conditions such as epilepsy. The development of eclampsia is accompanied by seizures. Local anesthetic toxicity is possible in any patient in whom an epidural has been sited. What are the features of local anesthetic systemic toxicity (LAST)? (2*)

Answer: Management of LAST starts by stopping the infusion of any more local anesthetic. The patient should then be assessed taking an ABCD approach providing organ support as required. In the event of cardiac arrest, cardiopulmonary resuscitation (CPR) should be started following the advanced life support algorithm. Specific management is by a bolus and then infusion of lipid emulsion (intralipid). Prolonged CPR may be required to allow an intralipid infusion to be completed. The mechanism of action of intralipid is incompletely understood. It may act as a ‘lipid sink’, pulling the lipophilic local anesthetic from tissues. It may also prevent local anesthetic agents from binding to sodium channels within the heart. Another theory is that intralipid reverses the inhibition of fatty acid metabolism within the myocardium caused by local anesthetic agents.

Answer:

Intensive Care Medicine

LAST is associated with excitatory and depressive symptoms primarily in the cardiovascular and central nervous systems. It can present with any combination of features or as cardiac arrest. Perioral tingling, tinnitus, dysphagia, dizziness and confusion often precede seizure or loss of consciousness. Cardiovascular symptoms can include tachycardia, hypertension, hypotension, arrhythmia, and cardiac arrest.

10. A 67-year-old man has presented to the emergency department with a five-day history of dyspnoea, dry cough, fever. He is breathless at rest. His arterial blood gas whilst receiving 10 L/min oxygen (O2)via a face mask (Fig. 23) is:

How do you manage local anesthetic toxicity? (3*)

pH PaO2 PaCO2 HCO3− Base Excess (BE)

7.32 7.7 5.6 20.2 2.5

[7.35–7.45] [10–14 kPa] [4.5–6.0 kPa] [22–28 mmol/L] [−2–2 mmol/L]

Anesthesia and Intensive Care

189

Fig. 23 A patient with supplemental oxygen via a face mask Full and informed patient consent has been obtained to reproduce the image in a commercial publication

Can you interpret this arterial blood gas? (1*)

Answer:

Answer:

Investigations to include/exclude the differential diagnoses and the management would need to occur simultaneously. Along with this, a decision needs to be made as to whether this patient can be cared for on a ward or would they require admission to intensive care. In the first instance his oxygen therapy should be increased to 15 L/min via a non-rebreather mask, and he should have a portable chest x-ray as soon as possible. The chest x-ray should provide clues as to the etiology. However, if there is still uncertainty, a computed tomography pulmonary angiogram (CTPA) could help with excluding/including a pulmonary embolism, and point-of-care lung ultrasound ± echocardiogram (echo) would provide further information to exclude/include cardiogenic pulmonary edema. This patient had a severe bilateral pneumonitis due to COVID-19. CTPA excluded any large pulmonary emboli and point-of-care lung ultrasound, and echocardiogram revealed some pulmonary edema in the absence of cardiac failure—typical for this disease.

Taking into consideration the additional oxygen that the patient is receiving, there is a severe type 1 respiratory failure. The marginally low pH (7.32) and negative base excess indicates a mild uncompensated metabolic acidosis. Give three differential diagnoses? (2*) Answer: The differential diagnoses for the blood gas findings should include pneumonia, pulmonary embolism, and cardiogenic pulmonary edema. Given the history, pneumonia would be most likely. What would be the appropriate investigations and management for this patient? (3*)

190

M. Turnbull et al.

His oxygen requirements were too severe to be managed on a ward safely and so he was treated with high-flow nasal oxygen therapy in intensive care. 11. A 60-year-old man is in the intensive care unit following an emergency bowel resection for gut ischemia. The nurse has asked for a review of the patient’s blood pressure which is 81/55 mmHg. What further information would you like to assess this patient? (1*) Answer: It would be important to know some additional information to assess the patient’s cardiovascular state. These would include—the BP trend, heart rate and rhythm, warmth of peripheries, capillary refill time, urine output and response to a straight-leg-raise. What information can be gained from point-of-care echocardiography (Fig. 24)? (2*) Answer: Point-of-care echocardiography (echo) is not the same as a formal echo and cannot give detailed measurements and indices. However, it can provide answers to some important questions. In this case assessing inferior vena cava (IVC) collapsibility would help determine if the patient’s hypotension should be treated with fluid resuscitation or vasopressors. What is the contractile function of the left and right ventricles? Is there any dilation of any chambers? Is there any regional wall abnormality? Is there any evidence of right heart strain? Is there any evidence of cardiac failure? What is the volume status? What would be the appropriate management to stabilize this patient? (3*). Answer: This patient was hypotensive over the past two hours with diminishing urine output. He was diaphoretic with a below normal temperature. The monitor showed a sinus tachycardia. A straight-leg-raise showed a greater than 10% increase in mean arterial pressure. A point-of-care echo showed good cardiac function. All of this pointed towards septic shock, and he benefited from fluid resuscitation in the first instance. This patient should be managed taking an airway, breathing, circulation, disability, exposure (ABCDE)

Fig. 24 Subcostal four chamber view on point-of-care echocardiogram demonstrating normal heart chambers. Image acquired by OPENi— open access image bank, original cropped (CC BY 2.0)

approach with stabilization happening simultaneously to assessment. The patient’s ability to maintain their airway should be assessed and an endotracheal (ET) tube placed if indicated. Their gas exchange and oxygenation should be assessed via serial blood gasses with supplementary oxygen and/or respiratory support provided if indicated. If the patient remains hypotensive despite adequate filling, vasopressor support will likely be required. The Glasgow coma score (GCS) and urine output should be monitored. Sepsis should be treated initially with broad spectrum antibiotics until blood cultures reveal antibiotic sensitivities of the causative organism. Further imaging of his abdomen revealed an anastomotic leak which required a return to the operating theater. 12. A 50-year-old woman has been admitted to intensive care with severe respiratory failure requiring invasive mechanical ventilation, vasopressors, and this machine (red arrow) shown in Fig. 25. What is this machine? (1*) What are the indications for its use? (2*) What are the possible complications from its use? (3*)

Anesthesia and Intensive Care

191

Fig. 25 A machine on the intensive care unit

Answer: This is a hemofiltration machine. It is one of the most frequently used forms of renal replacement therapy (RRT) in intensive care. The main indications for using RRT in intensive care are refractory hyperkalemia, pulmonary edema, and severe metabolic acidosis. Other indications may include uremia, sepsis, rhabdomyolysis, and toxins. The complications of RRT can be located to the line/extracorporeal circuit needed to run the machine. This includes vessel bleeding, infection, air embolism. RRT itself can cause hypotension due to fluid removal, deranged electrolytes, and adverse effects of anticoagulation. 13. A 41-year-old woman is admitted with a low GCS (Glasgow Coma Scale). Upon arrival, she has her airway maintained with an oropharyngeal airway adjunct in place. On painful stimulus, her eyes do not open, she localizes to this stimulus and is only making incomprehensible sounds. What is her score on the Glasgow Coma Scale? (1*)

E1V2M5 gives a GCS score of 8. (Cross reference Table 12.1 in Neurosurgery section of Chap. Neurology and Neurosurgery). This patient required immediate CT imaging of her brain to determine the cause of her low level of consciousness. How should the airway be managed in the CT scanner? (2*) While her airway is maintained with an oropharyngeal airway adjunct, this would not be safe inside a CT scanner with limited access. The safest approach would be to perform a rapid sequence induction with a trained anesthetic assistant and place an endotracheal tube which would then be connected to a portable ventilator to facilitate the CT scan. What does the CT scan in Fig. 26 show and which neuroprotective measures are important to use? (3*) Answer: The CT images show a spontaneous subarachnoid hemorrhage with intraventricular extension. The patient would require immediate discussion with a neurosurgical team for possible surgical intervention. In any case, neuroprotective

192

M. Turnbull et al.

Fig. 26 Axial views of a patient’s computed tomography (CT) of the head. Image acquired by OPENi—open access image bank (CC BY-NC-SA 3.0)

measures should be used to prevent secondary brain injury. Once the airway has been secured, adequate sedation and analgesia should be maintained. The aim should be for normocapnia, normothermia, and avoidance of hypoxemia, hypotension and hypovolemia. It is also important to avoid neck ties for the endotracheal tube and to maintain 30-degree head elevation and minimize positive end expiratory pressure (PEEP). 14. A 65-year-old woman with a history of chronic obstructive pulmonary disease (COPD) attends Accident and Emergency Department (A&E) with an infective exacerbation of COPD. Blood gases show: pH pO2pCO2 Bicarbonate Base excess (BE) Sat O2

7.30 6.5 7.48 26 1.9 88%

[7.35–7.45] [10–14 kPa] (on room air) [4.5–6.0 kPa] [22–28 mmol/L] [−2–2 mmol/L] [94–98%]

What do the blood gas results indicate? (1*) Answer: It shows respiratory acidosis with Type 2 respiratory failure but no metabolic compensation. This indicates a new respiratory problem. The raised carbon dioxide (pCO2) indicates poor ventilation and low oxygen (pO2) indicates hypoxia. How can this respiratory acidosis be managed? (2*) Answer: Respiratory acidosis can be managed by bi-level non-invasive ventilation (BiPAP) as shown in Fig. 27, where a tight-fitting mask is applied over the patient’s face and 2 levels of pressure is delivered to expand the lung and help with CO2 clearance. This helps to reduce respiratory acidosis in such patients.

Anesthesia and Intensive Care

193

Fig. 27 Bi-level Positive Airway Pressure (BIPAP) Machine

The pressure support (Ps) and peak end expiratory pressure (PEEP) can be adjusted to support lung expansion and oxygenation respectively. These settings may also appear as inspiratory Pressure (IPAP) which is equivalent to Ps plus PEEP, and Expiratory Pressure (EPAP) which is equivalent to PEEP. How should this patient’s oxygenation be managed once they no longer require non-invasive ventilation? (3*)

A Venturi-mask (Fig. 28) delivers fixed oxygen concentration varying from 24 to 60%. There are different color nozzles delivering respective oxygen concentration. Oxygen is administered at a set flow rate for each adaptor. A specific amount of air is then entrained into the device, via the venturi effect, to dilute the administered oxygen to the desired concentration. What are the additional measures required while managing such patients? (4*)

Answer: Answer: Administer oxygen using a Venturi-mask which provides fixed concentration of oxygen titrated according to oxygen saturation (SpO2) monitoring.

These patients also require nebulizers (salbutamol, ipratropium), antibiotics, corticosteroids, and physiotherapy.

194

M. Turnbull et al.

Further Reading 1. Chambers D. Basic physiology for anesthetists. 2nd ed. Cambridge, UK: Cambridge University Press; 2019. 2. West JB. West’s respiratory physiology: the essentials. 11th ed. Philadelphia, USA: Lippincott Williams & Wilkins; 2020. 3. Peck TE. Pharmacology for Anesthesia intensive care. 5th ed. Cambridge, UK: Cambridge University Press; 2021. 4. Bersten AD. Oh’s intensive care manual. 8th ed. Amsterdam, Netherlands: Elsevier; 2018.

Fig. 28 Venturi-mask

Cardiothoracic Surgery Mahmoud Loubani, Konstantinos Kotidis, and Mubarak A. Chaudhry

Learning Outcomes

Answer: Non ST elevation myocardial infarction (NSTEMI)

• The student should be able to diagnose common cardiothoracic diseases which require surgery. • Gain knowledge on how to inform patients of the benefits and risks of common surgical procedures such as coronary artery bypass grafting (CABG). • The student should be able to name common causes of pleural effusion and inform patients about further investigations to plan the best treatment and gain an understanding of the presentation and investigation of lung cancer.

Based on the ECG abnormalities, what might the coronary angiogram show? (4*)

Cardiac Surgery (1) An 80 year old man who is normally fit and well presents to A&E with a 4 week history of central chest pain radiating to the neck on minimal exertion, culminating in a severe episode lasting 1 h, associated with nausea and sweating. What significant abnormalities are shown in the ECG Fig. 1? (3*). Answer: ST depression in the anterior and lateral leads (V2-V6, I, aVL) as well as the inferior leads. ST elevation in lead aVR. Troponin T (TnT) was significantly raised. What is the diagnosis? (3*)

Answer: Coronary angiogram might show significant disease in all 3 coronary arteries OR left main stem as well as disease in the right coronary artery. This patient’s Coronary angiogram shows significant left main stem disease affecting the bifurcation into the left anterior descending (LAD) and circumflex arteries as well as disease in all 3 coronary arteries. Advanced ECG interpretation ST elevation in lead aVR is a subtle sign that the patient may have left main stem disease. However, it should be noted that ST elevation in this lead alone does NOT constitute criterion for primary PCI or thrombolysis (needs at least 1 mm in 2 contiguous limb leads, or 2 mm in 2 contiguous chest leads). He was treated with antiplatelet therapy, bisoprolol (a beta-blocker), low molecular weight heparin, and intravenous nitrate. What is the treatment of choice if he does not respond to optimal medical therapy? (3*)

M. Loubani (&) . K. Kotidis . M. A. Chaudhry Department of Cardiothoracic Surgery, Castle Hill Hospital, Hull University Teaching Hospitals NHS Trust, Cottingham, HU16 5JQ, UK e-mail: [email protected] K. Kotidis e-mail: [email protected] M. A. Chaudhry e-mail: [email protected] © The Author(s), under exclusive license to Springer Nature Switzerland AG 2023 K. Wong et al. (eds.), Practical Guide to Visualizing Medicine, https://doi.org/10.1007/978-3-031-24465-0_10

195

196

M. Loubani et al.

Fig. 1 ECG of a patient with chest painPermission granted by Hull York Medical School (HYMS)

Answer: urgent CABG CABG is preferable to percutaneous coronary intervention for the treatment of left main stem disease affecting the bifurcation. How should the patient be advised regarding benefits and risks before the operation? Answer: 1. Surgery is indicated on prognostic and symptomatic grounds 2. The operation is carried out under general anaesthesia through a median sternotomy and bypasses are constructed using a combination of mammary artery and saphenous vein 3. Possible complications include death, stroke, bleeding, chest and wound infections and atrial fibrillation 4. Risk of death 6% as estimated by logistic Euroscore 4 years later, this man’s twin brother who is 84 years old, and had a CABG 10 years ago with vein grafts only for stable angina, presents with recurrence of stable angina 10 years after CABG. This is a 3D reconstruction of a CT coronary angiogram shown in Fig. 2. Identify the stenoses in the saphenous vein bypass grafts (SVG) to the obtuse marginal branch of the circumflex artery

Fig. 2 CT coronary angiogram (CTCA) of a patient with stable angina and saphenous vein grafts (SVG). The figure is shown with permission by Dr Paul Jenkins, European Scanning Centre, London. Permission granted by Hull York Medical School (HYMS)

(OM1), diagonal branch of the left anterior descending (LAD) artery or the SVG to the LAD itself. (1*) Answer: Stenoses were seen in the SVG to Obtuse Marginal branch of the circumflex artery (OM1) and diagonal branch of the LAD artery. No stenosis was seen in the SVG to the LAD itself.

Cardiothoracic Surgery

Image interpretation top tips—Let us examine Fig. 2 together to see how this CT image helps us understand anatomy, pathology, clinical presentation (stable angina), treatment (CABG), and natural history/development of stenoses of the bypass grafts. Can you see 3 arteries [in red] arising from the ascending aorta [coloured white], near the top left of the image? The top artery was the Saphenous vein graft to the obtuse marginal branch of the circumflex artery that supplies blood to the lateral wall of the left ventricle. Can you see a narrowing about half-way across the image? (near the intersection with the LAD artery coursing down the front of the heart). The 2nd artery from the top is the vein graft to the diagonal branch of the LAD artery which also supplies the lateral wall of the left ventricle. This vein graft has a tight narrowing near the origin of the pulmonary artery (coloured brown). The 3rd artery (bottom of the 3 vein grafts originating from the ascending aorta) is connected to the LAD artery itself (coloured white, coursing down the front of the heart near the middle of the image. There is no stenosis in the LAD vein graft, which bypassed the tight narrowing of the LAD artery (seen clearly near the centre of the image). Visualising the Anatomy also helps us understand where to place our stethoscope to listen to the aortic and pulmonary areas.

Fig. 3 ECG of a patient with a systolic murmur

197

Can you see how Fig. 2 helps you understand why the aortic area (coloured white) is in the RIGHT 2nd intercostal space whilst the pulmonary area is in the LEFT 2nd intercostal space (coloured brown)? The sternum lies over the right ventricle which pumps blood into the pulmonary artery. That is why you palpate for a right ventricular heave over the lower left sternal edge. Can you see the apex of the left ventricle? Where can you usually palpate the apex if the left ventricle is not dilated? (1*) Answer: 5th intercostal space, midclavicular line What is the treatment of choice if he does not respond to optimal anti-anginal therapy? (3*) Answer: percutaneous coronary intervention (PCI) to improve symptoms Note re-do CABG does not have prognostic benefit. (2) A 62 year old woman who is normally fit and well presents as an emergency with chest pain, dyspnea on minimal exertion and frequent syncope. On examination, she has an ejection systolic murmur loudest in the aortic area, radiating to the neck. What does the electrocardiogram (ECG) in Fig. 3 show? (3*)

198

Answer: ECG shows left ventricular hypertrophy (LVH). Serial high sensitivity Troponin T was normal. What is the likely diagnosis? (3*) Answer: aortic stenosis What is the treatment of choice? (3*) Answer: Aortic valve replacement

M. Loubani et al.

2. Symptoms of aortic stenosis are shortness of breath, angina and syncope 3. The operation is carried out under general anaesthesia through a median sternotomy and a metallic valve is used for replacement in this 62 year old person (Fig. 4). 4. Possible complications include death, stroke, bleeding, chest and wound infections and atrial fibrillation. 5. Risk of death 8% as estimated by logistic Euroscore Which valve is replaced in the patient whose CT is shown in Fig. 4? Is it metallic or bioprosthetic? (4*)

How should this patient be advised regarding benefits and risks before the operation? Answer: 1. Surgery is indicated on symptomatic and prognostic grounds.

Fig. 4 320-slice CT image of the heart. The figure is shown with permission by Dr Paul Jenkins, European Scanning Centre, London. Permission granted by Hull York Medical School (HYMS)

Cardiothoracic Surgery

199

she was in primary school and she stayed at home for nearly 2 months. On examination she has a malar flush, an irregular heart rate and a low pitched mid- and late-diastolic murmur, best heard at the cardiac apex with the patient on her left side.

Answer: Metallic aortic valve (dense structure between the left ventricle and aorta). What is the main advantage and disadvantage of metallic versus bioprosthetic aortic valve implant? (4*) Answer: Metallic valves tend to be more durable, but require long-term warfarin. On the other hand, bioprosthetic valves are preferable in patients aged 65 or over. In patients older than 65 undergoing AVR with a porcine bioprosthesis, the rate of deterioration is 10%) unintentional weight loss. Clinical examination is normal. Describe the findings in his CXR in Fig. 18. (2*)

Fig. 17 Chest X-ray of a patient presenting with acute shortness of breath

Answer: there is enlargement of the right hilum, consistent with a mass, giving rise to an extensive opacity in the right upper zone. This is in keeping with collapse of the right upper lobe (RUL), secondary to obstruction caused by the centrally located mass. The right middle and lower lobes appear to be hyperexpanded with increased aeration, and the right hemidiaphragm is elevated. These findings are consistent with volume loss or (less likely) involvement of the right phrenic nerve by the tumour. Other findings (5*): Golden’s S sign: the hilar mass along with the collapse of the RUL, though not specifically S shaped, represent the components of the latter. The juxtaphrenic peak sign, also known as diaphragmatic tenting or Kattan sign, refers to the peaked or tented appearance of a hemidiaphragm which can occur in the setting of lobar collapse or post lobectomy (lung). It is caused by retraction of the lower end of diaphragm at an inferior accessory fissure (most common), major fissure or

208

M. Loubani et al.

inferior pulmonary ligament. It is commonly seen in collapse of the left or right upper lobes but may also be seen in middle lobe collapse. First described by radiologist Dr. R. Golden in 1925, it is named such, because it resembles a reverse S shape and is therefore sometimes referred to as the reverse S-sign of Golden. It can be seen on both PA chest radiographs and on CT scans. Typically seen with RUL collapse caused by a central mass obstructing the upper lobe bronchus, the S-sign can also be seen with the collapse of other lobes. The commonest malignancy associated with it is primary bronchogenic carcinoma, however, other centrally located masses (mediastinal tumours, metastases, enlarged lymph nodes) may manifest themselves in a similar fashion. Symptoms/signs associated with the above condition may include (select all that apply) (3*) 1. 2. 3. 4. 5. 6. 7. 8.

Cough Hemoptysis Voice hoarseness Palpitations Chest pain Headache/Dizziness Heartburn Hyperhidrosis

imply cardiac or esophageal involvement, and 8 is also unlikely as this is not a Pancoast tumour. What are the further management options? (3*) Answer: 1. 2. 3. 4. 5. 6.

CT PET-CT CT-guided biopsy Bronchoscopy/EBUS Mediastinoscopy VATS biopsy

The above modalities are suitable to establish histological diagnosis and accurately stage the tumour, in order for the lung cancer MDT to recommend the appropriate treatment. The tumour was found to be a small cell lung cancer and the patient was referred for non-surgical oncological treatment. (b) A 70 years old man presented with hemoptysis. He has smoked for 50 years. Question: What does the CT scan (Fig. 19a, b) show? (2*) Answer: Spiculated mass in the right lung Question: Which lobe? (1*)

Answer: 1, 2, 3, 5, 6 represent manifestations of lung cancer involving the lung or surrounding tissues, with 6 indicating potentially metastatic disease. 4. and 7 are unlikely, as they

Answer: Right lower lobe

Fig. 19 a CT images of a patient presenting with hemoptysis. b CT images of a patient presenting with hemoptysis

Cardiothoracic Surgery

209

MCQs 1. A 50 year old man presents with a 4 week history of central chest pain radiating to the neck. The pain would occur after he walks 200 yards, and is relieved by resting within 5 min. It is not worse on inspiration, and is no better leaning forward. There is no acid taste or other associated features. There is no significant past history. He has a 30 pack-year smoking history. Examination is normal. What is the most likely diagnosis? Fig. 20 PET scan image of patient with hemoptysis

What is the next investigation and why? (3*) Answer: PET Scan to demonstrate metabolic activity in the mass by uptake of FDG as seen in Fig. 20

A. B. C. D. E.

Aortic stenosis Gastro-oesophageal reflux disease Musculoskeletal chest pain Pericarditis Stable angina

Answer: E Question: What other information should be obtained from the PET Scan? (4*) Answer: No other intrathoracic PET positive masses, no mediastinal lymph node involvement and no distant metastases in the liver or adrenal glands. Question: How can a tissue diagnosis be made? (5*) Answer: (1) CT guided biopsy (2) Bronchoscopy and biopsy (3) Navigational Bronchoscopy and Biopsy Question: What other information is required before deciding a plan of action? (5*) Answer: (1) Pulmonary function tests with particular attention to FEV1, FVC, FEV1/FVC ratio and transfer factor TLCO and KCO (2) Performance status: should be 0-1. 2 would be doubtful and anything above will not be suitable for surgery. Question: What is the percentage of patients suitable for surgical resection at presentation? Answer: 20%. Lung cancer normally presents late when curative resection is not possible. Surgery in early stage may have 60–70% 5 year survival.

Explanation- This patient has typical stable angina (See chapter “Top Tips for Image Interpretation” for nuclear imaging figure 25 of chapter noting the correlation of ischemia and the clinical symptom of angina. Also review Chap. Cardiology and Vascular Diseases). Pericarditis pain may follow an MI or CABG, but tends to be worse on inspiration and better leaning forward. Musculoskeletal chest pain is often worse on movement and may get worse when the patient breathes in. Important pitfall to remember: Reproducibility on palpation is not a very specific sign. Gastro-oesophageal reflux disease should be suspected if the patient’s discomfort is worse bending down, if there is associated acid taste or past history of hiatus hernia. Always examine the abdomen as well as the cardiorespiratory system when the patient presents with chest pain. A useful clue might be found on palpating the epigastric area and the patient reports tenderness or even his typical presenting symptom. Aortic stenosis is typically associated with a small volume, slow rising pulse, and an ejection systolic murmur, but this patient’s clinical examination was normal. 2. A 55 year old woman with no previous medical history is admitted with an anterior non- ST elevation myocardial infarction. Coronary angiogram shows tight distal left main stem disease extending to the proximal left anterior descending and circumflex arteries. She has not

210

M. Loubani et al.

responded to optimum medical therapy and has ongoing chest pain with dynamic ST depression. Her blood pressure is 90/60. What is the treatment of choice? A. Thrombolysis B. Percutaneous coronary intervention C. Coronary artery bypass surgery D. Morphine and palliative care E. Increase intravenous nitrate

discussed the typical findings of aortic valve replacement in the text above. 4. A 60 year old man with a metallic aortic valve replacement presents with dyspnea. On examination he has a loud early diastolic murmur, heard best in the aortic area and lower left sternal edge, and bibasal lung crackles. What is the most likely diagnosis?

Answer: C Explanation: As the patient has not responded to optimal medical therapy, and her BP is low, increasing iv nitrate will not be advisable. She is very young and had no previous history, so palliative care would also be inappropriate at this stage. Thrombolysis is not indicated for NSTEMI. CABG is preferable to percutaneous coronary intervention for the treatment of left main stem disease affecting the bifurcation. 3. A 60 year old man presents with dyspnea. He had a valve replacement 20 years previously, but he cannot remember which valve was replaced. On cardiac auscultation, a metallic click from a prosthetic valve is loudest in the apex, coinciding with the 1st heart sound, What metal valve replacement did the patient have? (A) (B) (C) (D) (E)

Aortic Mitral Tricuspid Pulmonary None of the above

(A) (B) (C) (D) (E)

Aortic stenosis with left ventricular failure Aortic regurgitation with right ventricular failure Aortic stenosis with right ventricular failure Aortic regurgitation with left ventricular failure Mitral stenosis with left ventricular failure

Answer: D Explanations—An early diastolic murmur loudest in the aortic area and lower left sternal edge is typical of aortic regurgitation (AR). Bibasal (dependent) lung crackles are typically found in left ventricular failure. Right heart failure tends to cause edema, with a raised JVP. It is worth noting the shorter the “early diastolic murmur”, the more severe the AR. In the online E-book, there are additional useful resources to support learning e.g. echocardiography. This will not only help better understanding of cardiac anatomy and pathology, but also help “visualization of heart murmurs” e.g. why AR murmurs are diastolic, and why the AR murmur might be heard loudest in the lower left sternal edge. (Video 7.2).

Answer: B

Recommended Further Reading

Explanation: The metallic click is loudest in the mitral area, suggesting that the patient had a mitral valve replacement. If the click is loudest in the tricuspid area, the patient might have had a tricuspid valve replacement. We have already

1. Oxorn DC, Otto CM. Atlas of intraoperative transesophageal echocardiography: surgical and radiologic correlations, Text with DVD. 1st ed. Philadelphia: Saunders; 2007.

Gastrointestinal Surgery, Gastroenterology and Hepatology Richard Young, Wingzou Wong, and Jessica Shearer

Learning Outcomes

Pre-study Questions

. Assess the likelihood of acute appendicitis in a patient with an acute abdomen . Recognise the signs of bowel obstruction and understand the principles of surgical management . Understand the range of surgical diseases of the gallbladder . Appreciate important benign causes of esophageal symptoms . Outline methods of endoscopic visualisation of the gastrointestinal tract . Describe the pathology and appearance of common causes of inflammation of the gastrointestinal tract . Outline the major causes and main approaches to the investigation and management of upper gastrointestinal bleeding . Understand the principles of managing portal hypertension

MCQS 1. Acute abdominal pain A 21 year old man presents with a 48 h history of right iliac fossa abdominal pain. Appendicitis is suspected. To assist in making a clinical decision the Appendicitis Inflammatory Response (AIR) Score is used. The following parameter is part of the AIR Score: (a) (b) (c) (d) (e)

Sex CRP level mg/L Procalcitonin level ng/ml Temperature >40 °C Diarrhoea

2. Bowel obstruction A 50 year old woman presents with crampy abdominal pain, nausea and vomiting. On clinical examination she appears dehydrated. The abdomen is distended. She undergoes the investigation seen in Fig. 1.

R. Young (&) . W. Wong Department of Gastrointestinal Surgery, York and Scarborough Teaching Hospitals, NHS Foundation Trust & Hull York Medical School (HYMS), York, UK e-mail: [email protected] W. Wong e-mail: [email protected] J. Shearer Department of Gastroenterology and Hepatology, York and Scarborough Teaching Hospitals NHS Foundation Trust, York, UK e-mail: [email protected] © The Author(s), under exclusive license to Springer Nature Switzerland AG 2023 K. Wong et al. (eds.), Practical Guide to Visualizing Medicine, https://doi.org/10.1007/978-3-031-24465-0_11

211

212

R. Young et al.

Which of the following parameters is measured as part of the Childs-Pugh Score: (a) (b) (c) (d)

Alanine transaminase ALT level (U/L) Alkaline phosphatase ALP level (U/L) Ascites Activated Partial Thromboplastin Time (seconds) (e) Haemoglobin (g/dL)

APTT

5. Gastrointestinal bleeding A 72 year old woman presented with a 1 day history of passing black tarry stools. She also reports dizziness and feeling faint. You suspect she is having an upper gastrointestinal bleed. She is assessed using the Glasgow Blatchford Score (GBS) What is the purpose of this score?

Fig. 1 SBO Pre study Q RY

What is the most likely cause? (a) (b) (c) (d) (e)

Malignancy Volvulus Adhesions Gallstone ileus Hernia

3. Gallstone disease The overall prevalence of gallstones in the adult population is: (a) (b) (c) (d) (e)

1–2% 5% 10–15% 30% >50%

4. Jaundice A 54 year old man presents with a 3-week history of jaundice, lethargy and abdominal swelling. On examination there are spider naevi and palmar erythema. The abdomen is distended with dullness in the flanks, cirrhosis is suspected.

(a) To identify which patients should receive blood products (b) To identify which patients are at low risk of requiring intervention and can be safely discharged (c) To identify which patients will require medical therapy with a proton pump inhibitor (d) To differentiate between variceal and non-variceal bleeding (e) To identify patients at risk of adverse outcomes following gastrointestinal bleeding Answers 1) b; 2) c; 3) c; 4) c; 5) c. Cases 1. 2. 3. 4. 5. 6. 7.

GI Surgery: Acute Abdomen GI Surgery: Bowel Obstruction GI Surgery: Gallstones GI Surgery: Dysphagia Gastroenterology: Inflammatory Bowel Disease (IBD) Gastroenterology: Gastrointestinal (GI) Bleed Hepatology: Portal Hypertension

1. A 50 year old man presents with a 2-day history of abdominal pain. He reports initial periumbilical crampy pain, which then localised after 24 h to the right iliac fossa (RIF) and became constant. He has vomited and has a temperature. On examination he is in pain, slightly flushed but with no significant tenderness on examination and palpation in his RIF. He undergoes a CT scan of his abdomen and pelvis with IV contrast. An image from which is included in Fig. 2.

Gastrointestinal Surgery, Gastroenterology and Hepatology

213

Answer:

Fig. 2 Abdominal CT scan case 1. A 50 year old gentlemen with acute abdominal pain

Describe what the CT demonstrates in Fig. 2? (1*) Why does the patient have no significant RIF tenderness on examination? (1*) What is the clinical test shown in Fig. 3 and why could it be useful in this case? (3*)

This is an axial slice from an abdominal CT scan with IV contrast that demonstrates fat-stranding around a tubular structure behind the caecum in the RIF. The structure has enhancing walls and there is reactive thickening of the surrounding fat and fascia. This is in keeping with acute appendicitis. On this single slice there is no evidence of any fecalith, perforation or abscess; important considerations when planning treatment approaches. This is a retrocecal appendix and the highly variable position of the appendix in relation to nearby structures affects clinical presentation. Often patients with retrocecal appendicitis present slightly later than typical and may have fewer abdominal findings on abdominal examination since the inflamed appendix lies behind the caecum overlying the retroperitoneum. Abdominal CT Scan Interpretation Top Tips . Look for asymmetrical features . Anatomical principles will help you localise pathology . Oedematous, inflamed fat enhances = stranding on the CT . School of Surgery “How to Interpret Abdominal CT Scans” https://www.youtube.com/watch?v=Eg3GeUmOeYE Figure 3 this is a useful clinical examination technique known as the psoas stretch test. It can be used to elicit pain on testing in patients with retrocecal appendicitis. Passive extension of the right hip by the examiner with the patient in the left lateral position stretches the psoas tendon that is inflamed secondary to the overlying appendix. Top Tip

Fig. 3 Clinical examination technique

What clinical decision making tools are available and validated for use in this clinical scenario? (4*)

Use your left hand to stabilise the hip at the anterior superior iliac spine (ASIS) whilst your right hand carefully extends the hip Clinical decision making tools exist to support diagnostic assessments for patients with RIF pain and suspected appendicitis. The Appendicitis Inflammatory Response score (AIR) is one of the newer tests and has been externally validated [1, 2]. It is included for use in the latest clinical guidelines on the management of acute appendicitis [3]. It stratifies patients into low, moderate or high risk. It can be used to support discharge of low risk patients and avoid unnecessary imaging and investigations in this group (Table 1).

214 Table 1 Andersson et al. Appendicitis Inflammatory Response (AIR) Score. Clinical findings and results are scored as per the criteria and summed to give the overall score. This is then used to help stratify the patients into low/moderate/high risk for acute appendicitis as per the second part of the table. This risk score is used to aid decision making regarding imaging, observation, treatment and safe discharge(2)

R. Young et al. Clinical characteristic

Score

Vomiting

+1

RIF pain

+1

Rebound tenderness Light

+1

Medium

+2

Strong

+3

Temperature >38.5 °C

+1

Polymorphonuclear leukocytes 70–84%

+1

> 85%

+2

WBC 10,000–14,999 cells/L

+1

> 15,000 cells/L

+2

CRP 10–49 mg/L

+1

> 50 mg/L

+2

Total air score

Interpretation

0–4

Low risk 3 seconds. In this case, a joint decision between the cardiology specialist and patient with the help of his next of kin (and an interpreter) was important to weigh risks and benefits of the pacemaker in the context of his functional ability and his personal preference. He had capacity to decide and refused a pacemaker insertion. ICD has a different function from a pacemaker as it treats arrhythmias like ventricular tachycardia or fibrillation which is not present in this patient. Beta-blocker and ICD also help patients with severe left ventricular systolic dysfunction (LVSD). However, there is no suggestion of symptoms or signs of LVSD in this case. Beta-blocker will make his bradycardia and sinus pause worse.

271

References 1. Dix MR, Hallpike CS. The pathology symptomatology and diagnosis of certain common disorders of the vestibular system. Proc R Soc Med. 1952;45(6):341–54. 2. Wade DT, Collin C. The barthel ADL index: a standard measure of physical disability?. Int Disabil Stud. 1988; 10(2):64−7. 3. McMichael KA, VanderBilt J, Lavery L, Rodriguez E, Ganguli M. Simple balance and mobility tests can assess falls risk when cognition is impaired. Geriatr Nurs. 2008;29(5):311–23.

Further Reading 1. Turner G, Clegg A, Youde J. Fit for frailty. Consensus best practice guidance for the care of older people living with frailty in community and outpatient settings. London: British Geriatrics Society; 2014. http://www.bgs.org.uk/campaigns/fff/fff_full.pdf Google Scholar. 2. Wass S. The problem of delirium in the elderly. In: Nair B, editors. Geriatric Medicine. Singapore: Springer; 2018. https://doi.org/ 10.1007/978-981-10-3253-0_5. 3. Flynn B. Dementia: making a diagnosis and managing behavioural and psychological symptoms. In: Nair B, editors. Geriatric Medicine. Singapore: Springer; 2018. https://doi.org/10.1007/978981-10-3253-0_6. 4. Tinetti ME, Speechley M, Ginter SF. Risk factors for falls XE “Falls” among elderly persons living in the community. N Engl J. 1988;319:1701–7.

Renal and Transplant Medicine and Urology Sunil Bhandari and Sigurd Kraus

Learning Outcomes: Demonstrate Basic Knowledge-Anatomy; Physiology; Biochemistry as follows: . Structure and function of the kidneys . Histological structure of kidney and component parts of nephron . Structure of glomerular and processes of glomerular ultrafiltration . Acid base balance . Sodium and potassium Balance.

Managing clinical and biochemical problems with knowledge of drugs used: . Managing hyperkalaemia . Describe the pharmacology of common classes of diuretics . Describe the microbiology of urinary infections. Extend your depth of knowledge as in example below: Vasculitis–small vessel disease and larger vessel SLE. Acquire specialist knowledge in the following:

Integrate key basic Science−Pathophysiology and Key Investigations into clinical scenarios: . Describe features and consequences of Acute kidney injury (AKI) . Describe features and consequences of chronic kidney disease (CKD).

. . . . .

Rare disorders Drugs and the kidney Transplantation Stones Dialysis.

Pre-study questions Demonstrate Diagnostic Skills in the following: EMQ: . Common causes of Acute kidney injury (AKI) and chronic kidney disease (CKD) . The use of imaging . Electrolyte disorders.

S. Bhandari (&) Department of Renal Medicine, Hull University Teaching Hospitals NHS Trust, Hull, HU32JZ, England e-mail: [email protected] S. Kraus Department of Urology, Hull University Teaching Hospitals NHS Trust, Hull, HU32JZ, England e-mail: [email protected]

Theme: Proteinuria Options: A. B. C. D. E. F. G. H.

Minimal change disease Hypertension Focal segmental glomerulosclerosis Cellulitis Membranous nephropathy IgA nephropathy Amyloidosis Orthostatic proteinuria.

© The Author(s), under exclusive license to Springer Nature Switzerland AG 2023 K. Wong et al. (eds.), Practical Guide to Visualizing Medicine, https://doi.org/10.1007/978-3-031-24465-0_14

273

274

Fig. 1 Kidney biopsy, H&E staining showing a glomerulus and surrounding tubules. Permission granted by Hull York Medical School (HYMS)

1. A 16-year-old female presents with increasing swelling of both ankles for 2 weeks. Prior to this she had an upper respiratory tract viral illness. On examination she has bilateral ankle oedema. BP 100/70 mmHg, Temp. 36 °C. Urine dipstick protein 4+, blood negative. Urine protein: creatinine ratio is 400 mg/mmol, serum albumin 24 g/L (normal range 35–42), serum creatinine 60 lmol/L(normal range 85–110 lmol/L). Renal Scan is normal the results of a kidney biopsy show normal glomeruli (Fig. 1). What is the most likely diagnosis from the list above? (2*). Answer: A This is a normal glomerulus on histology but on electron microscopy (EM), there is effacement of foot processes (podocytes), confirming minimal change disease. This is commonly idiopathic in children but may be associated with use of non steroidal anti-inflammatory drugs (NSAiDs). 2. A 30-year-old male presents to his GP with ankle swelling over 7 days. On examination he has bilateral ankle swelling, BP 150/100 mmHg, Urine Protein 3+, blood+, serum albumin 26 g/L, serum creatinine 150 mol/L. The kidney biopsy shows scarring of part of the glomerulus (Fig. 2).

S. Bhandari and S. Kraus

Fig. 2 Kidney biopsy stained for collagen showing a glomerulus and surrounding tubules; The Yellow arrow points to part of the glomerulus showing an abnormal area. Permission granted by Hull York Medical School (HYMS)

glomerulosclerosis (FSGS) in this patient causing the nephrotic syndrome. In adults the main causes of nephrotic syndrome to consider are: . . . .

Membranous nephropathy FSGS Diabetic nephropathy Amyloid.

2. A man aged 64 known to be hepatitis C positive presents with increasing leg swelling for 2 months. On examination he had lower limb oedema up to both knees and sacral oedema. His chest was clear with normal heart sounds. BP 145/94. Urine dipstick showed protein 4+, blood negative. Renal scan is normal;urinary protein-creatinine ratio (uPCR) 1600 mg/mmol, serum albumin 14 g/L, serum creatinine 200 mol/l; normal serum complements. The result of a kidney biopsy (Silver staining) shows thickening of the capillary basement membrane (yellow arrow) in Fig. 3. What is the most likely diagnosis from the list above? (4*).

What is the most likely diagnosis from the list above? (3*). Answer: E Answer: C The yellow arrow points to segmental scarring of the glomerulus. Less than 50% of all glomeruli were affected, and the affected glomerulus showed segmental scarring. These 2 points are classic descriptions of focal segmental

This is classic membranous nephropathy which can be either primary idiopathic or secondary as in this case due to hepatitis C. In approximately 10% of cases there is an underlying malignancy as the cause of the nephrotic syndrome and this resolves when the underlying disorder is treated. More

Renal and Transplant Medicine and Urology

275 Fig. 4 Image of both lower limbs showing a characteristic rash. Permission granted by Hull York Medical School (HYMS)

Fig. 3 Kidney biopsy (Silver staining) at high power showing a glomerulus. The yellow arrow points to the capillary basement membrane. Permission granted by Dr. Anu Roy

recently Phospholipase A2 antibodies have been shown to be suggestive of idiopathic membranous nephropathy. MCQ: Theme: Haematuria 1. An 18-year-old man presents with periorbital oedema, pretibial oedema and smoky urine. Three weeks earlier he complained of a sore throat and headaches for which he took ibuprofen. His BP was 150/100 mmHg and urine dip stix showed blood 3+, protein 2+. Serum creatinine was elevated at 150 mol/L. He had a low serum C3 and normal serum C4. ASO titre is raised. Throat swab was negative.

2. A 36-year-old female complains of left loin pain and visible haematuria for one week. On examination, BP 150/100 mmHg, she had a skin rash (Fig. 4). Urine dipstick showed blood 3+, protein 3+. Her Serum creatinine was raised at 250 mol/L with a normal white cell count and CRP. Urine protein creatinine ratio of 350 mg/mmol. Serum immunoglobulins and complements were normal as was a renal scan. What is the most likely cause? (3*). A. B. C. D. E.

Renal vasculitis Interstitial nephritis Post infectious glomerulonephritis IgA nephropathy/Henoch Schonlein Purpura (HSP) Renal stone disease.

Which is the most likely diagnosis? (3*). Answer: D A. B. C. D. E.

Renal vasculitis Interstitial nephritis Post-infectious glomerulonephritis IgA nephropathy/Henoch Schonlein Purpura( HSP) Renal stone disease.

Answer: C

This is classic HSP, which overlaps with IgA nephropathy. It is characterised by a non-thrombocytopenic rash localised to the bathing trunk and lower limbs. It is also associated with painful joints and abdominal pain from gastrointestinal haemorrhage. Treatment is normally symptomatic but may require immunosuppression.

Post streptococcal Glomerulonephritis (GN) is uncommon in the UK now and more common in South America but should not be forgotten. These infections provoke antibody formation leading to an immune complex associated nephritis.

3. A 30-year-old female presents with frank terminal haematuria. She takes aspirin for migraines. Three months ago, she had visited Egypt for a camping trip and had been swimming in the nearby lake. Urine microscopy showed haematuria.

276

S. Bhandari and S. Kraus

What is the most likely cause? (3*). A. B. C. D. E.

Renal cell tumour Interstitial nephritis Schistosomiasis IgA nephropathy/Henoch Schonlein Purpura Renal stone disease.

Answer: C A travel history is important here in addition to the recreation to give the probable diagnosis even before a test. Classically it causes terminal haematuria unlike initial haematuria from urethral bleeding. Theme: Acute Kidney Injury 1. A 70-year-old man presents with fever, loss of appetite and nausea for three days. On examination his temperature was 38.5 C, BP 100/70 mmHg. His white cell count was 18 with a CRP of 40 g/L. Serum urea was 30 mmol/L with a creatinine of 400 mol/L. At a routine medical check-up 4 weeks earlier, he had normal renal function. An ultrasound scan and intravenous pyelogram (IVP) of his renal tract are shown in Figs. 5, 6. What is the cause of the acute kidney injury? (2*). Fig. 6 Contrast IVP of the kidneys, ureters and bladder

A. B. C. D. E.

Rhabdomyolysis Acute interstitial nephritis Haemolytic uraemic syndromes Pneumonia Obstructive uropathy

Answer: E Acute Kidney Injury is usually split into 3 main aetiologies. Prerenal Renal Post renal The majority, over 90%, is pre-renal due to hypovolaemia and sepsis. Next is obstruction as seen in Figs. 5, 6. The ultrasound (Fig. 5) shows a dilated pelvicalyceal renal system with hydronephrosis, while the IVP (Fig. 6) shows the left kidney has contrast remaining in the system with evidence of clubbing of the calyces confirming obstruction of the left kidney. Occasionally this can be due to ureteral narrowing from ischaemia to the ureter. What is the most likely cause of obstruction in this case? (2*).

Fig. 5 Ultrasound scan of the right kidney

Renal and Transplant Medicine and Urology

277

Answer:

What is the cause of the acute kidney injury? (2*).

In men, prostate pathology, either benign prostatic hypertrophy or prostate cancer. In prostate cancer urethral obstruction occurs but bilateral ureteric obstruction can also occur.

A. B. C. D. E.

Rhabdomyolysis Acute interstitial nephritis Haemolytic uraemic syndrome Ethylene glycol poisoning Pneumonia

What is the commonest cause in women? (2*). Answer: A Answer: Ovarian cancer

The diagnosis should be suspected initially based on the history and in particular the social history of drug use and trauma

Where does the blood supply originate for the left ureter? (1*).

Why are the phosphate and potassium very high in this case?

Answer:

Answer:

Left renal artery and superior vesical artery.

These are normally intracellular and hence suggestive of cell damage.

2. A 45-year-old man was treated with amoxicillin for a chest infection. Two weeks later he had a routine blood test which showed a serum creatinine 250 mol/L. He had eosinophilia. C Reactive protein (CRP) was 20. Urine dipstick was normal. What is the cause of the acute kidney injury (AKI)? (3*). A. B. C. D. E.

Rapidly progressive glomerulonephritis Acute interstitial nephritis Haemolytic uraemic syndrome Pneumonia Ethylene glycol poisoning.

Answer: B Interstitial nephritis is a common renal cause of AKI. The most common causes are drugs such as antibiotics or proton pump inhibitors. The urine sediment can be normal. Initial treatment is discontinuation of the offending drug. 3. A 28-year-old man known to use recreational drugs is brought into Accident and Emergency (A&E) by ambulance. He was involved in a road traffic accident and has sustained multiple injuries. He is passing dark urine which is positive for myoglobin on urine microscopy. Laboratory results showed low serum calcium, very high serum phosphate, hyperkalaemia and metabolic acidosis. Serum Creatine Kinase was 40,000 mg/L (normal range 3 g/day (uPCR (protein mg/L /creatinine mmol/L ratio) > 300 mg/mmol) . Hypoalbuminemia (50% of the bone diameter) and the cortex is thinned. There is an ill-defined zone of transition around each lesion. These features would be

299

Fig. 9 Right thigh pain—distal femur. Full and informed patient consent has been obtained to reproduce the image in a commercial publication

consistent with malignancy. Based on the age of the patient, secondary metastatic disease is most likely and investigation should focus on finding the primary cause as well as determining whether there are other secondaries in bone or soft tissues. The patient should be further investigated with a full history (smoking, previous history of cancer, systemic symptoms) and examination including breast and rectal exams. Bloods (full blood count, urea and electrolytes, liver and bone profiles, calcium, prostate specific antigen and myeloma screen) should be done. The serum calcium may be high and this will require emergency treatment with intravenous fluids and bisphosphonates to bring the level down. A CT scan of the thorax, abdomen and pelvis should be performed to look for the primary source and other metastases such as lung and liver. Further lateral views of the femur should be done, and consideration should be given to

300

H. Cattermole et al.

usually curative and it should be assumed that the disease will continue to progress locally. In this case, as there were lesions at high risk of fracture at both proximal and distal ends of the bone, and the estimated prognosis was just 3 months with a poor performance status, locked intramedullary nailing was performed, spanning both lesions. Peri-operative radiotherapy may be considered to reduce local spread caused by surgical intervention. Complications may include implant failure, pathological fracture, disease progression and spread, pulmonary embolism, including tumor embolus, and peri-operative death, in addition to the usual risks of operating on someone with advanced cancer. 4. An 85-year-old lady presented with a 10-month history of tingling and numbness in her left index finger and thumb, worse at night. She was now dropping things and was finding it increasingly difficult to manage alone at home. She had noticed gradual development of deformities in her fingers and thumbs on both hands since her mid-sixties and said that her mother’s hands had been very similar at the same age. On clinical examination there is reduced sensation over the radial border of her left index finger and her left thumb, and the tingling can be reproduced by tapping over the volar aspect of her wrist (Figs. 11 and 12). Describe the features of these hands? (2*) What two conditions are present? (3*) What other tests can be done to confirm the severity of the condition? (3*) What treatment options are available for this condition? (4*)

Fig. 10 Right thigh pain—proximal femur. Full and informed patient consent has been obtained to reproduce the image in a commercial publication

getting an MR scan of the pelvis and both femurs to help pre-op planning. A bone scan could also be done to look for other bone lesions but may be negative with some cancers such as myeloma. There is a risk of pathological fracture on minimal trauma as the bone is so thin. Mirels’ score [2] is used to classify the fracture risk and determine the need for prophylactic surgery —the risk is scored on the lesion’s size, site, density and amount of pain experienced. Patients with a score of 9 or above (out of a maximum of 12) should be considered for surgery. Any implant should be load bearing and the surgery should be planned to last the lifespan of the patient; replacement is preferred if the lesion is close to a joint and cement should be used where possible. Treatment is not

Fig. 11 Both hands in patient complaining of numbness. Full and informed patient consent has been obtained to reproduce the image in a commercial publication

Trauma and Orthopedics

301

branch of the median nerve usually arises before the nerve enters the carpal tunnel. The diagnosis can be confirmed by nerve conduction studies. Treatment options are conservative and operative. Conservative options include wrist splints (particularly for nocturnal symptoms), weight loss (carpal tunnel syndrome is worsened by obesity) and steroid injections into the carpal tunnel. If these fail to control the symptoms, then carpal tunnel release can be performed, usually under local anesthetic. 5. A 22-year-old man broke his left ankle playing soccer and this was treated operatively. One week later he complained of pain underneath his cast and returned to the hospital; his cast was removed and his leg was noted to be swollen. He underwent a Doppler ultrasound scan of the affected leg (Figs. 13, 14, 15 and 16). Describe the clinical findings and the X-rays and the scan pictures? (2*) What is the diagnosis? (3*) How would you manage the condition? (4*) What preventative measures could you take for this condition? (4*) Answer:

Answer:

The clinical pictures show a swollen and deformed left ankle and foot with an abrasion over the medial malleolus (Fig. 13). There is bruising extending distal to the ankle over the medial side of the calcaneus. The plain X-rays are an AP (Fig. 15) and lateral view (Fig. 14) of the left ankle. There is an oblique or spiral fracture through the left fibula which can

Both hands show fusiform swellings at the proximal interphalangeal (PIP) joints of all the fingers (Fig. 11). Additionally, there is some asymmetrical swelling at the distal interphalangeal (DIP) joints of the right and left index fingers, known as Heberden’s nodes. The carpometacarpal (CMC) joints at the base of both thumbs are squared off. All of these features are consistent with osteoarthritis. The left hand demonstrates marked muscle wasting in the thenar eminence and in the first web space (Fig. 12). Along with the clinical history and described examination findings, carpal tunnel syndrome on the left is suspected. Carpal tunnel syndrome (CTS) is caused by compression of the median nerve at the wrist. X-rays should be taken of the affected hand to rule out any bony compression; usually the compression of the nerve is caused by thickening of the flexor retinaculum or swelling of the structures within the carpal tunnel itself. Clinically, CTS can be differentiated from compression proximal to the carpal tunnel by testing sensation over the thenar eminence; the superficial sensory

Fig. 13 Left ankle following soccer injury. Full and informed patient consent has been obtained to reproduce the image in a commercial publication

Fig. 12 Left hand in patient complaining of numbness. Full and informed patient consent has been obtained to reproduce the image in a commercial publication

302

Fig. 14 Lateral X-ray left ankle following soccer injury. Full and informed patient consent has been obtained to reproduce the image in a commercial publication

Fig. 15 AP X-ray left ankle following soccer injury. Full and informed patient consent has been obtained to reproduce the image in a commercial publication

H. Cattermole et al.

be seen more clearly in the lateral view. The fracture is at the level of the ankle joint. On the AP view there is widening of the medial side of the ankle joint indicating talar shift and therefore injury to the deltoid ligament of the ankle. The Doppler scan picture of the popliteal vein (Fig. 16) shows disturbance and reduced flow in the vein, indicating turbulence which might be expected in a non-occlusive thrombus with altered fluid dynamics (a deep vein thrombosis). The ankle joint will need to be reduced and held to allow this fracture and ligament injury to heal before rehabilitation. This can be done conservatively or operatively, however as there is talar shift, the ankle is likely to be unstable making surgery the preferred option. The fibula needs to be fixed with a plate and screws in the anatomical position and the ankle mortise needs to be restored to normal before the leg is immobilized to allow healing to take place. Deep vein thrombosis is a known complication of lower limb injury and is particularly prevalent if the limb has to be immobilized in a cast or brace. Other risk factors include a history of smoking, obesity, cancer, previous thrombosis, family history and certain drugs such as the oral contraceptive pill. The treatment for a diagnosed deep vein thrombosis is oral anticoagulation, usually with rivaroxaban or warfarin, although injectable heparin is sometimes used. This treatment continues for three months after diagnosis and is designed to prevent the clot propagating and/or breaking off to form a pulmonary embolus. Prevention of venous thromboembolism is difficult in acute trauma since risk factors cannot all be mitigated, however, if it is possible to avoid immobilization of both knee and ankle while still stabilizing the injury, this can help. Some authors advocate chemoprophylaxis for high risk patients, with prescription of injectable heparin or selected oral anticoagulants after a thorough risk assessment. 6. A 62-year-old man, who had had a knee replacement seven years previously, slipped on a wet pavement and twisted his right leg. He presented to the ED with an obvious deformity to his right leg and a wound over the medial side of the leg (Figs. 17, 18 and 19). Describe the clinical findings and the X-rays? (2*) What immediate treatment should be done in the ED for this injury? (3*) How would you manage the condition? (4*)

Trauma and Orthopedics

303

Fig. 16 Doppler ultrasound scan. Full and informed patient consent has been obtained to reproduce the image in a commercial publication

Answer:

Fig. 17 Clinical picture right leg following twisting injury. Full and informed patient consent has been obtained to reproduce the image in a commercial publication

The clinical picture shows a 2 cm laceration with a small amount of active bleeding and bruising over the medial side of the right lower leg (Fig. 17). It is just possible to see that the foot is rotated, but without the knee joint included in the picture this is not completely clear. The initial AP X-ray of the right tibia and fibula shows a spiral fracture of both bones beneath a knee replacement; there is 90° external rotation of the foot compared to the knee (Fig. 18). The second AP X-ray of the tibia (Fig. 19) shows that the fracture has been manipulated and placed into a cast. The rotation has been corrected but there is a displaced spiral fracture of the right tibia at the junction of its middle and distal thirds and a more proximal spiral fracture of the fibula. The knee replacement appears to be intact. Any wound near the site of a fracture should be treated as an open fracture until proven otherwise. Given the mechanism of injury this is most likely to be an ‘inside-out’ injury. The immediate management of this injury in the ED should focus on resuscitation of the patient and management of any associated life-threatening injuries. Gross

304 Fig. 18 X-ray right leg following twisting injury. Full and informed patient consent has been obtained to reproduce the image in a commercial publication

Fig. 19 AP X-ray right leg after initial treatment. Full and informed patient consent has been obtained to reproduce the image in a commercial publication

contamination of the wound should be removed, a photograph should be taken of the wound, and it should be covered with a saline-soaked gauze and occlusive dressing. The limb should be re-aligned and splinted in the ED, with documentation of neurovascular status before and after manipulation. Intravenous antibiotics and tetanus prophylaxis should be administered within one hour of the injury as recommended by BOAST guidelines [3]. The patient should be discussed with a specialist orthoplastic unit and transferred there for definitive care.

H. Cattermole et al.

The Gustilo and Anderson classification [4] can be used to guide the management of open fractures; this classification is based on the energy of the injury and the degree of contamination. An initial classification is based on the clinical findings, but a final classification cannot be confirmed until after the patient has been to the operating room for wound debridement. This injury requires operative intervention, in a combined orthopedic and plastic team. As this was a relatively low-energy injury, surgery should be carried out within 24 h; higher energy injuries need surgery more quickly. The principles of surgery are wound debridement (removal of any contamination and any dead or dying tissue) and fracture stabilization, with definitive soft tissue cover of the fracture ideally immediately, or within a maximum of 72 h. There are many options both for surgical stabilization (internal fixation with nail or plate, or external fixation with standard or circular frame) and for soft tissue cover, and these should be decided in a joint operative plan between the two teams. In this case the choice was made to treat the patient with a circular frame due to the knee replacement proximally.

Pitfalls/Mistakes to avoid Compartment syndrome is a potential complication of any injury in a location where skeletal muscle is surrounded by fascia. It is most common in the lower leg, but can occur in the arm, buttock, hand, foot etc. In this condition the pressure in a muscle compartment rises above the perfusion pressure of that compartment, leading to muscle ischemia and potential irreversible muscle and neurovascular damage. Compartment syndrome should be considered in all injuries, particularly when the patient is unable to co-operate e.g. head injury, sedation. In conscious patients, severe and progressive pain that worsens with passive movements is highly suspicious, and this may be associated with firmness of the compartments. Pulses are usually present. The pressure in the affected compartment can be measured by inserting a slit catheter into the muscle compartment and attaching it to a pressure transducer, such as an anesthetic machine or a commercial pressure monitor. Needle compartment pressures are diagnostic (absolute pressure, and pressure relative to the diastolic blood pressure) and the treatment is emergent surgical compartment decompression (fasciotomy). 7. A 68-year-old lady complained of left groin pain and stiffness which was limiting her mobility. The pain was worse on rising from sitting or after activity, and she was also finding it an increasing struggle to put on shoes and

Trauma and Orthopedics

305

Describe the clinical picture and the X-rays? (2*) How might you classify this injury? (3*) How will you manage the condition? (4*) What complications might arise from this injury? (4*) Answer:

Fig. 20 X-ray pelvis in patient with hip pain. Full and informed patient consent has been obtained to reproduce the image in a commercial publication

socks. She had similar symptoms on the right side four years previously and this was treated surgically (Fig. 20). Describe the X-ray findings? (2*) What is the differential diagnosis? (4*) How will you manage the condition? (4*) Answer: There is a right sided total hip replacement, which has a cemented stem and an uncemented cup. The left hip joint space is narrowed, with sclerosis of the bone and some osteophyte formation. These appearances are consistent with osteoarthritis of the hip. The differential diagnosis of hip osteoarthritis includes inflammatory arthritis such as rheumatoid arthritis, osteonecrosis, acetabular dysplasia and infective arthritis. There are conservative and operative options for managing this condition. Simple analgesia e.g. paracetamol, codeine may help the painful symptoms and physiotherapy can be useful in managing the pain and maintaining joint mobility. Weight loss is often helpful, although patients do find it difficult to exercise with hip pain; swimming can be very beneficial. If these are insufficient to control the symptoms, then surgery can be considered, and joint replacement is very successful at returning patients to a formerly active lifestyle. Often these patients are in an older age group and have other comorbidities, such as diabetes or hypertension that will need to be optimized before surgery; weight loss is also important in order to improve surgical safety and outcomes. 8. A 14-year-old boy fell playing soccer and landed on his outstretched right hand. He complained of wrist pain and had an obviously deformed right wrist (Figs. 21, 22 and 23).

The right wrist is deformed, with dorsal angulation (Fig. 21). This is confirmed on the X-rays, which show a dorsally angulated and radially displaced fracture of the right distal radius through the growth plate (Figs. 22 and 23). Pediatric fractures involving the growth plate are classified by the Salter and Harris classification [5]. In this case, the fracture has gone through the growth plate and has taken off a small corner of the metaphysis; this is a Salter Harris II injury and is the commonest type of growth plate fracture. Children are vulnerable to fractures through the growth plate as this is an area of rapidly dividing cartilage and is thus an area of weakness in the bone. This fracture needs to be reduced and held to allow healing before rehabilitation can occur. Reduction can often be performed by a simple manipulation under anesthetic and application of a plaster cast, but in some cases additional stabilization will be required in the form of wires or plates. As the growth plate is involved in this injury, there is a risk of damage to the growing part of the bone. If the damage is complete, the bone may stop growing completely (‘growth arrest’). If it is only partly damaged the subsequent growth may be uneven and cause increasing deformity as the child grows. This is more of a risk with younger children than with older adolescents where growth may be nearly finished. Top tips for imaging . TWO views—always get images in two planes e.g. antero-posterior and lateral views. . TWO joints—always make sure you visualize the joint above and the joint below an injury to avoid missing a nearby dislocation. . TWO sides—pediatric fractures can be difficult to interpret due to the growth plates, and occasionally taking an X-ray of the uninjured side is necessary. . TWO occasions—some injuries e.g. scaphoid fractures, may not be visible on initial X-ray but will show up very clearly 1–2 weeks later when there has been bone absorption at the fracture site. . TWO modalities—some injuries do not show on plain X-ray but are easily seen using another imaging modality e.g. ultrasound, CT or MR scans. If clinically suspicious, consider a different imaging method.

306

H. Cattermole et al.

Fig. 21 Right wrist following fall playing soccer. Full and informed patient consent has been obtained to reproduce the image in a commercial publication

9. A fifty-seven-year-old man woke up one morning with a painful left knee. It was red, hot and swollen, and he felt sweaty and unwell. He was normally fit and active. Earlier that week he had been laying some carpet tiles at home but did not recollect any injury to the knee (Figs. 24 and 25). Describe the clinical pictures? (2*) What is the diagnosis? (3*) How will you manage the condition? (4*)

Fig. 22 AP X-ray right wrist following soccer injury. Full and informed patient consent has been obtained to reproduce the image in a commercial publication

Answer: The left knee of this patient shows erythema and swelling over the anterior aspect of the knee, extending from the proximal pole of the patella to the tibial tuberosity. The skin looks thickened and slightly indurated, but there are no obvious wounds or discharge. The patient is fully able to flex the knee. The diagnosis is prepatellar bursitis, which is an inflammation of the prepatellar bursa, a fluid-filled sac in front of the kneecap. Colloquially, this condition is known as ‘housemaid’s knee’ and may be caused by irritation of the bursa due to prolonged kneeling. It is not likely to be a septic knee joint since the patient has full movement of the knee. The treatment of prepatellar bursitis depends on whether this is thought to be an infected bursa or not. If it is not believed to be infected, treatment is symptomatic, with rest, ice, gentle exercise and measures to protect the knee and prevent this recurring. Non-steroidal anti-inflammatory medications may help the pain and inflammation. Recurrent non-infected prepatellar bursitis may be helped by steroid injection.

Fig. 23 Lateral X-ray right wrist following soccer injury. Full and informed patient consent has been obtained to reproduce the image in a commercial publication

In this case, there needs to be suspicion that the bursa is infected, as the patient has given a history of feeling sweaty and unwell. He should be examined to rule out other causes of malaise, his temperature and bloods should be checked for raised inflammatory markers or a raised white cell count. Some authors advocate aspirating the bursa, but this does run the risk of creating a sinus or spreading the infection; most would start empirical antibiotics, which can be oral or intravenous, and should be targeted at the common infective organisms such as Staphylococcus aureus or other skin organisms. Antibiotic treatment should continue for 10 days; in severe, refractory infections surgical drainage may be indicated.

Trauma and Orthopedics

307

Fig. 24 Left knee pain—extension. Full and informed patient consent has been obtained to reproduce the image in a commercial publication

10. A seventeen-year-old boy fell down while running and injured his left shoulder (Figs. 26 and 27). Describe the clinical pictures? (2*) Describe the X-ray? (2*) How will you manage the condition? (3*) What are the potential complications of this injury? (4*) Answer: There is an obvious fracture with a deformity over the midshaft of the left clavicle. The sharp edge of the proximal end of the fractured bone is very clearly seen beneath the skin, although the skin appears to be intact and is not obviously blanching. The AP X-ray of the left clavicle confirms a fracture of the mid-shaft of the bone, consistent with the clinical findings. The fracture is angulated but not shortened, and does not appear to be comminuted. Further X-rays taken obliquely (AP cephalad angle) will be needed to view the fracture in more detail; these views can sometimes be useful in detecting occult fractures. Fractures of the clavicle can be treated conservatively or operatively. Operative treatment is used for open fractures, some pathological fractures, and when there is significant shortening, separation or comminution (high energy fractures) where healing is less likely or function will be poor. All other

Fig. 25 Left knee pain—flexion. Full and informed patient consent has been obtained to reproduce the image in a commercial publication

fractures are treated conservatively with a shoulder immobilizer until the pain has settled, followed by early mobilization (within two–three weeks) to regain shoulder function. In this case, although the angulation was quite dramatic, there was very little shortening, and the shoulder was in a functional position; rapid healing was expected with resulting full function, so the fracture was treated conservatively.

11. a. A 12-year-old boy sustained an injury to his right leg after some friends fell on him while fighting (Figs. 28 and 29). Describe the X-ray? (2*) How will you manage the condition? (3*) Answer: There is an oblique fracture of the distal tibia in a young person with open epiphyses. There is minimal displacement and angulation and the best treatment is conservative management. He should be placed in an above knee cast (to control rotation) with the foot in neutral, and followed up with weekly X-rays to ensure the position is not lost.

308

H. Cattermole et al.

Fig. 26 Left shoulder injury. Full and informed patient consent has been obtained to reproduce the image in a commercial publication

Fig. 28 Right leg injury AP. Full and informed patient consent has been obtained to reproduce the image in a commercial publication

Fig. 27 Left shoulder X-ray. Full and informed patient consent has been obtained to reproduce the image in a commercial publication

b. The fracture was initially managed nonoperatively in a cast but the position was lost and, after discussion, he was treated with internal fixation (Figs. 30 and 31). Describe the X-rays? (2*) What are the potential complications of this treatment? (4*) Answer: This is an AP and a lateral X-ray of the right distal tibia in a young person with open epiphyses. The oblique fracture of the tibia seen on the previous X-rays has been treated with a plate and screws; there is a single antero-posterior screw in the distal tibial epiphysis, below the growth plate, and lag screw fixation of the fracture with a medial tibial neutralization plate. Potential complications of this treatment include immediate complications such as bleeding, neurovascular injury,

and anesthetic issues, and later complications such as wound infection (superficial and deep), non-union, malunion, growth plate disturbances, metalwork failure and the need for subsequent surgery e.g. to remove the plate. Deep vein thrombosis is less of a risk in children but should be considered in adults having the same surgery. c. Two weeks after surgery he developed some erythema and skin changes around the wound that were initially thought to be eczematous changes but he subsequently developed a foul smelling discharge and the findings below were seen when the cast was removed (Fig. 32). Describe the clinical picture? (2*) What is the diagnosis? (3*) How will you manage the condition? (4*) Answer: There is erythema and what appears to be three sinuses or possible areas of over-granulation along the length of the wound. Given the history of a foul-smelling discharge, sinuses are more likely.

Trauma and Orthopedics

309

Fig. 30 Internal fixation right tibia AP. Full and informed patient consent has been obtained to reproduce the image in a commercial publication

Fig. 29 Right leg injury lateral. Full and informed patient consent has been obtained to reproduce the image in a commercial publication

This clinical picture raises the possibility of deep infection of the metalwork. Infected metalwork is a significant concern in the presence of an unhealed fracture. The key to getting the infection to settle is to provide stability for the tissues and therefore the fracture needs to heal. The other concern would be that the bone becomes infected causing osteomyelitis.

X-rays should be taken to see if there is any sign of deep bone infection—if there are no bony changes then the patient could be offered antibiotic suppression of infection until the fracture heals. Once the fracture has healed the metalwork would be removed and any infected tissue debrided to allow the infection to settle. The other option would be to remove the metalwork at this stage and provide stability with a circular frame until the fracture heals. The risk of suppressing the infection and not removing the metalwork at this stage is that of developing an infected non-union; in this situation the fracture does not unite, and therefore the metalwork has to be removed,

310

H. Cattermole et al.

Fig. 32 Clinical picture following right tibia surgery. Full and informed patient and parent consent has been obtained to reproduce the image in a commercial publication

References

Fig. 31 Internal fixation right tibia lateral. Full and informed patient consent has been obtained to reproduce the image in a commercial publication

infected tissue debrided and an external fixator applied anyway. If there are signs of deep bone infection at this stage the management should be aggressive with removal of metalwork, debridement, stabilization with a frame and antibiotics. Deep bone infection does not allow a ‘watch and wait’ policy and antibiotic suppression is not an option.

1. American College of Surgeons Committee on Trauma. Advanced trauma life support: student course manual. 10th ed. Chicago: American College of Surgeons; 2018. 2. Mirels H. Metastatic disease in long bones. A proposed scoring system for diagnosing impending pathologic fractures. Clin Orthop Relat Res. 1989;249:256–64. PMID: 2684463. 3. BOAST Open Fractures. British Orthopedic Association 2017. https://www.boa.ac.uk/resources/boast-4-pdf.html. 4. Gustilo RB, Anderson JT. Prevention of infection in the treatment of one thousand and twenty-five open fractures of long bones: retrospective and prospective analyses. J Bone Joint Surg Am. 1976;58(4):453–8 PMID: 773941. 5. Salter RB, Harris WR. Injuries involving the epiphyseal plate. J Bone Joint Surg. 1963;45(3):587–622.

Rheumatology Rohini Subramanya and Radhika Raghunath

Glossary

ASAS ACE ANA ANCA ARB AS Anti-TNF BASDAI BMD COPD CRP CCP DEXA DIP DMARDs ECG ESR FBC GCA GPA HRCT HIV ILD IBP MCP MMF MRI

Assessment of SpondyloArthritis International Society Angiotensin converting enzyme Antinuclear antibody Anti-neutrophil cytoplasmic antibody Angiotensin receptor blocker Ankylosing Spondylitis Anti-Tumor necrosis factor Bath Ankylosing Spondylitis Disease Activity Index Bone mineral density Chronic obstructive pulmonary disease C-reactive protein Cyclic citrullinated peptide Dual-Energy X-ray absorptiometry Distal interphalangeal Disease modifying antirheumatic drugs Electrocardiogram Erythrocyte sedimentation rate Full Blood count Giant cell arteritis Granulomatosis with polyangiitis High resolution computed tomography Human Immunodeficiency Virus Interstitial lung disease Inflammatory back pain Metacarpophalangeal Mycophenolate mofetil Magnetic Resonance Imaging

R. Subramanya . R. Raghunath (&) Department of Rheumatology, Hull University Teaching Hospitals NHS Trust, Hull, UK e-mail: [email protected]

MRA NSAID PIP PDE-5 PAH PMR PET–CT PCR PPI PsA RA ReA SSRI SLE ULT VAS

Magnetic resonance Angiogram Non-steroidal anti-inflammatory drug Proximal Interphalangeal Phosphodiesterase-5 Pulmonary arterial hypertension Polymyalgia rheumatica Positron emission tomography–Computed Tomography Protein: creatinine ratio Proton Pump Inhibitors Psoriatic arthritis Rheumatoid arthritis Reactive arthritis Selective serotonin reuptake inhibitor Systemic lupus erythematosus Urate lowering therapy Visual Analogue Scale

Learning Outcome • Describe presenting features of common rheumatological conditions. • Discuss clinical assessment and appropriate investigations. • Describe a few relevant complications of these conditions. • Discuss common principles of management. Pre-study Questions 1. A 64-year-old man presented with a history of fever and a swollen knee joint. He previously had a minor injury to the skin over his knee causing an abrasion. He was taking insulin for diabetes mellitus. On examination, he was pyrexial and had tachycardia. His left knee was warm, swollen, and erythematous. What is the next important step?

R. Subramanya e-mail: [email protected] © The Author(s), under exclusive license to Springer Nature Switzerland AG 2023 K. Wong et al. (eds.), Practical Guide to Visualizing Medicine, https://doi.org/10.1007/978-3-031-24465-0_16

311

312

R. Subramanya and R. Raghunath

A. Give NSAIDs (Non-steroidal anti-inflammatory drugs) for pain relief B. Perform aspiration of the left knee and start appropriate antibiotics C. Request an X-ray and discharge for GP to review D. Intra-articular steroids for symptom relief Answer: B 2. A 19-year-old University student returned after a holiday from Turkey with a rash on her face and was feeling unwell. Examination revealed an erythematous raised rash over the malar area of the face. Her wrists and metacarpophalangeal joints were swollen and tender. There is a family history of lupus. What is the appropriate investigation to be considered? A. Skin biopsy B. Serum angiotensin-converting enzyme (ACE) levels C. Antinuclear antibody test D. CT scan of the sinuses to look for sinusitis Answer: C 3. A 38-year-old woman presented with severe dryness of her eyes and mouth for 6 months. In the last month, she developed pain and stiffness in her small joints. On examination, she was noted to have prominent Parotid glands. Small joints were tender without any significant swelling. Which of the following are true? A. The above symptoms (dry eyes and dry mouth) are referred to as Sicca symptoms and this patient has Sjogren’s syndrome B. This patient has sialolithiasis C. Ultrasound of the salivary glands supports the diagnosis D. Schirmer’s test is helpful in making a diagnosis Answer: A, C and D are true 1. A 32-year-old woman presented with a 4-week history of pain in her hands. She was 3 months postpartum, on further questioning, she gave a history of pain and swelling of her knuckles and wrists, stiffness in joints lasting until mid-day, making it difficult to carry her baby. Her symptoms were worse in the mornings, and she felt some improvement with activity (Fig. 1). Q1. Describe the findings in the image, Fig. 1. (2*) Q2. What is the likely diagnosis? (2*) Q3. What are the investigations that should be arranged for this patient? (3*) Q4. How should this patient be managed? (4*)

Fig. 1 Swelling of proximal and distal interphalangeal joints of both hands. Courtesy Dr. Yusuf Patel

Answers: The hands show evidence of symmetrical involvement of the small joints of the hands with swelling around the metacarpophalangeal (MCP) joints and proximal interphalangeal (PIP) joints. There is subtle erythema around a few joints. These signs are indicative of synovitis of the joints. With the given history and findings, the diagnosis is likely to be rheumatoid arthritis (RA). Rheumatoid arthritis is an autoimmune inflammatory condition affecting the joints causing symmetric polyarthritis. It commonly affects the small joints of the hands, though other joints can also be affected. If left untreated it can cause significant joint damage and marked disability. The investigations to be considered include full blood count (FBC), renal and liver profile, and C-reactive protein (CRP). Immunological tests like anti-CCP antibody test, which is specific for rheumatoid arthritis, and rheumatoid factor test should be done. The radiological investigations include X-rays of the hands and feet, and ultrasound scans of the hands and/or feet. Baseline blood tests show evidence of inflammation (raised CRP, raised platelets, normochromic normocytic anemia) and help with initial workup before considering disease-modifying anti-rheumatic drugs (DMARDs). X-Ray features of early rheumatoid arthritis include soft tissue swelling, juxta-articular osteopenia and occasional marginal erosions. An ultrasound scan can show joint effusion, synovial proliferation, or hypertrophy with features of active synovitis (presence of power Doppler indicating increased vascularity secondary to inflammation) and occasionally erosions. Patients with a new diagnosis of RA are managed with a “treat-to-target” approach. The disease activity is measured using a disease activity score (DAS28 score) based on the

Rheumatology

313

number of tender and swollen joints, CRP or ESR and patient global health assessment score. Patients with a new diagnosis of RA need to be started on DMARD therapy at the earliest opportunity to prevent joint damage. Pain and swelling are managed with analgesics and anti-inflammatories (NSAIDs). During an acute, severe flare, as in this patient, a short course of systemic steroids is helpful in controlling inflammation. Commonly used first choice DMARD is methotrexate. Other alternatives include sulfasalazine, leflunomide and hydroxychloroquine. Contraceptive advice is important in patients in the reproductive age group when considering methotrexate or leflunomide. If therapy with conventional DMARDs fails to control the disease, we need to consider escalating to biologic therapies. Examples of some of the biologics commonly used include anti-tumor necrosis factor antibodies (anti-TNF like adalimumab, etanercept, certolizumab), and other biologics like tocilizumab, tofacitinib and rituximab. Response to treatment is regularly monitored with an aim to achieve early remission or low disease activity. A holistic approach with the involvement of allied health professionals

including physiotherapists and occupational therapists is important to minimize functional impairment and for joint protection.

Fig. 2 A–C Hand deformities in a patient with Rheumatoid Arthritis. Courtesy Dr. Yusuf Patel. D X Ray shows osteopenia and erosions (white arrows) at the left second and fifth proximal phalanx at the PIP

joint. Subluxation at the MCP joints and ulnar deviation are seen in both hands

2. A 55-year-old man presents with long-standing joint problems. He complained that his joints had deteriorated over the last few years. He had been diagnosed with rheumatoid arthritis 25 years previously and had declined any treatment over the years, preferring to use diclofenac intermittently for his symptoms. His main carer reported that he had difficulty in using his hands and required help with activities of daily living over the last year. On examination, he was in a wheelchair. The small joints of the hands and elbows were swollen with significant deformities. There was a limited range of movement in his hips, knees and shoulders, with some swelling in his knees. He had severe deformities of his hands (Fig. 2A–C). Q1. Describe the hand deformities in the images, Fig. 2A–C. (3*) Q2. Describe the X-Ray findings in Fig. 2D. (3*) Q3. How should this patient be managed? (4*) Q4. How could this deformity have been avoided? (3*)

314

Answers: The hands show deformities due to advanced, poorly controlled rheumatoid arthritis. There is swelling over the wrist joint due to synovial hypertrophy. At the metacarpophalangeal (MCP) joints, we can see subluxation (Fig. 2B) and ulnar deviation (Fig. 2A). The little finger shows “swan neck deformity” (Fig. 2B) which is caused by hyperextension at the proximal interphalangeal (PIP) joint and flexion at the distal interphalangeal (DIP) joint. There is loss of extension of the little finger due to rupture of the extensor digiti minimi tendon. Other deformities that could be seen in advanced RA include “Boutonniere” deformity, which is the opposite of “Swan neck” deformity with flexion at the PIP joint and hyperextension at the DIP joint, and “Z thumb” deformity, which is caused by hyperextension at the interphalangeal joint and flexion and subluxation of the MCP joint of the thumb. X-ray shows significant generalized osteopenia and erosions (white arrows) at the left second and fifth proximal phalanx at the PIP joint. Subluxation at the MCP joints and ulnar deviation are seen in both hands. The erosive disease has led to the destruction of the carpal bones and secondary degenerative changes. Principles of management are to treat the condition to prevent progression, symptom control and address the functional disability. Prevention of progression includes pharmacological measures like steroids, DMARDs and biologics. Symptom control is with analgesics including anti-inflammatories. Physiotherapists, occupational therapists, and orthotics should be involved to help with rehabilitation, improving function, and reducing deformity. This presentation could have been avoided with early and effective treatment with appropriate DMARDs and biologics to achieve early remission and good disease control. Referral to a physiotherapist and occupational therapist at diagnosis helps in preventing deformities and disability in the majority. Poorly controlled rheumatoid arthritis increases the cardiovascular risk in individuals. This needs to be addressed in all patients with RA. Patients with RA can also have extra-articular manifestations such as Interstitial lung disease, pulmonary nodules, pleural effusion, pericardial effusion, keratoconjunctivitis sicca and corneal melt, mononeuritis multiplex, vasculitis and neutropenia (can be as part of Felty’s syndrome—a triad of RA, splenomegaly and neutropenia). 3. A 25-year-old woman presented with an 8-year history of pale to bluish discoloration of fingers in cold weather associated with discomfort. They return to their normal pinkish color after rewarming the hands. She normally keeps herself warm while outdoors. There was no history of digital ulcers, skin rash or photosensitivity (Fig. 3).

R. Subramanya and R. Raghunath

Q1. What are the diagnosis and pathophysiology? (3*) Q2. How do you differentiate primary Raynaud’s from secondary Raynaud’s? (4*) Q3. How should this condition be managed? (5*)

Fig. 3 Pale to bluish discoloration of fingers

Answers: The patient most likely has Raynaud’s disease. This is due to reversible vasospasm of the digital vessels on exposure to cold or due to emotional stress. It usually affects the fingers but can also affect the toes, and rarely affects the nose, ears, lips and nipples. The affected digits turn generally white, then blue, sometimes purplish, and then red (triphasic Raynaud’s). These color changes are often associated with pain or a tingling feeling. These changes can last from a few minutes to a few hours. The attacks can be stopped by keeping oneself (both core and extremities) warm. Primary Raynaud’s usually has onset during teenage to early 20s, affects women more often than men and sometimes runs in families. It usually does not progress or worsen in severity over time. Secondary Raynaud’s (Raynaud’s phenomenon) is associated with other conditions such as scleroderma, lupus, Sjogren’s, dermatomyositis and polymyositis. In secondary Raynaud’s patients will have abnormal nail fold capillaries and can develop sclerodactyly, digital ulcers and in extreme cases can develop tuft resorption (resorption of distal bony phalanges – acral osteolysis). Raynaud’s phenomenon can also occur from exposure to vibrating machinery or as a side effect of medications like beta-blockers and ergotamine. Investigations: If based on the history and clinical examination, the likely diagnosis is primary Raynaud’s, then there is no need to investigate further unless the symptoms worsen. If symptoms are severe or there is a likelihood of secondary Raynaud’s further investigations are warranted.

Rheumatology

Blood tests include full blood count (FBC) looking for raised platelet count or raised hematocrit (indicating other potential causes for digital discoloration), urea and electrolytes, Liver function tests and anti-nuclear antibodies (ANA) tests. Nail fold capillaroscopy can be used to look for abnormal capillary loops seen in scleroderma and dermatomyositis. X-rays of the extremities are indicated if the patient has digital ulcers or if there is suspicion of osteomyelitis. Managing symptoms-Advising the patient to keep the core temperature and extremities warm. Advice regarding quitting smoking and minimizing intake of caffeine is helpful. Medications: If the attacks are prolonged and occur despite precautions, severe attacks affect lifestyle, if the patient is developing complications (due to digital ischemia) such as digital ulcers or necrosis of the digital tips then pharmacological therapy needs to be considered. Vasodilators such as calcium channel blockers (nifedipine, nicardipine, diltiazem), angiotensin receptor blockers (losartan), selective serotonin reuptake inhibitors (fluoxetine), phosphodiesterase-5 (PDE-5) inhibitors (sildenafil) and others (discussed further in the scleroderma topic) can be used. 4. A 27-year-old woman presented with a history of progressive shortness of breath and chest pain over a few weeks. On further enquiry, she gave a history of a prominent facial rash over the malar area of her cheeks, which was particularly worse in summer, recurrent oral ulcers, and joint pain mainly affecting the small joints of her hands. Chest X-ray showed a small pleural effusion and her ECG showed changes consistent with pericarditis. Q1. Describe the abnormalities in the images; Fig. 4A, B. (3*) Q2. What is the likely diagnosis? (3*)

Fig. 4 A Malar rash, sparing nasolabial folds. B Livedo reticularis

315

Q3. How should this patient’s problem be taken further? (4*) Q4. How should this patient be managed? (4*) Answers: Figure 4A shows a malar rash, also known as a ‘butterfly’ rash, which is a photosensitive, erythematous flat or raised rash involving the bridge of the nose and the cheeks. Typically, it spares the nasolabial folds. Figure 4B shows livedo reticularis, mottled discoloration of the skin with a reticular or lace-like appearance and cyanotic discoloration of the surrounding pale skin. The likely diagnosis is systemic lupus erythematosus (SLE). It is an autoimmune condition. Patients commonly present with photosensitive facial rash, arthralgias, recurrent and multiple oral ulcers, hair loss, constitutional symptoms like fatigue and fever, headache, chest pain or shortness of breath. On examination, the features include malar rash, oral ulcers, a patchy or generalized non-scarring alopecia, synovitis in the small joints and occasionally lymphadenopathy. They can also present with serositis leading to pleural and pericardial effusion, neurological manifestations including psychosis, delirium and seizures, renal and hematological manifestations. Initial workup includes a detailed systems review, personal history including smoking, pregnancy or plans for pregnancy in women of childbearing age and family history of autoimmune conditions. Comprehensive examination of all systems is important looking for the signs mentioned above. Blood investigations include full blood count to look for anemia and cytopenia, urea and electrolytes to look for renal function, liver function test and immunologic tests such as

316

antinuclear antibody test (anti-chromatin, anti-Sm, anti-dsDNA antibodies are specific). Complement levels (low C3, C4) and anti-phospholipid antibody tests, may be indicated on clinical grounds. Urine dipstick analysis to look for the presence of blood and protein in the urine, which could indicate the possibility of renal involvement (Lupus nephritis). This can be further confirmed with urine protein: creatinine ratio to quantify the proteinuria. Other tests to consider include an electrocardiogram (ECG) to look for evidence of pericarditis and a chest X-ray for pleural effusion. Further investigations are guided by the presentation. Management should be holistic. Acute presentations as in the above scenario will need treatment with systemic steroids. Glucocorticoids are indicated in pericarditis, pleuritis or pleural effusion, severe cytopenia, lupus nephritis, neurological involvement and any other acute or organ-threatening presentations. Glucocorticoids also help in providing rapid symptom relief. Infection should be ruled out prior to starting glucocorticoids. An anti-malarial drug such as hydroxychloroquine is recommended for all patients diagnosed with SLE. Immunosuppressants such as azathioprine and methotrexate help with glucocorticoid tapering and prevent disease flare. Choice of agent depends on age, childbearing potential, disease manifestation and safety concerns. Other immunosuppressants used are mycophenolate mofetil, cyclophosphamide and Rituximab. Cyclophosphamide is used in selected patients with organ-threatening disease. Poorly controlled SLE is a cardiovascular risk factor. 5. A 75-year-old woman presented with a 6-month history of progressive difficulty in gripping things due to tightening of the skin and recurrent discoloration of fingers on exposure to cold. She had a history of frequent digital ulcers with delayed and poor healing. More recently she had noticed progressive shortness of breath on exertion. Q1. Based on the image of the hands (Fig. 5A), what is the likely diagnosis? (4*) Q2. What should be looked for in the history and clinical examination? (4*) Q3. How should this problem be investigated? (5*) Q4. How should this problem be managed? (5*) Answers: The image (Fig. 5A) shows shiny and tight skin over the digits. Few of the digital tips show periungual erythema. Given these features, the likely diagnosis is scleroderma.

R. Subramanya and R. Raghunath

Patients might report the typical triphasic color change of Raynaud’s on fingers even in the absence of significant exposure to cold. Thickening or tightening of the skin over digits resulting in difficulty in making a fist and poor hand grip is seen. Involvement of perioral skin leads to puckered appearance and difficulty in opening the mouth due to decreased aperture. In severe cases, digital ulceration and necrosis or gangrene are seen. With disease progression, sclerodactyly (thickening of the skin of digits) and digital tip resorption (acral osteolysis) may occur. Perioral and digital telangiectasia can also be present. Other presenting features are arthralgia, arthritis, myalgia, and gastro-esophageal reflux due to esophageal dysmotility. Shortness of breath can be due to either lung involvement with interstitial lung disease (ILD) or pulmonary arterial hypertension (PAH). Scleroderma renal crisis is a rare presentation but is a rheumatological emergency and is characterized by new onset malignant hypertension with renal failure. Patients with advanced scleroderma can occasionally develop subcutaneous calcification called calcinosis. Figure 5B shows an X-ray of advanced scleroderma hands with calcinosis. The examination should conclude with a blood pressure check and urine examination. Investigations include full blood count (normocytic anemia), renal and liver function tests, creatinine kinase if patients present with myalgia and ANA screening for anti-centromere and anti-Scl-70 antibodies. Imaging investigations include chest X-ray and high-resolution CT (HRCT) (which might show ILD changes), X-rays of the hands and feet, electrocardiogram, echocardiogram (for evidence of PAH) and lung function tests at baseline and as part of annual monitoring. Changes suggestive of PAH should be followed up by right heart catheterization to confirm the diagnosis. Management of Raynaud’s phenomenon in scleroderma is similar to the management of Raynaud’s as discussed previously (Question: 3) with the addition of topical nitroglycerine patches. In refractory Raynaud’s and pulmonary hypertension, treatment options include prostacyclin analogues (Iloprost and epoprostenol), sildenafil and endothelin receptor antagonist (bosentan). For skin involvement, mycophenolate mofetil (MMF) and methotrexate are used. Cyclophosphamide is used in diffuse and aggressive disease. ACE inhibitors form the cornerstone of treatment in renal crisis.

Rheumatology

Fig. 5 A Shiny and tight skin over the digits. B Calcium deposition in subcutaneous tissue of fingers (calcinosis). Sourced from Radiological identification and analysis of soft tissue musculoskeletal calcification in

317

Insights into imaging (authors: Véronique Freire et al.), licensed under CC-BY 4.0

318

6. A 79-year-old woman was referred urgently with several weeks of headache, scalp tenderness in both temporal regions and a raised CRP. She gave a further history of generalized aches and extreme stiffness across her shoulder girdle, lower back and hips. She was unable to do her activities of daily living, having been independent and enjoying gardening until a few weeks previously. She had switched to pureed foods because of jaw ache when chewing her food. She had significant fatigue. She was struggling to brush and wash her hair due to shoulder aches and stiffness as well as scalp sensitivity. On questioning, she admitted to some blurring of vision in her right eye. Q1. What is the likely diagnosis? (3*) Q2. Why was the referral urgent? (4*) Q3. What investigations should be organized for this patient? (4*) Q4. Describe the management. (5*) Answers: This patient has presented with typical features of polymyalgia rheumatica (PMR) and giant cell arteritis (GCA), also known as temporal arteritis. PMR and GCA are forms of systemic inflammatory vasculitis. They are different stages of the same disease process. PMR is characterized by the recent onset of proximal myalgia in the hips and shoulder girdles with significant morning stiffness lasting for more than 45 min and stiffness after a period of inactivity. It usually presents in patients older than 50 years of age and peaks at age around 70– 80 years. It is more common in women. Onset is usually acute and typical symptoms are stiffness, aches, and difficulty with raising the arm above shoulder in the upper limb, and groin pain and stiffness around the hips and thighs in the lower limb. Systemic symptoms like fatigue and weight loss are commonly associated. GCA presents commonly with new onset or a new type of headache, neck, torso and pelvic girdle pain and stiffness, associated with constitutional symptoms such as fatigue, malaise, weight loss, occasionally fever and night sweats. They can present with scalp tenderness, visual disturbance, jaw and tongue claudication. Patients can also have symptoms of PMR. The onset of symptoms may be either abrupt or insidious lasting for a few days to several weeks before medical attention is sought. Rare complications such as aneurysms, stenosis or dissection can occur in the aortic arch and its branches if the condition is inadequately treated.

R. Subramanya and R. Raghunath

Fig. 6 Ultrasound scan of the temporal arteries showing the “halo” sign

GCA is a medical emergency because of the risk of developing permanent visual loss, stroke or limb ischemia if untreated or inadequately treated. Investigations to be done include full blood count, CRP and ESR or plasma viscosity looking for raised inflammatory markers and thrombocytosis, and liver and renal function tests. Radiological investigations include a chest X-ray to look for mediastinal widening and an ultrasound scan of the temporal arteries looking for the “halo” sign indicative of inflammation of the vessel wall (as shown below in Fig. 6) on transverse view. The arrow points to the temporal artery with thickened vessel wall and the blood flow is shown by the central red area. A temporal artery shows intimal hyperplasia with inflammatory infiltrate and the presence of giant cells. Other investigations to be considered when large vessel vasculitis is suspected include PET-CT (positron emission tomography-computed tomography) and magnetic resonance angiogram (MRA). Glucocorticoids are the mainstay of treatment in a clinically suspected case of GCA and must be started immediately, especially in the presence of visual symptoms or jaw claudication. Blood samples should be taken before or immediately after commencing steroids. If clinical suspicion is high, the first dose of glucocorticoids can be given without awaiting the laboratory results. It is important to educate patients about potential complications of long-term glucocorticoid therapy such as weight gain, osteoporosis and associated fracture risk, increased risk of diabetes mellitus, hypertension, cardiovascular disease, infections, glaucoma and rarely psychosis. High-dose glucocorticoids (prednisolone 40–60 mg orally) are used with gradual tapering over 12–24 months. In patients with significant steroid-related side effects or complications, methotrexate or azathioprine are used as steroid-sparing agents. Tocilizumab (biologic agent) can be used in refractory GCA and in case of relapse.

Rheumatology

319

Answers:

bronchitis, pneumonia, pulmonary nodules (occasionally with cavitation) and pulmonary infiltrates. Patients can present with hemoptysis due to alveolar hemorrhage, which rarely can be life-threatening. Renal manifestations as glomerulonephritis can present with haemato-proteinuria, hypertension and renal impairment. Patients can also present with neurological symptoms such as mononeuritis multiplex. Blood investigations would reveal anemia, neutrophilia and thrombocytosis, inflammatory markers (CRP and ESR/plasma viscosity) would be raised, renal function can be abnormal, and low albumin and total protein can be seen on liver function testing. Immunological tests would reveal a positive anti-neutrophil cytoplasmic antibody test (ANCA), with C-ANCA pattern and antibodies against proteinase 3 (PR3). Nasal cartilage biopsy (affected site in this case) would show necrotizing granulomatous inflammation and vasculitis. The chest X-Ray in Fig. 7 shows reticulonodular interstitial infiltrates and nodules. In addition to these diagnostic investigations, this patient needs the following investigations to look for the extent of the disease; urine dip to look for sediments and haematoproteinuria, urine protein: creatinine ratio (PCR) to quantify the proteinuria, and CT chest to look for lung nodules, infiltrates, cavitation and pulmonary hemorrhage. Sural nerve biopsy is to be considered in the presence of peripheral neurological symptoms. GPA is a potentially life- or organ-threatening condition. Glucocorticoids along with other immunosuppressive therapy are the mainstay of treatment. In the presence of major organ involvement, cyclophosphamide or rituximab are used in combination with glucocorticoids. In non-organ threatening GPA, methotrexate, mycophenolate or azathioprine are used with glucocorticoids. Trimethoprimsulfamethoxazole (Co-trimoxazole) is used for Pneumocystis Jerovicii prophylaxis and there is also some evidence showing it reduces the risk of relapse of the disease.

This patient had recurrent sinusitis, nasal bridge inflammation, arthralgia, cutaneous rash, respiratory symptoms and constitutional symptoms. Given this symptom complex, the likely diagnosis is granulomatosis with polyangiitis (GPA) previously known as Wegener’s granulomatosis. GPA is a necrotizing granulomatous inflammation of medium-sized blood vessels, typically affecting the upper and lower respiratory tracts, and kidneys, but it can involve any organ. Upper respiratory involvement can range from recurrent sinusitis to destruction of the nasal cartilage (due to chondritis) leading to saddle nose deformity. Ear involvement can present as otitis or sensorineural hearing loss. Lower respiratory tract manifestations are common and can range from

8. A 43-year-old woman presented with arthralgia of 2– 3 months duration involving several fingers with significant morning stiffness. On further questioning, she recalled a history of topical treatment for skin psoriasis 4–5 years ago previously with the resolution of symptoms. She also had a history of treatment for probable plantar fasciitis 2 years previously. Q1. Describe the findings in the images (Fig. 8A, B). What is the likely diagnosis? (4*) Q2. What are the other patterns of clinical presentation of this condition? (5*) Q3. How should this problem be worked up? (4*) Q4. How should this patient be managed? (4*)

7. A 34-year-old man presented to an ENT specialist with a 3-month history of swollen and painful nose and recurrent sinusitis. He had a history of joint pains, loss of appetite, weight loss, night sweats, shortness of breath and one episode of hemoptysis. On examination, he had a purpuric rash over his ankles and erythema of the nose with inflamed nasal bridge. Blood investigations, a chest X-ray and a nasal biopsy were performed and, as a result, he was started on treatment. Q1. What is the likely diagnosis, given the presentation and course of his disease? (5*) Q2. What would be expected from the blood tests and the biopsy? (5*) Q3. Describe the findings on the chest X-Ray (Fig. 7) (4*) Q4. What further investigations would be required? (5*) Q5. How should this condition be managed? (5*)

Fig. 7 Reticulonodular interstitial infiltrates and nodules

320

R. Subramanya and R. Raghunath

Fig. 8 A Symmetrical involvement of the small joints of the hands, showing erythema and swelling across several PIP and DIP joints in both hands. B Dactylitis which is a sausage-shaped swelling of toes or fingers. The right 3rd and 4th toes (marked with black arrows)

Answers: The image of the hands (Fig. 8A) shows asymmetrical involvement of the small joints of the hands. It shows erythema and swelling across several PIP and DIP joints in both hands. Image of the Feet (Fig. 8B) shows dactylitis which is a sausage-shaped swelling of toes or fingers. The right 3rd and 4th toes (marked with black arrows in the image) are affected in this patient. With the above features including the history of skin psoriasis, plantar fasciitis, and inflammatory joint symptoms, the likely diagnosis is psoriatic arthritis. Psoriatic arthritis (PsA) involves MCP, PIP and DIP joints. DIP joint involvement is typical of PsA unlike in RA where it is not common. Unlike rheumatoid arthritis, joint

involvement in PsA is often asymmetrical and may be oligoarticular (two to five joints are involved). Five clinical patterns of psoriatic arthritis have been described, but it is not uncommon for patients to have overlap features. a. Distal predominant pattern with involvement of DIP joints b. Oligoarticular asymmetrical pattern with involvement of large joints, mainly in the lower limbs. c. Polyarticular RA-like pattern d. Spondyloarthritis with involvement of axial skeletonPatients present with inflammatory backache with early morning stiffness lasting for more than 45 min. They also present with enthesitis (inflammation of the insertion of

Rheumatology

321

tendons and ligaments to the bone surface), tenosynovitis (inflammation of tendons and their synovial sheaths) and dactylitis (tenosynovitis involving the entire digit). The plantar fasciitis described in this patient is a form of enthesitis and similar changes can be seen at the insertion site of Achilles tendon in a few patients. e. Very rarely arthritis mutilans is seen, where the phalangeal bones are destroyed due to aggressive arthritis. As with rheumatoid arthritis, baseline investigations to be done are—full blood count, liver and renal functions, C-reactive protein and X-Rays of the involved joints. X-ray findings in psoriatic arthritis include joint erosions, joint narrowing, bony proliferation, shaft periostitis, osteolysis including ‘pencil–in–cup’ deformity, acro-osteolysis (resorption of the distal phalanx), ankylosis, spur formation and spondylitis (In axial skeleton). The management of PsA is similar to RA. Symptoms are managed with analgesics including anti-inflammatories and systemic or local intra-articular steroids in appropriate patients. DMARDS are the mainstay of initial management with methotrexate being the first choice if they have skin psoriasis. Other DMARDs include sulfasalazine and leflunomide. Biologics can be considered in patients with poorly controlled PsA despite DMARD therapy. Examples of some of the biologics commonly used are anti-TNF agents (adalimumab, etanercept, and certolizumab), secukinumab, ustekinumab etc. A holistic approach with the involvement of allied health professionals including physiotherapists and occupational therapists is important to minimize the functional impairment and for advice on joint protection. 9. A 65-year-old woman presented with a sudden onset of pain, redness and swelling over her right foot. She had had similar episodes involving her right 1st MTP joint in the previous few months and a medication review with her GP before this episode for heart failure. Q1. Describe the findings in Fig. 9A. What is the likely diagnosis? (2*) Q2. Figure 9B is of another patient with the same condition. Describe the findings in the image focusing on the right index finger. (3*) Q3. Describe the findings on the X-ray in Fig. 9C. (4*) Q4. What are the risk factors for this condition? (4*) Q5. How should this condition be managed? (4*) Answers: The diagnosis here is gout. The image (Fig. 9A) shows diffuse swelling of the right foot with erythema and induration, predominantly over the 1st MTP joint. Figure 9B shows nodular swelling in the soft tissue of the right index finger. This is a gouty tophus that is caused by the deposition of monosodium urate crystals in the soft

Fig. 9 A Diffuse swelling of the right foot with erythema and induration, predominantly over the 1st MTP joint Image courtesy Dr. Rahul Shah. B Nodular swelling in the soft tissue of the right index finger. Image courtesy Dr. Rahul Shah. C Soft tissue swelling, punched-out erosions (White arrow pointing to left 2nd MCP joint) with sclerotic borders and preserved joint space

tissue. The image also shows diffuse indurated swelling over the right MCP joints. Gout is a form of crystal arthropathy leading to inflammatory arthritis due to the deposition of monosodium urate crystals in the joints. Men are affected more commonly, and it can also affect postmenopausal women. The classical presentation is characterized by self-limiting episodes of sudden onset of severe pain, erythema and swelling of the

322

R. Subramanya and R. Raghunath

1st MTP joint. This is called “podagra”. Gout can also affect other joints and cause surrounding soft tissue inflammation due to tenosynovitis. The affected joint and periarticular area are warm to touch with symptoms reaching maximum intensity within 24 h. Other presentations include—gouty tophi (as seen in Image 9B), bursitis (commonly olecranon bursitis), and rarely it can present as polyarthritis. Tophaceous gout occurs in patients with long-standing uncontrolled gout and is due to subcutaneous and soft tissue deposition of the urate crystals resulting in white chalky material visible underneath the skin (as in Fig. 9B). Occasionally this chalky substance can discharge through the skin. Gouty tophi can cause erosions of the underlying bone. Risk factors for acute gout flare are alcohol intake, overconsumption of purine-rich food such as red meat and seafood, dehydration, medications like aspirin, diuretics (as in this case), nicotinic acid, fructose, cytotoxic agents etc. Relevant investigation in suspected gout includes blood tests such as full blood count (showing neutrophilia), renal and liver function tests, serum uric acid levels, and C-reactive protein. Joint fluid aspiration and examination under a polarized microscope for uric acid crystals help make a definitive diagnosis of gout. It also helps to rule out the possibility of septic arthritis. X-rays of the affected joint (Fig. 9C) show soft tissue swelling, punched-out erosions (White arrow pointing to left 2nd MCP joint) with sclerotic borders and preserved joint space. Treatment of acute gout includes rest and elevation of the limb to help with symptom control along with pharmacological agents. The first line of medications includes anti-inflammatories (NSAIDs) or colchicine. PPI’s are prescribed along with NSAIDs for gastroprotection and the most common side effect with colchicine is diarrhea. Intra articular steroids could be considered in monoarticular gout, after checking for contraindications such as infection and bleeding risk. Systemic steroids (oral or intramuscular) are considered in patients who cannot tolerate NSAIDs or colchicine, and in case of polyarticular gout. Prevention of recurrent gout attacks and long-term treatment of gout includes urate-lowering therapy (ULT) with allopurinol and febuxostat. Patients are at risk of developing acute gout attacks at the initiation of ULT due to a rapid reduction in serum uric acid levels. It is advisable to co-prescribe either NSAIDs or colchicine for initial few weeks to months along with ULT. It is important to discuss lifestyle modifications and risk factors as mentioned above with the patient at the time of diagnosis.

episodes of severe lower back pain and alternating buttock pain associated with significant early morning stiffness lasting up to 2 h. He noticed that movement and exercise improved his symptoms considerably. Q1. Describe the findings in Fig. 10A. (2*) Q2. What is the likely diagnosis? What are the clinical features? (4*) Q3. Comment on the image given below (Fig. 10B) from another patient who remained untreated for several years. (3*) Q4. What relevant investigations should be considered in this condition? (4*) Q5. Discuss the management. (4*)

10. A 43-year-old man presented with several years’ history of lower back pain and recent onset of neck stiffness affecting his work. On enquiry, he gave a history of

Fig. 10 A X-Ray of the sacroiliac joints shows sclerosis and fusion of sacroiliac joints bilaterally as highlighted by the black arrow at the left sacroiliac joint. B Bony ankylosis giving a ‘bamboo spine’ appearance

Rheumatology

Answers: X-ray of the sacroiliac joints shows sclerosis and fusion of sacroiliac joints bilaterally as highlighted by the black arrow at the left sacroiliac joint in Fig. 10A. The typical history of inflammatory back pain with onset at less than 45 years of age is suggestive of ankylosing spondylitis (AS). Inflammatory back pain (IBP) is characterized by pain and stiffness of the back (spine) and buttocks (sacroiliac joints) associated with significant early morning stiffness lasting for more than 30 min and improving with activity. Nocturnal pain, with an onset before 45 years of age and lasting for at least 3 months are characteristic. It responds well to NSAIDs within 48 h. Most patients with AS have a positive family history. Clinical features of AS include reduced range of movement of the spine, reduced mobility, and postural changes, including kyphosis, due to paraspinal muscle spasms. Extra-axial manifestations include peripheral arthritis, enthesitis (inflammation at the site where tendon, ligament or joint capsule inserts into the bone) and anterior uveitis. Fatigue can be a significant symptom. A comprehensive history and examination of the systems should be done in all patients with suspected AS. Commonly used diagnostic criteria include—Modified New York Criteria and ASAS (Assessment of Spondyloarthritis International Society) classification criteria based on symptoms, radiological features and HLA-B27 status. If the condition is left untreated vertebral bodies develop small erosions at the corners with reactive sclerosis (shiny corner sign on MRI), vertebral body squaring then occurs and diffuse syndesmophyte formation results in bony ankylosis giving a ‘bamboo spine’ appearance as seen in Fig. 10B. Investigations to be considered in AS include—full blood count, renal and liver function tests, C reactive protein, and HLA-B27 test. Radiological investigations to be considered include X-rays of the sacroiliac joints and the affected area of the spine. A Magnetic Resonance Imaging (MRI) scan helps identify early changes that may not be visualized on the X-rays (non-radiographic axial spondyloarthritis) and to identify active inflammation. Treatment of suspected AS consists of NSAIDs along with gastroprotection with proton pump inhibitors (PPI) and simple analgesics (paracetamol with or without codeine). Treatment of confirmed AS consists of pharmacological and non-pharmacological options. Pharmacological options include NSAIDs and biologics. Biologics including anti-TNF therapy should be considered in patients with severe disease who have tried at least two different NSAIDs with no benefit. Treatment is monitored by using the Bath Ankylosing Spondylitis Disease Activity Index (BASDAI) and Pain Visual Analogue Score (VAS). Non-pharmacological management mainly involves physiotherapy.

323

11. A 65-year-old woman presented with a history of sudden onset of severe, sharp pain in the mid-thoracic spine while she was trying to get out of the bath 4 weeks previously. The pain was not relieved with simple analgesics including NSAIDs, which prompted her to seek medical help. She gave a history of right distal ulnar fracture 5 months previously after a trivial fall. She had a history of longstanding rheumatoid arthritis with frequent systemic steroid therapy for flares of her arthritis. Her mother had been treated for a fractured hip. Q1. Based on the history and X-ray findings in Fig. 11, what is the likely cause of back pain? (2*) Q2. What are the predisposing factors for this condition? (3*) Q3. How should the diagnosis be confirmed? (3*) Q4. How should this patient be managed? (4*)

Fig. 11 A wedge-shaped compression fracture of the vertebral body as indicated by the white arrow

324

R. Subramanya and R. Raghunath

Answers: The above patient developed severe back pain after a routine activity and the X-ray of the spine (Fig. 11) shows a wedge-shaped compression fracture of the vertebral body as indicated by the white arrow. Sudden onset of back pain with no significant trauma in patients with risk factors for osteoporosis should arouse suspicion of vertebral fracture. An osteoporotic fracture is a fragility fracture occurring because of osteoporosis. Vertebral fractures may occur spontaneously or as a result of routine activities like bending or lifting. Other common sites of osteoporotic fractures include hip and distal radius, but can also occasionally involve the humerus, pelvis and ribs. These fractures occur following trivial trauma, for example a fall from standing height resulting in a hip fracture. Predisposing factors for developing osteoporosis include —female sex, early menopause, older age, family history of osteoporosis, malnutrition, immobility, inflammatory conditions (such as rheumatoid arthritis, inflammatory bowel disease), hyperthyroidism, hyperparathyroidism, long term systemic steroids, chronic kidney disease, chronic liver disease, a body mass index (BMI) of less than 18.5 kg per m2, an alcohol intake of more than 14 units per week, smoking and others. Initial confirmation of fracture is through the X-ray of the affected site (bones or joint). Further imaging in the form of Dual-Energy X-ray absorptiometry (DEXA) scan should be done to assess the bone mineral density (BMD). Osteoporosis is defined by WHO as a BMD 2.5 standard deviations below the mean peak mass (T-score < −2.5). Osteopenia is defined as a T-score between −1.0 and −2.5. Investigations in a suspected case of osteoporosis and fragility fracture include blood tests to look for underlying causes including full blood count (FBC), renal and liver function tests, bone profile including Vitamin D and parathyroid hormone levels and thyroid function tests. X-rays of the affected joint and bone are required. An X-ray of the spine should be considered as vertebral fractures are clinically silent in 50–70% of patients. DEXA scan is performed to assess the BMD and for monitoring while on treatment. Vertebral fractures can be mostly managed with symptomatic treatment including analgesics, gentle mobilization and physiotherapy. Vertebroplasty can be considered if the pain is poorly controlled despite optimum analgesia. If the fracture is felt to be unstable a neurosurgical referral should be made. Management of osteoporosis includes: a. Identify any underlying risk factors and address them with appropriate lifestyle advice where applicable, for

b. c.

d.

e.

example, smoking cessation, moderation of alcohol intake, regular exercise, healthy and balanced diet. Bone mineral density should be checked using a DEXA scan to assess the risk of further fragility fractures If the T-score is −2.5 or less, they should be considered for treatment with bisphosphonates (commonly used include alendronate and risedronate). Patients on corticosteroids may need to be considered for treatment with bisphosphonates with a higher T-score based on the clinical risk of fragility fractures. All patients with osteoporotic fractures should be prescribed Vitamin D and calcium supplements unless contraindicated. Analgesics for pain.

12. A 36-year-old woman presented with a painful and swollen knee, and difficulty in walking. She had gone on a camping holiday in the woodlands a few days previously where she had a minor injury around her left knee. Clinically, temperature 38 °C, heart rate 110 beats/min and systolic blood pressure of 86 mmHg. Her right knee joint was swollen, erythematous and warm (Fig. 12). Turbid fluid was aspirated from the affected joint. Q1. What is the likely diagnosis? (2*) Q2. What is the role of Joint aspiration? (2*) Q3. What are the management principles? (4*) Q4. What features in this patient highlight the severity of the condition? (4*)

Fig. 12 Right knee joint showing swelling and redness

Answers: This patient has acute monoarthritis. The differentials include septic arthritis, crystal arthropathy, reactive arthritis and inflammatory arthritis. She was febrile, tachycardic, hypotensive and had a history of recent trauma potentially acting as the site of entry of infective organism and turbid

Rheumatology

knee joint aspirate. This makes septic arthritis the likely diagnosis. Patients with septic arthritis usually present acutely with a triad of fever, pain, and impaired range of movement in the affected joint. Symptoms may evolve over a few days to weeks. It can be either monoarticular or polyarticular. There may be a previous history of joint disease or trauma. Conditions such as diabetes, liver disease, hematological malignancies, and treatment with immunosuppressive drugs, can predispose to the development of septic arthritis. Joint fluid analysis is the cornerstone of the diagnosis of septic arthritis. It includes microscopy, gram stain, culture and sensitivity. The joint fluid analysis also helps in the differential diagnosis of crystal arthropathy or reactive arthritis. Blood cultures should be done, ideally before commencing antibiotics. Other investigations to be carried out include full blood count, C reactive protein, renal and liver function tests. These tests are likely to be abnormal in the presence of infection and help monitor response to treatment. It is important to remember that normal blood tests do not exclude septic arthritis. X-rays of the affected joints can help exclude other conditions and can show destructive changes if septic arthritis is left untreated for more than 2 weeks. Common organisms in septic arthritis include Staphylococcus aureus and Streptococcus. Gram-negative organisms are found in immunocompromised patients, extremes of age and intravenous drug abusers. This patient was septic at presentation as she had hypotension, tachycardia and pyrexia. Septic arthritis is a medical emergency. General principles of management of sepsis include-fluid resuscitation, early initiation of broad-spectrum antibiotics while awaiting cultures and washing out of any involved large joints. Symptoms are controlled with analgesics and antipyretics. The commonly used antibiotic is flucloxacillin and further treatment is guided by cultures and response to initial treatment. 13. A 51-year-old man presented with a sudden onset of pain and swelling in multiple joints, feeling very ill and unable to move. He gave a history of pain and discomfort on micturition, a few days prior to the onset of his joint symptoms. He had no urethral discharge or fever. Examination revealed a swollen and tender right hand and digits (Fig. 13). He also had mild swelling of his left knee and right ankle. The remainder of the examination was unremarkable. Q1. What is the probable diagnosis in this patient? (2*) Q2. What further investigations should be considered in this patient? (3*) Q3. Are there specific specialist referrals to make and why? (3*) Q4. What treatment should be offered to this patient? (3*)

325

Fig. 13 Swollen and tender right hand and digits. Image courtesy Dr. Rahul Shah

Answers: The diagnosis here is likely to be reactive arthritis given the temporal association between his preceding genito-urinary symptoms and the onset of his joint symptoms. Reactive arthritis (ReA) develops in genetically predisposed healthy individuals after 1–6 weeks after an episode of triggering infection—either gastrointestinal or genitourinary. It is believed to be due to an aberrant autoimmune response to the infection. At the time of presentation, the patient usually has inflammatory joint symptoms, but the triggering infection has passed. Hence, a detailed history is very important. The recent infection can be demonstrated by serological tests. ReA has a good prognosis, and most patients recover within a few weeks or months. However, there is a tendency for arthritis to be recurrent in a few individuals. The presenting features include inflammatory arthritis, which could be peripheral or axial. Peripheral arthritis is usually asymmetrical oligoarthritis predominantly affecting the large joints of the lower limb. Rarely, it can also present as polyarthritis involving upper limb joints, as in this patient with the involvement of small joints of the hand (Fig. 13). Extra-articular manifestations are—fever, enthesitis, dactylitis (swollen, sausage-shaped toes/fingers), conjunctivitis and iritis. Skin involvement is in the form of keratoderma blenorrhagicum (hyperkeratotic hyperpigmented plaques over soles) and circinate balanitis (small, shallow painless ulcers of the glans penis or urethral meatus) and nail dystrophy. Laboratory tests and imaging are not definite for the diagnosis of ReA. History and physical examination findings are cornerstones for the diagnosis and a high index of suspicion is needed. Full blood count may show normocytic normochromic anemia with leukocytosis, inflammatory markers like ESR and C reactive protein are elevated and normalize when the inflammation subsides. Urine cultures for chlamydia or ureaplasma may be positive but can be negative if obtained several weeks after the onset of

326

symptoms. If a patient has cervicitis or urethritis, discharge should be sent for cultures. The urethral smears can be sent for anti-chlamydial staining using enzyme immunoassay or by direct fluorescent antibody testing. Polymerase chain reaction (PCR) assay can be considered. In suspected gonorrhea, staining of the urethral discharge reveals intracellular gram-negative diplococci. Common enteric pathogens include Salmonella, Shigella and Yersinia. Stool cultures are helpful in the presence of enteric infection to guide antibiotic choice; however, the cultures are often negative at the time of presentation. In patients with a history suggestive of high risk for HIV, referral to specialists for management is appropriate. HLA B27 positivity most probably represents a risk factor for severity. X-ray findings of the affected joints may vary from normal to various arthritic changes depending on the duration or chronicity of the symptoms. In more advanced disease, ReA erosions typically have indistinct margins with surrounding periostitis; the affected tendon insertion site shows periosteal reaction and proliferation. Exuberant plantar spurs are common. X-rays of the spine and sacroiliac joint may show sacroiliitis and syndesmophytes. If there is a preceding history suggestive of genitourinary infection as a cause for reactive arthritis, referral to the sexual health clinic for further tests and treatment is appropriate. Individuals with high-risk behavior are also screened for other sexually transmitted infections including HIV. Patients with suspected iritis should be referred to an ophthalmologist for diagnosis and management. In the acute phase, management is primarily symptom control with analgesics, rest and non-steroidal antiinflammatories (NSAIDs). Intra-articular or systemic steroids can be used. Persistent infections will need treatment with appropriate antibiotics. Reactive arthritis is self-limiting in most individuals and resolves in 3–12 months. It can occasionally persist for longer and may need longer-term therapy with disease modifying agents (DMARDs).

14. A 43-year-old woman presented with a history of pain and stiffness in the small joints of the hands for several weeks. On further questioning, she had a history of erythematous rash on her legs 3 months previously. She had had a recent episode of painful red eyes which was treated as uveitis with steroid eye drops. She had noticed shortness of breath for the past few weeks with limited exercise tolerance. On clinical examination, she had tender and swollen metacarpophalangeal joints and ankles bilaterally.

R. Subramanya and R. Raghunath

Q1. What does the image in Fig. 14A show? Describe the lesions on the feet. (3*) Q2. Describe the chest x-ray findings in Fig. 14B (4*) Q3. Based on the history and the findings in the image, what is the likely diagnosis? (3*) Q4. What other systems should be assessed to determine the extent of involvement? (4*) Q5. How should this patient be managed? (4*)

Fig. 14 A Erythematous subcutaneous lumps with some induration. Image courtesy Dr. Simi Sudhakaran. B Wide mediastinum due to hilar lymphadenopathy, marked with white arrow

Rheumatology

Answers: The image (Fig. 14A) shows erythematous subcutaneous lumps with some induration suggestive of erythema nodosum. It commonly involves the anterior shin but can also affect other areas as in this patient. The chest X-ray shows a widening of the mediastinum due to hilar lymphadenopathy (Fig. 14B). With a history of uveitis, arthritis, erythema nodosum and hilar lymphadenopathy, the diagnosis of sarcoidosis is highly probable. Sarcoidosis is a multisystem disease of unknown etiology and characterized by infiltration of various tissues by non-caseating granulomas. It is self-limiting in the majority and very few develop a chronic course. Musculoskeletal manifestations are seen in a third of patients. The most frequent manifestation is acute arthritis, which occurs as part of Lofgren’s syndrome, which is characterized by a triad of symmetric hilar lymphadenopathy, joint pain and erythema nodosum. Ankles are the most commonly affected joints, but others are also known to be affected. Lofgren’s syndrome is associated with an excellent prognosis settling spontaneously over a few months in the majority. Chronic sarcoid-related arthritis usually manifests along with other extrapulmonary involvement. It is a symmetric, medium to large joint oligoarthritis. Tenosynovitis is more common than true synovitis. In addition, sarcoidosis can also involve the axial skeleton, and cause dactylitis and myopathy. Other system involvement includes eyes (uveitis is the commonest), skin (erythema nodosum, lupus pernio), lungs (hilar lymphadenopathy and interstitial lung disease), cardiac (conduction defects), and rarely neurological, renal and hepatic manifestation. Mortality is due to progressive pulmonary fibrosis or Cardiac involvement leading to arrhythmias, congestive cardiac failure or sudden cardiac death.

327

The full blood count may show anemia, lymphopenia and thrombocytopenia indicating bone marrow involvement. Blood biochemistry shows raised alkaline phosphatase (ALP) levels and impaired renal function. If baseline screening shows raised calcium levels, Vitamin D levels should be measured. These can be raised because of the increased production of 1,25 hydroxyvitamin D. Serum angiotensin converting enzyme (ACE) levels can be elevated but it is not necessary for diagnosis. Further investigations for musculoskeletal involvement include X-rays of the affected joint/bone. Any further scans or investigations will be guided by the presentation and X-ray findings. Not all patients require systemic steroids. It is mainly used for life threatening organ involvement and severe acute arthritis. In this section, the focus has been on the treatment of musculoskeletal sarcoidosis. Symptomatic treatment with analgesics and NSAIDs is usually sufficient in acute arthritis. Systemic steroids and steroid sparing agents (methotrexate, leflunomide, mycophenolate) are used in chronic arthritis and myopathy. Other organ involvement would need a referral to the respective specialties for further management.

Recommended Reading 1. Clunie G. Oxford handbook of rheumatology. 4th ed. Oxford: Oxford University Press; 2018. 2. Klippel JH. Primer on rheumatic diseases. 13th ed. New York: Springer; 2008. 3. Firestein GS. Firestein and Kelley’s textbook of rheumatology. 11th ed. Philadelphia: Elsevier; 2020. 4. Watts RA. Oxford textbook of rheumatology. 4th ed. Oxford: Oxford University Press; 2013.

Infectious Diseases

17

Joanna Allen

Learning Outcomes . To use information from history-taking and examination to formulate an appropriate differential diagnosis for common presentations of infection such as fever, rash and breathlessness . To be aware of the most common and important presentations of infection in the returning traveller. . To perform basic chest X-ray interpretation. . To describe and recognise some of the more serious causes of skin rashes . To recognise the link between specific pathogenic organisms and clinical syndromes . To describe basic microbiological investigations such as microscopy and staining, culture and nucleic acid amplification techniques and to know when they are indicated. . To recognise common presentations of HIV infection and to appreciate the wide differential diagnosis of opportunistic infections and malignancy Pre-study Questions

For each of the following scenarios select the most likely diagnosis from the list above. 1. 29-year-old woman, originally from Uganda, who presents with a 3-month history of drenching night sweats, weight loss and a cough productive of blood-stained sputum. Examination is unremarkable. Answer: C 2. An 18-year-old man returns from university with a one-week history of fever, sore throat and difficulty swallowing. He has enlarged tonsils and cervical lymphadenopathy on examination. Blood tests reveal raised liver enzymes. Answer: D 3. A 61-year-old woman presents with a 7-day history of fever, headache, dry cough and myalgia with shortness of breath since yesterday. Examination reveals a temperature of 390C and saturations of 92%, respiratory examination otherwise normal.

EMQ Answer: E Fever A. B. C. D. E. F. G. H.

Malaria Typhoid Tuberculosis Infectious mononucleosis COVID-19 Pneumonia Infective endocarditis Primary HIV infection.

J. Allen (&) Department of Infectious Diseases and Travel Medicine, Leeds Teaching Hospitals NHS Trust, West Yorkshire, England e-mail: [email protected]

4. A 79-year-old man presents with a 5-day history of fever, anorexia and feeling generally unwell. On examination his oxygen saturations are 90% on air, his respiratory rate is 24 and he has bronchial breath sounds at the left base of the lungs. Answer: F 5. A 22-year-old man returns from 3 months backpacking around South-East Asia. He has a 4-day history of fever, sore throat, headache and rash. On examination his

© The Author(s), under exclusive license to Springer Nature Switzerland AG 2023 K. Wong et al. (eds.), Practical Guide to Visualizing Medicine, https://doi.org/10.1007/978-3-031-24465-0_17

329

330

J. Allen

temperature is 39.10C and his throat is erythematous. He has cervical, axillary and inguinal lymphadenopathy and a maculopapular rash on his neck, torso, limbs, palms and soles.

denies any other recent illnesses. On examination he has a unilateral vesicular rash extending from the mid-thoracic region round to the mid-axillary line. Answer: G

Answer: H 6. A 40-year-old woman presents with fever, headache, vomiting and myalgia for the past 2 days. She returned from a 10-day safari holiday in Kenya 6 days ago. On examination she looks unwell, pale, and is febrile at 38.4ºC with a pulse of 115 beats per minute. She is drowsy with a Glasgow Coma Scale (GCS) of 13. Answer: A

2. A 26-year-old man returns from a month-long holiday travelling around Brazil. He has been unwell since his return to the UK 3 days ago. He complains of fever, severe myalgia, headache and a pink rash on his chest, abdomen and thighs. Answer: C

7. A 34-year-old man presents with a 2-week history of fevers, night sweats, fatigue and breathlessness. He regularly uses substances and last injected heroin 4 days ago. On auscultation of his lungs, he has bilateral scattered crepitations, and a pansystolic murmur best heard at the left sternal border.

3. A 43-year-old lady returns from a business trip to Singapore with a 24-h stopover in Kuala Lumpur. While she was in Singapore, she recalls getting multiple mosquito bites on her lower legs. One bite on her left ankle was particularly itchy and her lower leg has become progressively more swollen, hot and painful over the past few days. She has now started to feel nauseous and unwell.

Answer: G

Answer: E

8. A 59-year-old lady presents with a 9-day history of intermittent fever. She is originally from India but lives in the UK. She returned from visiting her family in Delhi 2 weeks ago. She now complains of persistent fever, abdominal pain and constipation. On examination her temperature is 38.1ºC, pulse 82 beats per minute. Her abdomen is soft, but she has generalised abdominal tenderness.

4. A 52-year-old lady presents with an itchy rash of 24-h duration. She has a previous history of asthma and is recovering from a recent infective exacerbation, for which she was seen by her GP 2 days ago. Her chest symptoms are improving but on examination she has a widespread blanching maculopapular rash. Answer: H

Answer: B Skin lesions (A) (B) (C) (D) (E) (F) (G) (H)

Meningococcal meningitis Cutaneous larva migrans Dengue fever Varicella zoster Cellulitis Measles Shingles Drug reaction.

For each of the following scenarios select the most likely diagnosis from the list above. 1. An 84-year-old man presents with a painful rash on his back for the past 4 days. He is systemically well and

5. A 22-year-old man presents with an itchy rash on his back which has been present for the past 2 days. His only recent travel is to Tenerife for a beach holiday 2 months previously. On examination he has an excoriated raised linear lesion on his back. Answer: B 6. A 6-year-old boy is brought to his GP with a 2-day history of fever, malaise and anorexia followed a day later by an itchy rash. On examination he has multiple macules and a few vesicles on his face and torso. Answer: D 7. A 3-year-old girl was brought to her GP with a history of fever, malaise, runny nose and eyes which began 5 days ago and was followed 2 days later by a rash. On

Infectious Diseases

examination she looks miserable with bilateral conjunctivitis, and a widespread maculopapular rash which is confluent in places. Examination of her throat is normal, but she has a few grey spots on her buccal mucosa.

331

C. Herpes zoster D. Recurrent respiratory tract infections E. Persistent generalised lymphadenopathy. Answer: B

Answer: F Cases: 8. A 16-year-old boy is brought to the emergency department with a 2-day history of fever, headache, neck pain and lethargy. On examination he is drowsy, febrile and looks unwell. Over the past few hours his mother has noticed the appearance of a rash over his legs which is purpuric and non-blanching. Answer: A MCQ

1. This 20-year-old man attends the Emergency Department with a 24-h history of fever, myalgia, headache and vomiting. A rash appeared on both feet 4 h previously and is now on his legs and torso. On examination his blood pressure is 90/60, pulse 120, beats per minute respiratory rate of 26, and a temperature of 38.5ºC. He has neck stiffness, photophobia, a widespread rash, GCS 13, neurological examination including fundoscopy is otherwise normal.

1. Which of these organisms is the most common cause of infectious endocarditis? A. Streptococcus bovis B. Staphylococcus epidermidis C. Enterococcus faecalis D. Staphylococcus aureus E. Candida albicans.

(1a) Describe this rash in Fig. 1 (2*).

Answer: D

Answer:

2. Which of these methods is the most sensitive in the diagnosis of Tuberculosis? A. Chest X-ray B. Auramine-phenol stain and microscopy C. Ziehl - Neelsen stain and microscopy D. Culture E. Nucleic acid amplification techniques.

Meningitis with meningococcal sepsis, caused by Neisseria meningitidis. This is a life-threatening condition that can lead to rapid deterioration of the patient and is associated with significant morbidity and mortality. Meningococcaemia with this characteristic rash and meningococcal meningitis can each occur in isolation, or together. This patient has neck stiffness, headache and photophobia which suggest meningitis.

Answer: D 3. Which of these statements regarding Varicella Zoster Virus (VZV) is correct? A. The primary infection commonly causes cold sores B. Can cause pneumonia C. Herpes zoster can be acquired by direct contact with the vesicle fluid. D. The incubation period is 5–10 days. E. There is no current vaccine for VZV. Answer: B 4. Which of these conditions is an AIDS-defining illness? A. Oral candidiasis B. Esophageal candidiasis

Answer: This is a petechial rash affecting the foot. (1b) Name this condition and the causative organism (3*).

(1c) What are the complications of this condition? (3*). Answer: . Shock—this results from increased vascular permeability and requires fluid resuscitation. Patients with shock that fails to respond to adequate fluid resuscitation will require inotropic support . Disseminated intravascular coagulation (DIC) . Raised intracranial pressure—causing papilloedema and seizures. Lumbar puncture is contra-indicated in patients with signs of raised intracranial pressure . Focal neurological deficits such as sensorineural hearing loss

332

J. Allen

Fig. 1 Rash on foot. Permission granted by Hull York Medical School (HYMS). Full and informed consent to use in commercial publication

. Necrosis of skin and extremities—results from DIC and vasculitis and may require surgical debridement and skin grafting. (1d) How is this condition managed? (2*). Answer: . . . . . .

Airway, Breathing, and Circulation (ABC) approach Follow hospital sepsis guidelines Intravenous fluid resuscitation Blood cultures Intravenous antibiotics Senior review

As with any unwell patient, the ABC (airway, breathing, circulation) approach should be used. Patients with sepsis should be risk-assessed using Sepsis Guidelines and the Sepsis Bundle of care followed. This patient has high risk features of sepsis (impaired consciousness, high respiratory rate, low blood pressure and a non-blanching rash). Blood cultures should be taken, and patients treated with intravenous antibiotics according to local guidelines as soon as possible and certainly within an hour of arrival at hospital. He should also have a lactate, full blood count, urea and electrolytes, C-reactive protein, blood gas, glucose, and a blood meningococcus/pneumococcus PCR. This patient also requires fluid resuscitation. A lumbar puncture is used to confirm a diagnosis of meningitis, however in this patient with sepsis this investigation should be delayed. Priority is given to management of

the sepsis. Other reasons to delay a lumbar puncture include if there are signs of significantly raised intracranial pressure and brain shift such as GCS < / = 12, focal neurological signs, papilloedema and continuous or uncontrolled seizures. Patients who have hypotension despite initial fluid resuscitation, a reduced level of consciousness or organ failure will need assessment by the Critical Care team. (1e) How would the diagnosis be confirmed here? (3*). Answer: Blood cultures taken before antibiotics should flag as positive within the first 48 h. Microscopy at this point would reveal Gram negative diplococci and hopefully later identified as Neisseria meningitidis. The blood sample sent for PCR should also detect N. meningitidis. 2. A 45-year-old man presents with a 3-week history of fever, drenching night sweats and lethargy. He is normally fit and well, has no known previous medical history and is not on any prescribed or recreational drugs. He points out lesions on his nails, hands and feet which have appeared over the past few days (Fig. 2). (2a) Name these lesions (2*). Answer: . Splinter haemorrhage—red/brown linear lesions found under the nail bed.

Infectious Diseases

333

Fig. 2 Lesions on hands and feet. Permission granted by Hull York Medical School (HYMS). Full and informed consent to use in commercial publication

. Janeway lesion—non-tender macular erythematous lesions occurring on the palmar aspect of the hands and the plantar surface of the feet as a result of septic microemboli. . Osler's node—tender violaceous or erythematous nodules or pustules on the fingers and toes which occur as a result of vasculitis and consequent immune complex deposition.

. Emboli—causing stroke, paralysis from spinal emboli, limb ischaemia, visual defects/blindness from retinal artery emboli and pulmonary emboli. . Metastatic infection – spread of infection, most commonly to lungs, bones, spine and brain. . Renal involvement – glomerulonephritis and renal infarcts causing renal impairment and microscopic haematuria.

(2b) What is the most likely diagnosis? (2*). Answer: Infective endocarditis (IE) On further history he reveals that he was treated for a tooth abscess 2 months previously. On examination he is febrile with a blood pressure of 120/55 and pulse of 85 bpm. He has an early diastolic murmur of aortic regurgitation which is loudest at the left sternal edge and on sitting forward. (2c) How is this condition diagnosed? (2*)

Radiological—transthoracic or transoesophageal echocardiography should be performed as soon as possible. Features of IE include vegetations, abscesses, and valvular regurgitation. Microbiological—three sets of blood cultures at least 6 h apart, should be taken using meticulous aseptic technique prior to the initiation of any antibiotic therapy in suspected IE. Patients with high-risk sepsis as per sepsis guidelines, should have two sets of blood cultures taken at different times within the first hour before starting empiric antibiotics. (2d) What are the organisms most commonly recognised to cause this condition? (4*).

Answer: Answer: Diagnosis of IE is clinical, radiological and microbiological Clinical . Features of valvular dysfunction—including heart failure . Peripheral stigmata—splinter haemorrhages, Janeway lesions, Osler's nodes, Roth spots of the retina, skin petechiae.

. Staphylococcus aureus—this is a skin commensal and is the most common cause of IE. It may be associated with intravenous drug use or healthcare interventions. . Viridans streptococci—these organisms are commonly found in the mouth and are associated with dental infections.

334

J. Allen

. Other streptococci and enterococci. . HACEK group (Haemophilus spp, Aggregatibacter actinomycetemcomitans, Cardiobacterium hominis, Eikenella spp, Kingella spp)—this is a group of fastidious gram-negative organisms. (2e) What are the predisposing conditions and risk factors associated with this condition? (3*) Answer: . . . . . . .

Prosthetic heart valve Pre-existing valvular heart disease Congenital heart defects Intravenous drug use Hypertrophic cardiomyopathy Poor dentition or dental infection Presence of an intravascular device such as a catheter or shunt.

(2f) How is this condition treated? (2*). Answer: Intravenous antibiotics and sometimes valve replacement/ repair. Antibiotic choice is guided by positive culture results, and treatment is normally required for 4–6 weeks. Early surgery is indicated for severe valvular regurgitation causing heart failure, patients with uncontrolled infection, and to prevent emboli in those with large vegetations or who have had emboli despite antibiotics. 3. A 33-year-old man presents with a 2-day history of fever, headache and myalgia. He is originally from Nigeria but has been living in the UK for the past 8 years. He returned a week ago from a trip to Nigeria to visit his family. He did not travel outside of the capital, Lagos. He has no previous medical history and is not on any medication. Examination reveals a fever of 38.9ºC but is otherwise unremarkable. (3a) What is this investigation ( Fig. 3) and why is it performed? (2*).

Fig. 3 Blood Film Centres for Disease Control and Prevention Public Health Image Library, 1971. http://phil.cdc.gov

marker of severity. Antigen tests are used as an alternative where thick and thin films cannot be performed but may remain positive after treatment and do not give a parasitaemia count which is used to assess severity of falciparum malaria. (3b) What is the name of this organism? (3*). Answer: Plasmodium falciparum This is the most severe form of malaria and can be distinguished from the other 4 species of Plasmodium (P. vivax, P. ovale, P. malariae, P. knowlesi) by its small ring forms and the presence of multiple ring forms within some red blood cells. (3c) What is the vector for this disease? (3*). Answer: Malaria is transmitted by the bite of the female Anopheles mosquito (3d) What are the complications of this condition? (3*).

Answer: Answer: This is a thin blood film to detect malaria. This is the gold standard investigation for malaria and is performed alongside a thick blood film. Thick films allow a larger quantity of blood to be examined and are more sensitive, while thin films allow species identification and calculation of the percentage of parasitised red blood cells, a

. Acute kidney injury—due to dehydration, hypovolaemia or sometimes acute tubular necrosis. . Hypoglycemia—this may be exacerbated by quinine treatment. . Metabolic acidosis—as a result of hypoperfusion.

Infectious Diseases

335

. Seizures - from cerebral malaria, metabolic disturbance or hypoglycaemia. . Severe anaemia—resulting from red cell lysis, autoimmune haemolysis and disturbed bone marrow function. . Disseminated intravascular Coagulation—may cause bleeding gums, petechiae or more overt bleeding. . Concomitant bacterial infection—unusual but typically from gram negative organisms. . Acute respiratory distress syndrome—associated with high mortality.

(4a) Describe the abnormalities on this chest X-ray (2*).

(3e) What other infection screening would be indicated in this man? (3*).

Answer:

Answer: There are bilateral lung infiltrates involving all lobes of the lung, but more marked in the apices, with cavities visible in both the right upper and lower zones. (4b) What is the most likely diagnosis given this history and chest X-ray? (2*).

Pulmonary tuberculosis Answer: HIV Testing While malaria can be more severe in individuals with HIV, testing should be routinely offered and recommended in all individuals from a country of high prevalence (>1%) regardless of their presentation. 4. This 28-year-old lady from Lithuania presents with a 3-month history of cough productive of blood stained sputum, fever, night sweats and weight loss. On auscultation of the lungs, she has scattered bilateral crepitations. She is afebrile with a normal respiratory rate and oxygen saturation. A chest X-ray is performed (Fig. 4).

Fig. 4 Chest X-ray showing pulmonary changes. Permission granted by Hull York Medical School (HYMS)

The cavities and apical predilection are typical of pulmonary tuberculosis (TB) caused by Mycobacterium tuberculosis complex (MTBC). The presentation here is more insidious than would normally be expected from bacterial pneumonia. TB should be considered in any patient with a cough lasting more than 3 weeks, and particularly in a patient originating from an area of high incidence such as Eastern Europe. (4c) How is this diagnosis confirmed? (3*). Answer: Sputum microscopy and culture Three sputum samples should be obtained over a period of 1–3 days. Patients without a productive cough may have sputum induced by nebulised hypertonic saline with physiotherapy or bronchoalveolar lavage samples obtained by bronchoscopy. Acid-fast staining (auramine-phenol or Ziehl–Neelsen) and microscopy is performed on sample smears to detect acid-fast bacilli (AFB), which in this scenario would strongly suggest tuberculosis. However, AFB will only be seen on microscopy if there is a high number of bacilli present. All AFB smear-positive and negative samples are therefore cultured for mycobacteria. Once grown this allows speciation of the mycobacterium to confirm MTBC over non-tuberculous mycobacteria and also allows drug susceptibility testing to be carried out. Nucleic acid amplification testing (NAAT) is a method based on the polymerase chain reaction (PCR) and can be used to detect MTBC as well as genes associated with drug resistance, while awaiting culture of the organism. This is used when more rapid diagnosis is required, or drug resistance is suspected.

336

(4d) How is this condition managed? (3*). Answer: Standard treatment of drug-sensitive pulmonary tuberculosis is 2 months of quadruple therapy (Rifampicin, Isoniazid, Pyrazinamide and Ethambutol) followed by a further 4 months of Rifampicin and Isoniazid. Pyridoxine (vitamin B6) is given to reduce the risk of Isoniazid-induced peripheral neuropathy. All cases of TB should be notified to Public Health and contact tracing carried out. Smear-positivity is associated with increased risk of transmission.

J. Allen

chemotherapy 3 months previously. He lives alone but has family who live nearby. He reports that his two grandchildren were unwell two weeks previously. (5a) What is the diagnosis and causative organism? (3*). Answer: Disseminated varicella caused by varicella-zoster virus (VZV). (5b) Describe the clinical syndromes associated with this organism (2*). Answer:

(4e) Name 4 risk factors for drug-resistance in this condition (5*). Answer: . . . .

Previous TB treatment Contact with a known case of drug-resistant TB HIV infection Birth or residence in a country with high rates of resistance.

5. A 65-year-old man presents with a 4-day history of fever, followed by a widespread, itchy rash of 2 days duration (Fig. 5). He has a previous medical history of non-Hodgkin’s lymphoma for which he completed

VZV causes two distinct forms of disease: . Varicella (chickenpox)—primary VZV which normally occurs in early childhood and is characterised by widespread vesicular lesions affecting the face, then spreading to the torso and extremities. In this patient older crusted lesions can be seen alongside new vesicular lesions on the torso. Disseminated varicella occurs in patients with underlying malignancy, those on immunosuppressive treatment or steroids, patients with HIV infection, or those who have had solid organ transplantation. . Herpes zoster (shingles)—reactivation of latent VZV, often during immunosuppression, in concurrent illness or in the elderly, causing painful, normally dermatomal distribution of vesicles. (5c) How is this condition diagnosed? (3*). Answer: The diagnosis of varicella zoster is clinical. However, if the presentation is atypical confirmation is required, PCR of the vesicular fluid can be performed. (5d) What are the complications of this condition? (3*). Answer: Complications are rare in children but can occur in adults with varicella and those with immunocompromise:

Fig. 5 Widespread itchy rash in a febrile patient. Permission granted by Hull York Medical School (HYMS). Full and informed consent to use in commercial publication

. Secondary bacterial skin infection . Neurological—encephalitis, meningitis, myelitis, vasculitis . Pneumonia—this is the predominant varicella-related mortality.

transverse cause

of

Infectious Diseases

337

. Hepatitis—ranges from mild transaminitis to fatal disease. (5e) How is this condition acquired and how can it be prevented? (2*). Answer: Primary infection can be acquired by an individual without a previous history of VZV infection via aerosolized droplets of nasopharyngeal secretions from a patient with varicella or by direct contact with vesicle fluid from a patient with either varicella or zoster. The incubation period is 10–21 days and patients are infective from 2 days prior to the onset of rash until all the skin lesions have crusted over. It is a highly contagious infection which will cause disease in up to 90% of susceptible household contacts. Disease can be particularly severe in pregnant or immunocompromised adults so contact should be strictly avoided. Those who have been exposed may require varicella zoster immunoglobulin or oral aciclovir. The VZV vaccine is a live vaccine so cannot be given to either of these groups. However, its use is recommended in susceptible healthcare-workers and household contacts of people who are significantly immunosuppressed. 6. A 29-year-old man presented with extensive maculopapular rash on trunk, palms and soles for the last 7 days (Fig. 6). He denies any fevers or systemic upset other than some mild fatigue. He reports having receptive oral and anal sex with a casual male contact 2 months ago and on direct questioning recalls a small ulcer on his penis appearing around a month ago which has now resolved. His blood tests show alanine aminotransferase (ALT) 256 but are otherwise are unremarkable. His point of care HIV test is negative. (6a) What is the most likely diagnosis? (3*).

Fig. 6 Extensive maculopapular rash on trunk, palms and soles. Photo credit Dr. Uday Joshi, Consultant Genitourinary Physician, Hull University Teaching Hospitals NHS Trust

However, these conditions would not account for the genital ulcer in this patient.

Answer: (6b) How is this condition acquired? (3*). Secondary syphilis Answer: Other causes of a similar rash with a transaminitis would include a drug reaction, primary HIV infection (HIV seroconversion) and Infectious Mononucleosis. Erythema multiforme can cause a similar appearance particularly to the palm lesions shown.

This infection is caused by the transmission of the spirochete bacterium Treponema pallidum, usually by direct sexual contact with an infectious lesion.

338

Most new sexual transmission is currently amongst men who have sex with men (oral, anal and genital) though heterosexual transmission is also common. Vertical transmission can occur at any stage of pregnancy causing congenital syphilis. Intravenous drug use and transmission via blood products is unusual though possible. (6c) How would you confirm the diagnosis? (4*). Answer: Syphilis is confirmed using serology. However, serology cannot differentiate between T. pallidum and other endemic treponematoses (Yaws, Bejel and Pinta). Treponemal-specific tests e.g., Treponema pallidum haemagglutination test (TPHA), T. Pallidum particle agglutination (TPPA) or Enzyme immunoassay test (EIA) are used as screening tests. An anti-treponemal IgM test should be used if primary syphilis is suspected. All positive screening tests should be confirmed using a different confirmatory specific test. Non-specific quantitative serological tests are used to assess serological activity of syphilis which help determine the stage of the infection as well as the response to treatment. These include the rapid plasma reagin (RPR) or the venereal disease research laboratory (VDRL). An initial titre of 16 or more usually indicates active disease. Syphilis can also be confirmed from demonstration of T. pallidum from lesions such as the primary syphilitic ulcer (chancre) using dark ground microscopy in clinics experienced with its use, or via PCR on lesion swabs.

J. Allen

present, usually in the anogenital region. There may be hepatitis, splenomegaly, glomerulonephritis or neurological complications. These include acute meningitis, cranial nerve palsies, uveitis, optic neuropathy, interstitial keratitis and retinal involvement. Latent disease—Secondary syphilis usually resolves within 3–12 weeks as the disease enters the latent phase which may last 20–40 years. Late (tertiary) disease—occurs in around a third of untreated patients. Gummatous disease—inflammatory granulomatous destructive lesions can occur in any organ but most commonly affects the skin and bones. Cardiovascular disease—includes aortitis, aortic regurgitation, heart failure, coronary ostial stenosis, angina and aneurysms Neurological disease—meningovascular disease, general paresis, tabes dorsalis. (6d) How would you manage this condition? (4*). Answer: Early syphilis (primary, secondary and early latent) is managed with a single dose of intramuscular benzathine penicillin. Late syphilis (late latent, cardiovascular and gummatous) —intramuscular benzathine penicillin weekly for 3 doses. Neurosyphilis—intramuscular procaine penicillin daily and oral probenecid for 14 days or intravenous benzylpenicillin 4 hourly for 14 days. Alternative regimens are recommended in local and national guidelines.

(6d) What are the stages of this disease? (4*) Answer: Primary syphilis—has an incubation of around 21 days (range 9–90 days) and manifests as a chancre. This is usually, but not always painless and single and resolves over 3–8 weeks. Secondary syphilis—will develop in around 25% cases if left untreated 4–10 weeks after the initial chancre. Clinical manifestations include a widespread mucocutaneous rash which is sometimes itchy and can affect the palms, soles, mouth and genital mucosa. Condylomata lata are highly infectious flat-topped papules and plaques that may be

7. This 76-year-old man presents to the emergency department with a 4-week history of thoracic back pain which is constant, severe and disturbing his sleep. For the past week he has had fever and anorexia and today he has noticed his legs are weak. He is passing urine and opening his bowels normally. He has a previous history of gastrointestinal bleed requiring 5-day hospitalisation 2 months previously. On examination he is febrile and in pain. He has lower thoracic vertebral tenderness, bilateral lower limb increased tone and bilateral upgoing planters. Bloods show his C-reactive protein is 220 ( 38 °C, tubo-ovarian abscess or pelvic peritonitis) • Surgical emergencies cannot be excluded

Fig. 6 Trichomonas Infection—typical discharge and appearance of the cervix on pelvic examination. Reprinted by permission from Springer Nature. Candidiasis, Bacterial Vaginosis, Trichomoniasis and Other Vaginal Conditions Affecting the Vulva by Pedro Vieira-Baptista, Jacob Bornstein. Copyright © 2019

384

1. What is the most likely provisional diagnosis? (1*) 2. What is the most appropriate management? (2*) Answer: 1. The findings are typical of a trichomonas infection. Trichomonas vaginalis (TV) is a flagellated protozoan and is transmitted sexually. It is also associated with preterm delivery and low birth weight predisposes to maternal postpartum sepsis. However, there is conflicting evidence for treatment in pregnancy and screening is not recommended. Half of the patients are asymptomatic but vaginal discharge, vulval itching, dysuria, or offensive odor are common. Low abdominal discomfort or vulval ulceration is seen occasionally. One-third of patients have the classical frothy yellow vaginal discharge. Other signs include uvulitis and vaginitis, and strawberry cervix can be found only in 2% of patients. Written patient information is important and screening for other STI’s should be undertaken.

P. Palei and U. Rajesh

A. B. C. D. E.

Full blood count. Transvaginal Ultrasound scan. Outpatient hysteroscopy and biopsy. Tranexamic acid. LNG-IUS.

Answer: C Cases: A 45-year-old woman presents with heavy menstrual periods for 5 years. She was treated with a Mirena coil with improvement for the first four years, but periods are now heavier again. She is requesting a hysterectomy as her family is complete. 1. How are heavy menstrual periods (HMB) defined? (1*) 2. What are the possible causes and investigations? (2*) 3. What is the management of HMB? (3*) Answer:

2. The recommended regimen is metronidazole 2 g orally in a single dose or 400–500 mg 12 hourly for 5–7 days. An alternative regimen is metronidazole 2 g orally as a single dose.

Topic 7: Heavy Menstrual Bleeding (HMB) Pre-study questions: 1. A 20-year-old woman presents with heavy periods for 2 years. She had no intermenstrual and postcoital bleeding. Tranexamic acid 500 mg 8 hourly is ineffective. She is currently using combined pills as contraception. The abdominal examination is normal. What is the next most appropriate management? A. Continue current treatment. B. Levonorgestrel Intrauterine System (LNG-IUS). C. Cyclical Oral Progestogens. D. Ibuprofen. E. Hysteroscopy. Answer: B 2. A 38-year-old woman presents with abdominal pain and heavy periods and persistent intermenstrual bleeding for 6 months. Her BMI is 40. She is not using any contraception and is otherwise fit and well. Abdominal examination is inconclusive. What is the next step to confirm the diagnosis?

1. Heavy menstrual bleeding (HMB) is defined as 80 ml or more of blood loss during a period or a period longer than 7 days or both. It is not possible to measure accurately and if a woman feels that her period is heavier than normal, available treatment and options should be discussed with her as it has a major impact on quality of life. 2. Associated possible conditions/causes are myomas, endometrial polyp and endometrial pathology such as typical or atypical hyperplasia. Evaluation of Heavy Menstrual bleeding includes: • History and physical examination. • Full blood count and thyroid function tests • Hysteroscopy & endometrial Biopsy • Transvaginal Ultrasound scan for assessment of endometrium, exclude pathologies such as fibroids, adnexal masses, or polyps • Investigations and management vary depending on: – Patient’s age – Associated symptoms dysmenorrhea/dyspareunia/ Intermenstrualbleeding/premenstrual syndrome – Need for contraception – History – Preferences – Underlying pathology if any • The desire for future pregnancy. 3. Management Options Include: • Pharmacological treatments such as acid and tranexamic acid – Mefenamic (non-hormonal preparations; these are usually

Obstetrics & Gynecology





• •

considered as first-line measures and in the primary care setting by General practitioners). – Norethisterone, progesterone only or combined oral contraceptive pills (these are hormonal oral options that are indicated as contraceptive choices along with management of period problems or as stand-alone options. Careful counseling about common and rare side effects as well as the exclusion of contraindications is essential). Mirena (Levonorgestrel) intrauterine System is an excellent second-line agent. It is effective in more than 80% of patients. It is licensed for use in the management of menorrhagia as well as an excellent contraceptive choice. Endometrial Ablation (outpatient under local anesthetic or day surgery procedure under general anesthetic. Only useful as a second-line option in patients who have completed their family and have other suitable contraception sorted. Uterine artery embolization or Myomectomy as appropriate in selected patients with uterine fibroids. Abdominal, Vaginal or Laparoscopic Hysterectomy when non- hormonal, hormonal, Mirena or endometrial ablation is ineffective.

Further Reading 1. Antenatal screening Guidance, Public Health England. https:// www.gov.uk/government/publications/handbook-for-sickle-celland-thalassaemia-screening/antenatal-screening. 2. NICE Guidelines (CG62) for uncomplicated Antenatal Care Updated 2019. https://www.nice.org.uk/guidance/cg62. 3. NIPT guidance, Public Health England 2021. 4. Hypertension in Pregnancy: Diagnosis and Management NICE guidelines, June 2019. 5. Severe Pre-eclampsia/Eclampsia, Management. RCOG Green-top Guideline No. 10A, March 006.

385 6. The Magpie Trial Collaborative Group. The Magpie Trial: A Randomized placebo-controlled trial. Lancet. 2002;359:1877–90. 7. Screening for Down’s syndrome, Edwards’ syndrome and Patau’s syndrome: NIPT-GOV.UK 8. Hypertension in Pregnancy: Diagnosis and Management NICE guidelines, June 2019. 9. Severe Pre-eclampsia/Eclampsia, Management. RCOG Green-top Guideline No. 10A, March 2006. 10. The Magpie Trial Collaborative Group. The Magpie Trial: A Randomized placebo-controlled trial. Lancet. 2002;359:1877–90. 11. Care of women presenting with suspected preterm prelabour rupture of membrane from 24+0 week of gestation. RCOG Green-Top Guideline No. 73 June 2019 12. Kenyon SL, Taylor DJ, Tarnow-Mordi W, ORACLE Collaborative Group Broad-spectrum antibiotic for spontaneous preterm labor, the ORACLE II randomized trial. ORACLE Collaborative Group. Lancet. 2001; 357:989–94 13. Preterm labor and birth NICE guideline Updated Aug 2019 14. Magnesium sulfate for fetal neuroprotection at imminent risk for preterm delivery: a systematic review with meta-analysis and trial sequential analysis May 2017; https://www.thelancet.com/pdfs/ journals/lancet/PIIS0140-6736(02)08778-0.pdf. 15. Roberts D, Dalziel SR. Antenatal corticosteroids for accelerating fetal lung maturation for women at risk of preterm birth. Cochrane Database of Systematic Reviews 2006. 16. Crowther CA, Hiller JE, Doyle LW, Haslam RR for the Australasian Collaborative Trial of Magnesium Sulfate (ACTOMgS) Collaborative Group. Effect of magnesium sulfate given for neuroprotection before preterm birth. JAMA 2003; 290 (20):2669–76. 17. NHS England (2016). Saving Babies’ Lives: a care bundle for reducing stillbirth. Available From: https://www.england.nhs.uk/ wpcontent/uploads/2016/03/saving-babies-lives-car-bundl.pdf. 18. NHS England (2019) Saving Babies’ Lives Version Two: A care bundle for reducing perinatal mortality. 19. UK National Guideline for the Management of Pelvic Inflammatory Disease (2019 Interim Update). 20. Ectopic pregnancy and miscarriage: diagnosis and initial management NICE April 2019. 21. Diagnosis and Management of Ectopic Pregnancy Green-top Guideline No. 21/ RCOG and EPAU Joint Guideline November 2016. 22. UK National Guideline on the Management of Trichomonas vaginalis 2014. https://www.bashh.org/guidelines. 23. Heavy menstrual bleeding: assessment and management NICE May 2021.

Immunology Pavels Gordins

Glossary

FceRI HAE IgE MRGPRX2 PAF

Pre-study Questions High-affinity IgE receptor Hereditary Angioedema Immunoglobulin E Mas-related G-protein coupled receptor member X2 Platelet Activating Factor

Learning Outcomes • Recognize common triggers, symptoms and signs of allergic rhinitis and describe general approach to its management. • Describe the mechanisms causing prominent local reaction to insect stings and know how to treat it. • Describe the inheritance pattern of hereditary angioedema, recognize the symptoms and signs and be able to provide emergency treatment. • Describe the common triggers of seasonal allergic rhinoconjunctivitis and the therapeutic options available to manage this condition. • Describe the role of skin prick testing and measuring specific IgE antibodies in identifying relevant allergens and discuss their significance in the context of clinical history. • Be able to interpret laboratory diagnosis of acute anaphylactic reaction and institute lifesaving treatment.

P. Gordins (&) Department of Immunology, Hull University Teaching Hospitals NHS Trust, Hull, England e-mail: [email protected]

MCQs 1. A 28-year-old man attends an allergy clinic complaining of itchy and swollen lips and mouth, uncomfortable throat, stomach pain and nausea after eating fresh apples and stone fruits since early childhood. He can tolerate the same fruits in cakes and pies without any reactions. He also reports a history of early spring hay fever with symptoms of sneezing, rhinorrhea, itchy eyes and occasional chest tightness and wheeze. Which of the following is the most likely diagnosis? A. Food anaphylaxis B. Irritant contact stomatitis C. Oral allergy syndrome D. Burning mouth syndrome E. Hereditary angioedema Answer: C Oral allergy syndrome, also known as pollen food syndrome, is due to cross-reactivity between pollen allergens and similar allergens present in foods of plant origin such as PR-10 protein and profilin, and is very rarely associated with systemic allergic reactions or anaphylaxis. 2. A 22-year-old woman presents with symptoms of swollen lips and tongue, tight throat and chest as well as widespread urticarial rash approximately 10 min after eating an Indian meal at a local restaurant. She is known to be allergic to peanuts but can tolerate all common tree nuts. She always asks for nut-free food when eating out and had no problems with food she ordered from the same restaurant in the past. The restaurant has been recently acquired by a new owner and underwent

© The Author(s), under exclusive license to Springer Nature Switzerland AG 2023 K. Wong et al. (eds.), Practical Guide to Visualizing Medicine, https://doi.org/10.1007/978-3-031-24465-0_21

387

388

P. Gordins

extensive renovations before re-opening two weeks ago. What is the most likely cause for the above reaction? A. Toxic reaction to paint or other building materials used during recent renovation B. Acute attack of spontaneous urticaria and angioedema C. Monosodium glutamate poisoning D. Unintended food cross-contamination with peanut in restaurant’s kitchen E. Capillary leak syndrome Answer: D Cross-contamination is one of the most common causes of food anaphylaxis when eating in bars, restaurants or ordering takeout food. The risk is particularly high when eating in unfamiliar places or if there was a change in management at the previously known place. 3. Which is the key mechanism responsible for mast cell degranulation during acute allergic response to food? A. Cross-linking of IgE-FceRI complex by food antigens B. Activation of PAF (Platelet Activating Factor) receptors C. Direct mast cell activation by food antigens D. MRGPRX2 receptor-mediated mast cell activation E. None of the above Answer: A Cross-linking of IgE (immunoglobulin E)-FceRI (high-affinity IgE receptor) complex on the surface of mast cells by food antigens (allergens) is the key mechanism which results in mast cell degranulation during acute allergic reaction. Main Section: 1. A 12-year-old boy presents with a history of persistent nasal blockage, rhinorrhea and sneezing since early childhood; symptoms are all year round but tend to exacerbate in autumn and winter (Fig. 1). a. Describe the clinical signs as seen in the figure (2*) b. What are the most common causes of perennial allergic rhinitis with exacerbations during the colder part of the year? (2*) c. What are the commonly prescribed treatments for allergic rhinitis? (3*)

Fig. 1 Facial features in allergic rhinitis. Republished with permission of SLACK Inc. with permission conveyed through Copyright Clearance Center, Inc. (Scadding GK, Smith PK, Blaiss M, et al. Allergic Rhinitis in Childhood and New EUFOREA Algorithm. Front Allergy. 2021; 2:706589. https://doi.org/10.3389/falgy.2021.706589)

Answer: The child is noted to perform so-called “allergic salute”, looks pale with dark circles beneath the eyes, double eye creases, loss of lateral eyebrow and transverse nasal crease. All these signs can be seen in chronic allergic conditions of the upper airways such as perennial allergic rhinitis. The most common causes of perennial allergic rhinitis in temperate climates include allergic sensitization to indoor allergens such as house dust mites and animal dander as well as molds. Perennial allergies tend to exacerbate during the colder months of the year when people spend more time indoors. The commonly prescribed medications for allergic rhinitis (both seasonal and perennial) include oral and topical antihistamines, steroid nasal drops and sprays as well as allergen immunotherapy. Short courses (up to 5 days) of systemic steroids can be prescribed under specialist guidance when symptoms are particularly troublesome and do not respond to standard therapy. 2. A 17-year-old girl has been stung by a wasp on the inside of her right upper arm while gardening. She contacts her general practitioner to report that her arm remains swollen, hot and painful two days later as seen in Fig. 2. a. Describe the clinical signs (2*) b. What are the likely mechanisms for developing large local reactions to insect sting? (3*) c. How would you manage prominent local reaction to insect stings? (3*)

Immunology

389

Fig. 2 Redness and swelling of arm following insect sting

Answer: There is a prominent local reaction in the form of redness (erythema) and swelling at the sting site which extends to involve a large part of the upper arm. Such prominent local reactions are often associated with localized warmth and intense itching but can also be unusually painful. It is important to monitor for signs of systemic reaction such as itching and hives in other areas, abdominal pain, cramps, nausea, and diarrhea as well as chest tightness, difficulty in breathing or swallowing, hoarse voice, palpitations and dizziness. Systemic reactions usually occur within several minutes of the sting. Almost all individuals will develop a limited and short-lived local reaction when stung by wasp or bee and only a minority will present with large local reactions which are described as an area of swelling exceeding 10 cm in diameter, usually peaking between 24 to 48 h and lasting for up to 5 days. The mechanisms responsible for large local reactions are likely to include late-phase IgE mediated reactions, cell-mediated inflammatory responses, or a combination of both mechanisms. Treatment of large local reactions to insect stings is symptomatic and includes topical applications of ice pack or cold compress, oral antihistamines and occasionally a short course of oral steroids. History of large local reactions to insect stings is associated with a very low risk of systemic reactions to future stings and therefore long-term possession of epinephrine auto-injector and venom desensitization therapy (immunotherapy) would not be normally recommended. 3. A 22-year-old woman presents to the emergency department with a painful swelling (angioedema) of the right hand which developed approximately one hour earlier for no specific reason as seen in Fig. 3. She was given an intravenous injection of antihistamine and steroid by paramedics with minimal change in her symptoms; she is healthy, takes no regular medications and denies taking any painkillers or anti-inflammatory medications in the last 48 h; her breathing is not affected

Fig. 3 Hand swelling during attack of hereditary angioedema. This is an open access image distributed under the terms of the Creative Commons CC BY license, which permits unrestricted use, distribution, and reproduction in any medium, provided the original work is properly cited (Kanani A, Betschel SD, Warrington R. Urticaria and angioedema. Allergy Asthma Clin Immunol. 2018; 12;14(Suppl 2):59. https:// doi.org/10.1186/s13223-018-0288-z)

and she is feeling generally well; she has had similar episodes of soft tissue swellings affecting her lips, face, arms and legs in the past but has not been able to identify specific triggers; there is a history of similar episodes of swellings in her paternal grandmother, father, sister and brother. a. Discuss the likely inheritance pattern (4*) b. What is the likely diagnosis? (2*) c. How would you manage acute symptoms in the context of emergency room admission? (3*) Answer: The condition which equally likely affects men and women in every generation is very likely to be inherited in an autosomal dominant pattern. Recurrent attacks of swellings (angioedema) with no urticaria occurring for no specific reason and affecting several members of the same family would be highly suggestive of hereditary angioedema (HAE). HAE patients can also develop significant swellings of the upper airways which can be life-threatening as well as episodes of abdominal pain due to bowel edema masquerading as acute abdomen. Diagnosis of classical HAE is based on detecting deficiencies in complement systems including C1 esterase inhibitor and complement C4 which is almost universally low or undetectable during acute attacks. HAE symptoms are due to excessive accumulation of potent endogenous vasodilator bradykinin and would therefore not respond to antihistamine and steroid therapy which is routinely used to treat classical allergic reactions.

390

There is also usually no response to epinephrine. The treatment of acute HAE attacks includes administration of plasma-derived or recombinant products restoring C1 esterase inhibitor level and/or function to normal or near normal levels and the use of medications targeting plasma kallikrein system or blocking bradykinin receptors. 4. A 25-year-old woman reports seasonal symptoms of intense sneezing, rhinorrhea, watery and itchy eyes not responding to standard antihistamines and steroid nasal sprays during summer months from May till July; to control her symptoms, she has been previously prescribed short courses of systemic steroids by her general practitioner. a. Describe results of skin prick test performed in the clinic (Fig. 4) (3*) b. What are the most common triggers of seasonal allergic rhino-conjunctivitis? (2*) c. What are the therapeutic options to manage seasonal allergic rhino-conjunctivitis? (3*) Answer: Skin prick test performed with commercial extracts of common aeroallergens shows significant weal and flare response to Timothy grass which is a common grass native to most of Europe as well as a naturalized grass in North America. Timothy grass as well as rye grass pollen are primarily responsible for hay fever symptoms occurring between May and July. Other common triggers for seasonal

Fig. 4 Skin prick test with common aeroallergens on forearm

P. Gordins

allergic rhino-conjunctivitis include tree (birch, hazel and alder) pollen (March to May) and weed (mugwort and ragweed) pollen (July to September). Allergen avoidance, oral antihistamines and topical (intranasal) steroids are effective first-line therapies to treat seasonal allergic rhino-conjunctivitis. Allergen immunotherapy can be recommended to individuals with suboptimal response to standard pharmacological therapy. To achieve maximum efficacy, immunotherapy to seasonal allergens is commonly started in winter months and involves receiving monthly injections or daily tablet/oral solution placed under the tongue. The treatment is usually continued for three consecutive years. 5. A 35-year-old woman developed severe hypotension, prominently swollen face, and widespread itchy urticarial rash approximately 5 min after she was given an intravenous injection of co-amoxiclav following emergency cesarean section. Her symptoms improved rapidly following intravenous administration of epinephrine. Her blood samples showed mast cell tryptase levels as seen in Fig. 5 a. Discuss the observed changes in mast cell tryptase level and the likely diagnosis?( 4*) b. What advice would you give to the patient? (3*) Answer: The observed changes in mast cell tryptase level would be highly suggestive of acute anaphylactic reaction. The 24-h tryptase level is within reference range and this would essentially rule out primary mast cell disorders such as systemic mastocytosis. The patient should avoid beta-lactam antibiotics including penicillin, cephalosporins and carbapenems until investigated in the drug allergy clinic.

Fig. 5 Mast cell tryptase levels at 30 min, 1 h and 24 h during acute anaphylactic reaction

Immunology

Fig. 6 Results of allergy blood test to common foods in patient with atopic dermatitis (normal range for total IgE is 17 to 80 U/mL and specific IgE level above 0.35 represents positive result)

6. A 25-year-old lady with severe atopic dermatitis since teenage years has had allergy tests performed in primary care. She was found to have detectable specific IgE antibodies to several food allergens including wheat, cows’ milk, soybean, peanut and egg white (Fig. 6). Her total IgE level is also significantly elevated. She reports no immediate symptoms to any of the above foods on consumption. a. Discuss the clinical significance of the allergy blood tests (3*) b. What other tests can be used to differentiate true food allergy from asymptomatic sensitization? (4*) c. What advice would you give to the patient? (3*) Answer: Markedly elevated total IgE level is frequently seen in patients with atopic dermatitis and this can lead to falsely positive IgE antibodies to specific foods on allergy blood

391

tests. The clinical significance of the positive blood (or skin) test to food should always be interpreted in the context of clinical history including immediate or delayed allergic reaction following food consumption. In patients with atopic dermatitis random and indiscriminate screening for food allergies would be generally discouraged. Food challenge is the ultimate test to confirm or refute the diagnosis of food allergy where the specific food has already been avoided for a significant period of time. Food challenge should be only performed under medical supervision in low risk patients. Elimination diets can be considered where specific food is still consumed. The elimination phase would be normally followed by gradual and controlled reintroduction of eliminated food(s) and careful monitoring of symptom activity. If the foods which tested positive on the allergy blood or skin tests are already consumed on a regular basis without obvious adverse symptoms, the patient should be advised to continue eating all those foods until discussed with a qualified allergy or immunology specialist experienced in diagnosing and managing food allergy. There is always a risk that unnecessary and long-term avoidance of multiple food groups can lead to significant nutritional deficiencies.

Further Readings 1. Spickett G (2020) Oxford Handbook of Clinical Immunology and Allergy, 4th ed 2. Middleton E, Burks A, Holgate S,O'Hehir R, Broide D, Bacharier L, Khurana Hershey G, Peebles R (2020) Middleton's Allergy, 9th ed 3. Rich RR, Fleisher TA, Weyand C (2019) Clinical Immunology, 5th ed 4. Practice Parameters and Guidelines AAAAI. In: Aaaai.org. https:// www.aaaai.org/Allergist-Resources/Statements-PracticeParameters/Practice-Parameters-Guidelines. Accessed 31 Jan 2022 5. Guidelines-BSACI. In: BSACI. https://www.bsaci.org/guidelines/. Accessed 31 Jan 2022 6. EAACI Media Library. In: Medialibrary.eaaci.org. https:// medialibrary.eaaci.org/. Accessed 31 Jan 2022

Hematology Raymond Dang and James Bailey

Learning Objectives • Be familiar with components of a full blood count and reference ranges of individual cells and indices. • Be able to recognize normal blood cells under the microscope. • Be able to recognize morphological features of some common hematological conditions. • Be able to recognize that many non-hematological conditions have blood manifestations.

Basic Blood Count and Morphology Reference ranges for the full blood count (FBC) of an adult may differ between the sexes (note neonatal and pediatric reference ranges are different). When standard FBC is performed in a hospital Hematology laboratory, using any of a number of analyzers from various manufacturers, the following results are usually reported: Hemoglobin (Hb) Males

g/L (130–180)

Females

g/L (115–165)

Red cell count Males

1012/L (4.3–5.9)

Females

1012/L (3.5–5.0)

Males

(0.40–0.52)

Females

(0.36–0.47)

Mean cell volume (MCV)

fL (80–96)

White blood cells (WBC)

109/L (4–11)

Neutrophils

109/L (2.0–7.0)

Lymphocytes

109/L (1.5–4.0)

Monocytes

109/L (